Sunteți pe pagina 1din 148

Multistate Bar Examination

Questions 1992
See Caveat Inside

®
PREFACE
The Multistate Bar Examination (MBE) is an objective six-hour
examination developed by the National Conference of Bar Examiners
(NCBE) that contains 200 questions. It was first administered in
February 1972, and is currently a component of the bar examination
in most U.S. jurisdictions.

caveat!
The 581 questions contained in this document appeared on
MBEs administered between 1972 and 1991. Because of
their dated nature, many of the questions may test principles
that have been altered by changes in the law and thus are no
longer suitable topics to be tested. As a result, some of the
answers shown in the answer key may be incorrect under
currently accepted principles of law. Further, many of these
questions do not reflect the current style of MBE questions, and
a number of the questions appear in formats that are no longer
used on the MBE.

The questions and answers in this document are provided


only for the purpose of providing applicants with a sample
of the range and general format of questions that appeared
on previously administered MBEs, not as examples of the
content currently tested or of the material to be studied for
the substance of the examination. Many of these questions are
currently in use, sometimes with alteration, by commercial bar
review courses under a licensing arrangement with NCBE.
Because these questions are available in the marketplace,
NCBE is choosing to make them available online.

DO NOT USE THESE QUESTIONS TO STUDY CONTENT


FOR THE MULTISTATE BAR EXAMINATION!!

Applicants are encouraged to use as study aids the MBE


Online Practice Examinations 1 and 2, both available for purchase
online at www.ncbex2.org/catalog. These study aids, which
include explanations for each option selected, contain questions
from more recently administered MBEs that more accurately
represent the current content and format of the MBE.

Copyright © 1992 by the National Conference of Bar Examiners.


All rights reserved.
MULTISTATE BAR EXAMINATION
Questions 1992

TABLE OF CONTENTS

Directions . . . . . . . . . . . . . . . . . . . . . . . . . . . . . . . . . . . . . 2

Questions . . . . . . . . . . . . . . . . . . . . . . . . . . . . . . . . . . . . . . 3

Answer Key . . . . . . . . . . . . . . . . . . . . . . . . . . . . . . . . . 143


Directions

Each of the questions or incomplete statements below is followed


by four suggested answers or completions. You are to choose the
best of the stated alternatives. Answer all questions according to the
generally accepted view, except where otherwise noted.

For the purposes of this test, you are to assume that Articles 1 and 2
of the Uniform Commercial Code have been adopted. You are also
to assume relevant application of Article 9 of the UCC concerning
fixtures. The Federal Rules of Evidence are deemed to control.

The terms “Constitution,” “constitutional,” and “unconstitutional”


refer to the federal Constitution unless indicated to the contrary.

You are also to assume that there is no applicable community


property law, no guest statute, and no No-Fault Insurance Act unless
otherwise specified. In negligence cases, if fault on the claimant’s
part is or may be relevant, the statement of facts for the particular
question will identify the contributory or comparative negligence
rule that is to be applied.

2
Questions

1. Davis decided to kill Adams. He set out for in the head. Noting that Lender was getting the
Adams’s house. Before he got there he saw better of the fight, Mann pointed a gun at Lender
Brooks, who resembled Adams. Thinking that and said, “Stop, or I’ll shoot.” If Lender asserts
Brooks was Adams, Davis shot at Brooks. The a claim against Mann based on assault, will
shot missed Brooks but wounded Case, who was Lender prevail?
some distance away. Davis had not seen Case.
(A) Yes, because Mann threatened to use
In a prosecution under a statute that proscribes deadly force.
any attempt to commit murder, the district (B) Yes, unless Mann was related to Borrower.
attorney should indicate that the intended (C) No, if it was apparent that Lender was
victim(s) was/were about to inflict serious bodily harm upon
Borrower.
(A) Adams only. (D) No, because Lender was the original
(B) Brooks only. aggressor by threatening Borrower with a
(C) Case only. battery.
(D) Adams and Brooks.
4. Peter sued Don for breach of contract. The
2. A state statute requires any person licensed court admitted testimony by Peter that Don
to sell prescription drugs to file with the State and his wife quarreled frequently, a fact of no
Board of Health a report listing the types and consequence to the lawsuit. Don seeks to testify
amounts of such drugs sold if his or her sales in response that he and his wife never quarreled.
of such drugs exceed $50,000 during a calendar The court
year. The statute makes it a misdemeanor to
“knowingly fail to file” such a report. (A) must permit Don to answer if he had
objected to Peter’s testimony.
Nelson, who is licensed to sell prescription (B) may permit Don to answer, whether or not
drugs, sold $63,000 worth of prescription drugs he had objected to Peter’s testimony.
during 1976 but did not file the report. Charged (C) may permit Don to answer only if he had
with committing the misdemeanor, Nelson objected to Peter’s testimony.
testifies that he did a very poor job of keeping (D) cannot permit Don to answer, whether or
records and did not realize that his sales of not he had objected to Peter’s testimony.
prescription drugs had exceeded $50,000. If the
jury believes Nelson, he should be found
Questions 5–7 are based on the following fact
(A) guilty, because this is a public welfare situation.
offense.
(B) guilty, because he cannot be excused on Ames had painted Bell’s house under a contract which
the basis of his own failure to keep proper called for payment of $2,000. Bell, contending in good
records. faith that the porch had not been painted properly,
(C) not guilty, because the statute punishes refused to pay anything.
omissions and he was not given fair
warning of his duty to act. On June 15, Ames mailed a letter to Bell stating, “I
(D) not guilty, because he was not aware of the am in serious need of money. Please send the $2,000
value of the drugs he had sold. to me before July 1.” On June 18, Bell replied, “I will
settle for $1,800 provided that you agree to repaint the
3. Lender met Borrower on the street, demanded porch.” Ames did not reply to this letter.
that Borrower pay a debt owed to Lender, and
threatened to punch Borrower in the nose. A Thereafter Bell mailed a check for $1,800 marked
fight ensued between them. Mann came upon the “Payment in full on the Ames-Bell painting contract
scene just as Lender was about to kick Borrower as per letter dated June 18.” Ames received the check
on June 30. Because he was badly in need of money,

3
Ames cashed the check without objection and spent 8. The requirement that candidates for license
the proceeds but has refused to repaint the porch. must be graduates of barber schools in Aurora is
probably
5. Bell’s refusal to pay anything to Ames when he
finished painting was a (A) unconstitutional as an undue burden on
interstate commerce.
(A) partial breach of contract only if Ames had (B) unconstitutional as a violation of the
properly or substantially painted the porch. privileges and immunities clause of the
(B) partial breach of contract whether or not Fourteenth Amendment.
Ames had properly or substantially painted (C) constitutional, because the state does not
the porch. know the quality of out-of-state barber
(C) total breach of contract only if Ames had schools.
properly or substantially painted the porch. (D) constitutional, because barbering is a
(D) total breach of contract whether or not privilege and not a right.
Ames had properly or substantially painted
the porch. 9. The requirement that candidates for licenses
must be citizens is
6. After cashing the check Ames sued Bell for
$200. Ames probably will (A) constitutional as an effort to ensure that
barbers speak English adequately.
(A) succeed if he can prove that he had painted (B) constitutional as an exercise of the state
the porch according to specifications. police power.
(B) succeed, because he cashed the check (C) unconstitutional as a bill of attainder.
under economic duress. (D) unconstitutional as a denial of equal
(C) not succeed, because he cashed the check protection.
without objection.
(D) not succeed, because he is entitled to 10. Assume that a resident of the state of Aurora
recover only the reasonable value of his was denied a license because she graduated
services. from an out-of-state barber school. Her suit in
federal court to enjoin denial of the license on
7. In an action by Bell against Ames for any this ground would be
provable damages Bell sustained because the
porch was not repainted, Bell probably will (A) dismissed, because there is no diversity of
citizenship.
(A) succeed, because by cashing the check (B) dismissed, because of the abstention
Ames impliedly promised to repaint the doctrine.
porch. (C) decided on the merits, because federal
(B) succeed, because Ames accepted Bell’s jurisdiction extends to controversies
offer by not replying to the letter of June between two states.
18. (D) decided on the merits, because a federal
(C) not succeed, because Bell’s letter of June question is involved.
18 was a counteroffer which Ames never
accepted. 11. Which of the following is the strongest ground
(D) not succeed, because there is no on which to challenge the requirement that
consideration to support Ames’s promise, candidates for barber licenses must have been
if any. residents of the state for at least two years?

(A) The privileges and immunities clause of


the Fourteenth Amendment
Questions 8–11 are based on the following fact (B) The due process clause of the Fourteenth
situation. Amendment
(C) The equal protection clause of the
The State of Aurora requires licenses of persons Fourteenth Amendment
“who are engaged in the trade of barbering.” It will (D) The obligation of contracts clause
grant such licenses only to those who are graduates of
barber schools located in Aurora, who have resided
in the state for two years, and who are citizens of the
United States.

4
12. John was fired from his job. Too proud to (C) No, because the students were not
apply for unemployment benefits, he used his Merrill’s employees.
savings to feed his family. When one of his (D) No, because Merrill did not authorize the
children became ill, he did not seek medical car wash as a class project.
attention for the child at a state clinic because
he did not want to accept what he regarded as
charity. Eventually, weakened by malnutrition,
the child died as a result of the illness. John has 15. Oxnard owned Goldacre, a tract of land, in
committed fee simple. At a time when Goldacre was in
the adverse possession of Amos, Eric obtained
(A) murder. the oral permission of Oxnard to use a portion
(B) involuntary manslaughter. of Goldacre as a road or driveway to reach
(C) voluntary manslaughter. adjoining land, Twin Pines, which Eric owned
(D) no form of criminal homicide. in fee simple. Thereafter, during all times
relevant to this problem, Eric used this road
across Goldacre regularly for ingress and egress
Questions 13–14 are based on the following fact between Twin Pines and a public highway.
situation.
Amos quit possession of Goldacre before
Professor Merrill, in a lecture in her psychology acquiring title by adverse possession. Without
course at a private university, described an experiment any further communication between Oxnard and
in which a group of college students in a neighboring Eric, Eric continued to use the road for a total
city rushed out and washed cars stopped at traffic period, from the time he first began to use it,
lights during the rush hour. She described how people sufficient to acquire an easement by prescription.
reacted differently—with shock, joy, and surprise. Oxnard then blocked the road and refused to
At the conclusion of her report, she said, “You permit its continued use. Eric brought suit to
understand, of course, that you are not to undertake determine his right to continue use of the road.
this or any other experiment unless you first clear it Eric should
with me.” Four of Merrill’s students decided to try the
same experiment but did not clear it with Merrill. (A) win, because his use was adverse to Amos
and once adverse it continued adverse until
One subject of their experiment, Carr, said, “I was some affirmative showing of a change.
shocked. There were two people on each side of the (B) win, because Eric made no attempt
car. At first I thought negatively. I thought they were to renew permission after Amos quit
going to attack me and thought of driving away. Then possession of Goldacre.
I quieted down and decided there were too many dirty (C) lose, because his use was with permission.
cars in the city anyway.” (D) lose, because there is no evidence that he
continued adverse use for the required
Charitable immunity has been abolished in the period after Amos quit possession.
jurisdiction.

13. If Carr asserts a claim against the students who Questions 16–17 are based on the following fact
washed his car, his best theory is situation.

(A) assault. A brother and sister, Bruce and Sharon, acquired as


(B) negligence. joint tenants a 20-acre parcel of land called Greenacre.
(C) invasion of privacy. They contributed equally to the purchase price.
(D) false imprisonment. Several years later, Bruce proposed that they build an
apartment development on Greenacre. Sharon rejected
14. If Carr has a valid claim against the students, the proposal but orally agreed with Bruce that Bruce
will he also prevail against the university? could go ahead on his own on the northerly half of
Greenacre and Sharon could do what she wished with
(A) Yes, if the students would not have the southerly half of Greenacre. Bruce proceeded to
performed the experiment but for build an apartment development on, and generally
Merrill’s lecture. developed and improved, the northerly 10 acres of
(B) Yes, if Carr’s claim against the students is Greenacre. Sharon orally permitted the southerly 10
based on negligence.

5
acres of Greenacre to be used by the Audubon Society (B) An objection by Parr’s attorney on the
as a nature preserve. Bruce died, leaving his entire grounds of the doctor-patient privilege.
estate to his son, Stanley. Bruce’s will named Sharon (C) A finding by the trial judge that Zadok had
as executrix, but she refused to serve. left the office without actually receiving
treatment.
16. In an appropriate action to determine the (D) The assertion of a privilege by Zadok’s
respective interests of Sharon and Stanley in attorney, present at the trial as a spectator
Greenacre, if Stanley is adjudged to be the at Zadok’s request, and allowed by the trial
owner of the northerly 10 acres of Greenacre, judge to speak.
the most likely reason for the judgment will
be that 19. A leading question is LEAST likely to be
permitted over objection when
(A) the close blood relationship between
Sharon and Bruce removes the necessity to (A) asked on cross-examination of an expert
comply with the Statute of Frauds. witness.
(B) Sharon’s conduct during Bruce’s lifetime (B) asked on direct examination of a young
estops her from asserting title to the child.
northern half of Greenacre. (C) asked on direct examination of a
(C) the joint tenancy was terminated by the disinterested eyewitness.
oral agreement of Sharon and Bruce at the (D) related to preliminary matters such as the
time it was made. name or occupation of the witness.
(D) Sharon had a fiduciary obligation to her
nephew Stanley by reason of her being
named executrix of Bruce’s will. Questions 20–21 are based on the following fact
situation.
17. In an appropriate action to determine the
respective interests of Sharon and Stanley in Passer was driving his pickup truck along a lonely
Greenacre, if Sharon is adjudged to be the owner road on a very cold night. Passer saw Tom, who was
of all of Greenacre, the most likely reason for a stranger, lying in a field by the side of the road and
the judgment will be that apparently injured. Passer stopped his truck, alighted,
and, upon examining Tom, discovered that Tom was
(A) the Statute of Frauds prevents the proof intoxicated and in danger of suffering from exposure
of Sharon’s oral agreement. to the cold. However, Passer returned to his truck and
(B) Bruce could not unilaterally sever the joint drove away without making any effort to help Tom.
tenancy. Tom remained lying in the same place and was later
(C) Sharon’s nomination as executrix of injured when struck by a car driven by Traveler, who
Bruce’s estate does not prevent her from was drowsy and inattentive, had veered off the road
asserting her claim against Stanley. into the field and hit Tom. Traveler did not see Tom
(D) the record title of the joint tenancy in prior to hitting him.
Greenacre can be changed only by a duly
recorded instrument. 20. If Tom asserts a claim against Passer for
damages for his injuries, will Tom prevail?

(A) Yes, because by stopping and examining
18. Mary Webb, a physician called as a witness Tom, Passer assumed a duty to aid him.
by the defendant in the case of Parr v. Doan, (B) Yes, if a reasonably prudent person under
was asked to testify to statements made by the circumstances would have aided Tom.
Michael Zadok, her patient, for the purpose of (C) No, if Passer did not, in any way, make
obtaining treatment from Dr. Webb. Which of Tom’s situation worse.
the following is the best basis for excluding (D) No, because Tom himself created the risk
evidence of Zadok’s statements in a jurisdiction of harm by becoming intoxicated.
with a doctor-patient privilege?
21. If Tom asserts a claim against Traveler, will
(A) An objection by Dr. Webb asserting her Tom prevail?
privilege against disclosure of confidential
communications made by a patient. (A) Yes, because Traveler was negligent in
going off the road.

6
(B) Yes, because Tom was in a helpless immediate possession of Blackacre. The contract
condition. was silent as to the consequences of Homer’s
(C) No, because Traveler did not see Tom failure to give up possession within the 30-day
before Tom was struck. period, and the jurisdiction in which Blackacre is
(D) No, because Tom’s intoxication was the located has no statute dealing directly with this
cause in fact of his harm. situation, although the landlord-tenant law of the
jurisdiction requires a landlord to give a tenant
30 days notice before a tenant may be evicted.
Purcell did not give Homer any such 30-day
22. Roofer entered into a written contract with statutory notice. Purcell’s best legal argument
Orissa to repair the roof of Orissa’s home, the in support of his action to gain immediate
repairs to be done “in a workmanlike manner.” possession is that Homer is a
Roofer completed the repairs and took all of his
equipment away, with the exception of a 20-foot (A) trespasser ab initio.
extension ladder, which was left against the side (B) licensee.
of the house. He intended to come back and get (C) tenant at sufferance.
the ladder the next morning. At that time, Orissa (D) tenant from month to month.
and her family were away on a trip. During the
night, a thief, using the ladder to gain access
to an upstairs window, entered the house and Questions 24–25 are based on the following fact
stole some valuable jewels. Orissa has asserted a situation.
claim against Roofer for damages for the loss of
the jewels. Albert engaged Bertha, an inexperienced actress, to
do a small role in a new Broadway play for a period
In her claim against Roofer, Orissa will of six months at a salary of $200 a week. Bertha
turned down another role in order to accept this
(A) prevail, because by leaving the ladder, engagement. On the third day of the run, Bertha was
Roofer became a trespasser on Orissa’s hospitalized with influenza and Helen was hired to do
property. the part. A week later, Bertha recovered, but Albert
(B) prevail, because by leaving the ladder, refused to accept her services for the remainder of the
Roofer created the risk that a person might contract period. Bertha then brought an action against
unlawfully enter the house. Albert for breach of contract.
(C) not prevail, because the act of the thief was
a superseding cause. 24. Which of the following is Bertha’s best legal
(D) not prevail, because Orissa’s claim is theory?
limited to damages for breach of contract.
(A) Her acting contract with Albert was legally
23. Homer and Purcell entered into a valid, severable into weekly units.
enforceable written contract by which Homer (B) Her performance of the literal terms of the
agreed to sell and Purcell agreed to purchase contract was physically impossible.
Blackacre, which was Homer’s residence. One (C) Her reliance on the engagement with
of the contract provisions was that after closing, Albert by declining another acting role
Homer had the right to remain in residence at created an estoppel against Albert.
Blackacre for up to 30 days before delivering (D) Her failure to perform for one week was
possession to Purcell. The closing took place as not a material failure so as to discharge
scheduled. Title passed to Purcell and Homer Albert’s duty to perform.
remained in possession. Within a few days after
the closing, the new house next door which 25. Which of the following, if true, would adversely
was being constructed for Homer was burned affect Bertha’s rights in her action against
to the ground, and at the end of the 30-day Albert?
period Homer refused to move out of Blackacre;
instead, Homer tendered to Purcell a monthly (A) Albert could not find any substitute except
rental payment in excess of the fair rental value Helen, who demanded a contract for a
of Blackacre. Purcell rejected the proposal and minimum of six months if she was to
that day brought an appropriate action to gain perform at all.
(B) Helen, by general acclaim, was much
better in the role than Bertha had been.

7
(C) Albert had offered Bertha a position as (D) did not negligently contribute to the failure
Helen’s understudy at a salary of $100 a to have the complaint filed.
week, which Bertha declined.
(D) Albert had offered Bertha a secretarial
position at a salary of $300 a week, which
Bertha declined. 28. A state statute divides murder into degrees.
First-degree murder is defined as murder with
premeditation and deliberation or a homicide
in the commission of arson, rape, robbery,
Questions 26–27 are based on the following fact burglary, or kidnapping. Second-degree murder
situation. is all other murder at common law.

Lawyers Abel and Baker are the members of the law In which of the following situations is Defendant
partnership of Abel and Baker in a small town that most likely to be guilty of first-degree murder?
has only one other lawyer in it. Abel and Baker do a
substantial amount of personal injury work. Client was (A) Immediately after being insulted by
severely and permanently injured in an automobile Robert, Defendant takes a knife and stabs
collision when struck by an automobile driven by and kills Robert.
Motorist. Client employed the Abel and Baker firm to (B) Angered over having been struck by Sam,
represent her in obtaining damages for her injuries. At Defendant buys rat poison and puts it into
the time Client employed Abel and Baker, the statute Sam’s coffee. Sam drinks the coffee and
of limitations had six weeks to run on her claim. The dies as a result.
complaint was prepared but not filed. Abel and Baker (C) Intending to injure Fred, Defendant lies in
each thought that the other would file the complaint. wait and, as Fred comes by, strikes him
The statute of limitations ran on Client’s claim against with a broom handle. As a result of the
Motorist. blow, Fred dies.
(D) Defendant, highly intoxicated, discovers a
Client has filed suit against Abel and Baker for revolver on a table. He picks it up, points
negligence. That case is on trial with a jury in a court it at Alice, and pulls the trigger. The gun
of general jurisdiction. discharges, and Alice is killed.

26. In order to establish a breach of standard of care 29. On a camping trip in a state park, Rose
owed to her by Abel and Baker, Client discovered, near a rubbish heap, metal signs
reading, “Natural Wildlife Area—No Hunting.”
(A) must have a legal expert from the same She took two of the signs and used them to
locality testify that defendants’ conduct decorate her room at home. She is charged
was a breach. with violation of a state statute which provides,
(B) must have a legal expert from the same “Any person who appropriates to his own use
state testify that defendants’ conduct was a property owned by the state shall be guilty of
breach. a crime and shall be punished by a fine of not
(C) can rely on the application of the jurors’ more than $1,000, or by imprisonment for not
common knowledge as to whether there more than five years, or by both such fine and
was a breach. imprisonment.”
(D) can rely on the judge, as an expert in the
law, to advise the jury whether there was a At trial, Rose admits taking the signs but says
breach. she believed they had been thrown away. In fact,
the signs had not been abandoned.
27. In addition to proving that Abel and Baker were
negligent, Client must establish, as a minimum, Rose should be found
that she
(A) guilty, because this is a public welfare
(A) would have, but for her lawyers’ offense.
negligence, recovered from Motorist. (B) guilty, because she should have inquired
(B) had a good faith claim against Motorist whether the signs had been abandoned.
that was lost by her lawyers’ negligence. (C) not guilty if the jury finds that she honestly
(C) was severely and permanently injured believed the signs had been abandoned.
when struck by Motorist’s automobile. (D) not guilty unless the jury finds that the
state had taken adequate steps to inform
8
the public that the signs had not been 31. Assume for this question only that Pete has
abandoned. announced his intention of erecting a fast-food
restaurant on the 10-acre tract and that Sarah has
30. Ted frequently visited Janet, his next-door filed an action to enjoin Pete. If Sarah wins, it
neighbor. Janet was separated from her husband, will be because
Howard. Howard resided with his mother but
jointly owned the house in which Janet resided. (A) Sarah has an equitable servitude
Late one night, Ted and Janet were sitting on the concerning the use of the tract.
bed in Janet’s bedroom drinking when Howard (B) Sarah, as a taxpayer, has legal interest in
burst through the door and told Ted, “Get out.” the use of the tract.
When Ted refused, Howard challenged him to (C) Sarah is a creditor beneficiary of Oscar’s
go outside and “fight it out.” Ted again refused. promise with respect to the tract.
Howard then pulled a knife from his pocket (D) Pete is not a bona fide purchaser.
and lunged at Ted. Ted grabbed a lamp, struck
Howard on the head, and killed him. Ted is 32. Assume for this question only that Joe, who
charged with murder. On a charge of murder, purchased his lot from Max, has placed a mobile
Ted should be found home on it and that Sarah brings an action
against Joe to force him to remove it. The result
(A) not guilty, because Ted had as much right of this action will be in favor of
as Howard to be in the house.
(B) not guilty, because Howard attacked Ted (A) Sarah, because the restrictive covenant in
with a deadly weapon. her deed runs with the land.
(C) guilty, because Ted’s presence in Janet’s (B) Sarah, because the presence of the mobile
bedroom prompted Howard’s attack. home may adversely affect the market
(D) guilty, because Ted’s failure to obey value of her land.
Howard’s order to leave the house made (C) Joe, because his deed did not contain the
him a trespasser. restrictive covenant.
(D) Joe, because he is not a direct but a remote
grantee of Oscar.
Questions 31–33 are based on the following fact
situation. 33. Assume for this question only that in 1977 the
school board of the district in which Happy
In 1970, Oscar, owner of a 100-acre tract, prepared Acres is situated has voted to erect a new school
and duly recorded a subdivision plan called Happy on the 10-acre tract. In an appropriate action
Acres. The plan showed 90 one-acre lots and a ten- between the school board and Pete to determine
acre tract in the center that was designated “Future title, the result will be in favor of
Public School.” Oscar published and distributed a
brochure promoting Happy Acres which emphasized (A) Pete, because the school board has been
the proximity of the lots to the school property and guilty of laches.
indicated potential tax savings “because the school (B) Pete, because his deed did not refer to the
district will not have to expend tax money to acquire subdivision plan.
this property.” There is no specific statute concerning (C) the school board, because Pete had
the dedication of school sites. constructive notice of the proposed use of
the tract.
Oscar sold 50 of the lots to individual purchasers. (D) the school board, because there has been a
Each deed referred to the recorded plan and also dedication and acceptance of the tract.
contained the following clause: “No mobile home
shall be erected on any lot within Happy Acres.”
Sarah was one of the original purchasers from Oscar.
Questions 34–37 are based on the following fact
In 1976, Oscar sold the remaining 40 lots and the situation.
10-acre tract to Max by a deed which referred to the
plan and contained the restriction relating to mobile On March 1, Zeller orally agreed to sell his land,
homes. Max sold the 40 lots to individual purchasers Homestead, to Byer for $46,000 to be paid on March
and the 10-acre tract to Pete. None of the deeds from 31. Byer orally agreed to pay $25,000 of the purchase
Max referred to the plan or contained any reference to price to Quincy in satisfaction of a debt which Zeller
mobile homes. said he had promised to pay Quincy.

9
On March 10, Byer dictated the agreement to his (D) Before he instituted his action against
secretary but omitted all reference to the payment of Byer, Quincy had not notified either Byer
the $25,000 to Quincy. In typing the agreement, the or Zeller that he had accepted the Byer-
secretary mistakenly typed in $45,000 rather than Zeller arrangement for paying Quincy.
$46,000 as the purchase price. Neither Byer nor Zeller
carefully read the writing before signing it on March 37. If Byer refused to pay more than $45,000 for
15. Neither noticed the error in price and neither raised Homestead, in an action by Zeller against Byer
any question concerning omission of the payment to for an additional $1,000, it would be to Zeller’s
Quincy. advantage to try to prove that

34. In an action by Quincy against Byer for $25,000, (A) the writing was intended only as a sham.
which of the following is/are correct? (B) the writing was only a partial integration.
(C) there was a mistake in integration.
I. Byer could successfully raise the Statute (D) there was a misunderstanding between
of Frauds as a defense because the Byer- Zeller and Byer concerning the purchase
Zeller agreement was to answer for the price.
debt of another.
II. Byer could successfully raise the Statute
of Frauds as a defense because the Byer-
Zeller agreement was for the sale of an Questions 38–40 are based on the following fact
interest in land. situation.

(A) I only House owns his home in City. On the lawn in front of
(B) II only his home and within five feet of the public sidewalk
(C) Both I and II there was a large tree. The roots of the tree caused the
(D) Neither I nor II sidewalk to buckle severely and become dangerous.
An ordinance of City requires adjacent landowners
35. Which of the following would be most important to keep sidewalks in safe condition. House engaged
in deciding an action by Quincy against Byer for Contractor to repair the sidewalk, leaving it to
$25,000? Contractor to decide how the repair should be made.

(A) Whether the Byer-Zeller agreement was Contractor dug up the sidewalk, cut back the roots
completely integrated. of the tree, and laid a new sidewalk. Two days after
(B) Whether Byer was negligent in not having House had paid Contractor the agreed price of the
carefully read the written agreement. repair, the tree fell over onto the street and damaged a
(C) Whether Zeller was negligent in not having parked car belonging to Driver.
carefully read the written agreement.
(D) Whether Quincy was a party to the Driver has asserted claims against House and
contract. Contractor, and both defendants admit that cutting the
roots caused the tree to fall.
36. In an action by Quincy against Byer for $25,000,
which of the following, if proved, would best 38. The theory on which Driver is most likely to
serve Byer as a defense? prevail against House is that House is

(A) There was no consideration to support (A) strictly liable, because the tree was on his
Zeller’s antecedent promise to pay Quincy property.
the $25,000. (B) liable for Contractor’s negligence if,
(B) On March 5, before Quincy was aware to House’s knowledge, Contractor was
of the oral agreement between Zeller and engaged in hazardous activity.
Byer, Zeller agreed with Byer not to pay (C) liable, because he assumed responsibility
any part of the purchase price to Quincy. when he paid Contractor for the repair.
(C) Whatever action Quincy may have had (D) liable on the basis of respondeat superior.
against Byer was barred by the statute of
limitations prior to March 1. 39. In the claim of Driver against Contractor, the
best defense of Contractor is that

10
(A) the tree was on the property of House. (C) power to tax and spend for the general
(B) he repaired the sidewalk at the direction of welfare.
House. (D) power to enforce the privileges and
(C) he could not reasonably foresee that the immunities clause of the Fourteenth
tree would fall. Amendment.
(D) he was relieved of liability when House
paid for the repair.
Questions 43–44 are based on the following fact
40. If Driver recovers a judgment against House, situation.
does House have any recourse against
Contractor? In 1930, Owens, the owner in fee simple of
Barrenacres, a large, undeveloped tract of land,
(A) No, if payment by House was an granted an easement to the Water District “to install,
acceptance of the work. inspect, repair, maintain, and replace pipes” within a
(B) No, because House selected Contractor to properly delineated strip of land 20 feet wide across
do the work. Barrenacres. The easement permitted the Water
(C) Yes, if the judgment against House was District to enter Barrenacres for only the stated
based on vicarious liability. purposes. The Water District promptly and properly
(D) Yes, because House’s conduct was not a recorded the deed. In 1931, the Water District installed
factual cause of the harm. a water main which crossed Barrenacres within the
described strip; the Water District has not since
entered Barrenacres.

41. The State of Rio Grande entered into a contract In 1935, Owens sold Barrenacres to Peterson, but the
with Roads, Inc., for construction of a four- deed, which was promptly and properly recorded,
lane turnpike. Prior to commencement of failed to refer to the Water District easement. Peterson
construction, the legislature, in order to provide built his home on Barrenacres in 1935, and since that
funds for parks, repealed the statute authorizing time he has planted and maintained, at great expense
the turnpike and cancelled the agreement with in money, time, and effort, a formal garden area which
Roads, Inc. Roads, Inc., sued the state to enforce covers, among other areas, the surface of the 20-foot
its original agreement. In ruling on this case, a easement strip.
court should hold that the state statute cancelling
the agreement is In 1976, the Water District proposed to excavate the
entire length of its main in order to inspect, repair, and
(A) valid, because constitutionally the replace the main, to the extent necessary. At a public
sovereign is not liable except with its own meeting, at which Peterson was present, the Water
consent. District announced its plans and declared its intent
(B) valid, because the legislature is vested with to do as little damage as possible to any property
constitutional authority to repeal laws it involved. Peterson objected to the Water District
has enacted. plans.
(C) invalid, because a state is equitably
estopped to disclaim a valid bid once 43. Peterson asked his attorney to secure an
accepted by it. injunction against the Water District and
(D) invalid, because of the constitutional its proposed entry upon his property. The
prohibition against impairment of best advice that the attorney can give is that
contracts. Peterson’s attempt to secure injunctive relief
will be likely to
42. The strongest constitutional basis for the
enactment of a federal statute requiring colleges (A) succeed, because Peterson’s deed from
and universities receiving federal funds to offer Owens did not mention the easement.
student aid solely on the basis of need is the (B) succeed, because more than 40 years have
passed since the Water District last entered
(A) police power. Barrenacres.
(B) war and defense power. (C) fail, because the Water District’s plan is
within its rights.

11
(D) fail, because the Water District’s plan is (B) Church and Darin should be enjoined,
fair and equitable. and damages should be recovered but
impounded for future distribution.
44. Assume that Peterson reserved his rights and (C) The injunction should be granted, but
after the Water District completed its work sued damages should be denied, because Omar
for the $5,000 in damages he suffered by reason and Carl are not parties to the action.
of the Water District entry. Peterson’s attempt to (D) Damages should be awarded, but the
secure damages probably will injunction should be denied.

(A) succeed, because his deed from Owens did 46. In which of the following cases is a conviction
not mention the easement. of the named defendant for robbery LEAST
(B) succeed, because of an implied obligation likely to be upheld?
imposed on the Water District to restore
the surface to its condition prior to entry. (A) Johnson forced his way into a woman’s
(C) fail, because of the public interest in home, bound her, and compelled her to tell
maintaining a continuous water supply. him that her jewelry was in an adjoining
(D) fail, because the Water District acted room. Johnson went to the room, took the
within its rights. jewelry, and fled.
(B) A confederate of Brown pushed a man in
order to cause him to lose his balance and
drop his briefcase. Brown picked up the
45. In 1960, Omar, the owner in fee simple absolute, briefcase and ran off with it.
conveyed Stoneacre, a five-acre tract of land. (C) Having induced a woman to enter his
The relevant, operative words of the deed hotel room, Ritter forced her to telephone
conveyed to “Church [a duly organized religious her maid to tell the maid to bring certain
body having power to hold property] for the life jewelry to the hotel. Ritter locked the
of my son, Carl, and from and after the death of woman in the bathroom while he accepted
my said son, Carl, to all of my grandchildren and the jewelry from the maid when she
their heirs and assigns in equal shares; provided arrived.
that Church shall use the premises for church (D) Hayes unbuttoned the vest of a man too
purposes only.” drunk to notice and removed his wallet. A
minute later, the victim missed his wallet
In an existing building on Stoneacre, Church and accused Hayes of taking it. Hayes
immediately began to conduct religious services pretended to be insulted, slapped the
and other activities normally associated with a victim, and went off with the wallet.
church.

In 1975, Church granted to Darin a right to Questions 47–48 are based on the following fact
remove sand and gravel from a one-half-acre situation.
portion of Stoneacre upon the payment of a
royalty. Darin has regularly removed sand Green is cited for contempt of the House of
and gravel since 1975 and paid the royalty Representatives after she refuses to answer certain
to Church. Church has continued to conduct questions posed by a House Committee concerning
religious services and other church activities on her acts while serving as a United States Ambassador.
Stoneacre. A federal statute authorizes the Attorney General to
prosecute contempts of Congress. Pursuant to this
All four of the living grandchildren of Omar, law, the House directs the Attorney General to begin
joined by a guardian ad litem to represent unborn criminal proceedings against Green. A federal grand
grandchildren, instituted suit against Church and jury indicts Green, but the Attorney General refuses to
Darin seeking damages for the removal of sand sign the indictment.
and gravel and an injunction preventing further
acts of removal. There is no applicable statute. 47. Which of the following best describes the
Which of the following best describes the likely constitutionality of the Attorney General’s
disposition of this lawsuit? action?

(A) The plaintiffs should succeed, because the (A) Illegal, because the Attorney General must
interest of Church terminated with the first prosecute if the House of Representatives
removal of sand and gravel. directs.

12
(B) Illegal, because the Attorney General must (B) he consented to the entry.
prosecute those who violate federal law. (C) no overt act was committed by him.
(C) Legal, because ambassadors are immune (D) there was no intent to commit a felony.
from prosecution for acts committed in the
course of their duties. 50. Bill’s best argument for acquittal of burglary is
(D) Legal, because the decision to prosecute is that he
an exclusively executive act.
(A) acted under a mistake of law.
48. If the Attorney General signs the indictment, (B) had the consent of the owner.
the strongest argument Green could urge as a (C) reasonably thought he was in Harry’s
defense is that house.
(D) found the window unlocked.
(A) Green may refuse to answer the questions
if she can demonstrate that they are 51. If Harry and Bill are charged with a conspiracy
unrelated to matters upon which Congress to commit burglary, their best argument for
may legislate. acquittal is that
(B) the House may question Green on matters
pertaining to the expenditures of funds (A) Bill was the alter ego of Harry.
appropriated by Congress. (B) they did not intend to commit burglary.
(C) only the Senate may question Green on (C) there was no overt act.
matters that relate to the performance of (D) there was no agreement.
her duties.
(D) Congress may not ask questions relating to
the performance of duties executed by an
officer of the executive branch. Questions 52–55 are based on the following fact
situation.

Penn sued Duke’s Bar for injuries suffered in an
Questions 49–51 are based on the following fact automobile accident caused by Chase, who had been
situation. a patron of Duke’s Bar. Penn claimed that Chase was
permitted to drink too much liquor at Duke’s Bar
Harry met Bill, who was known to him to be a burglar, before the accident.
in a bar. Harry told Bill that he needed money. He
promised to pay Bill $500 if Bill would go to Harry’s 52. Wood, a patron of Duke’s Bar, testified that on
house the following night and take some silverware. the night of the accident Chase was drunk. Wood
Harry explained to Bill that, although the silverware then proposed to testify that he remarked to his
was legally his, his wife would object to his selling it. companion, “Chase is so drunk he can’t even
stand up.” Wood’s remark to his companion is
Harry pointed out his home, one of a group of similar
tract houses. He drew a floor plan of the house that (A) admissible as an excited utterance.
showed the location of the silverware. Harry said that (B) admissible as a prior consistent statement.
his wife usually took several sleeping pills before (C) admissible as a statement by Wood
retiring, and that he would make sure that she took regarding a condition he observed, made
them the next night. He promised to leave a window while he was observing it.
unlocked. (D) inadmissible if there was no evidence
that Wood had expertise in determining
Everything went according to the plan except that Bill, drunkenness.
deceived by the similarity of the tract houses, went
to the wrong house. He found a window unlocked, 53. Duke’s Bar called Chase to testify and expected
climbed in, and found silver where Harry had him to say that he was sober when he left
indicated. He took the silver to the cocktail lounge Duke’s Bar; however, on direct examination
where the payoff was to take place. At that point Chase testified that he may have had a little
police arrested the two men. too much to drink at Duke’s Bar. Duke’s Bar
now seeks to confront Chase with his statement
49. If Harry were charged with burglary, his best made on deposition that he was sober when he
argument for acquittal would be that left Duke’s Bar. Which of the following is true
concerning this statement?
(A) there was no breaking.

13
(A) It may be used only to refresh Chase’s (A) She had heard the same voice on a similar
recollection. tape recording identified to her by Daly’s
(B) It is admissible for impeachment and as brother.
substantive evidence that Chase was sober. (B) She had heard Daly speak many times, but
(C) It is inadmissible, because Duke’s Bar never over the telephone.
cannot impeach its own witness. (C) She had, specifically for the purpose of
(D) It is inadmissible, because it is hearsay, not preparing to testify, talked with Daly over
within any exception. the telephone at a time after the recording
was made.
54. Penn offered evidence that, after the accident, (D) She had been present with Daly when he
the manager of Duke’s Bar established house engaged in the conversation in question
rules limiting all customers to two drinks per but had heard only Daly’s side of the
hour, with a maximum limit of four drinks per conversation.
night. This evidence is

(A) admissible to show that the prior conduct Questions 57–58 are based on the following fact
of Duke’s Bar was negligent. situation.
(B) admissible to show that Duke’s Bar was
aware of the need for taking precautionary Devlin was the owner of a large subdivision. Parnell
measures. became interested in purchasing a lot but could not
(C) inadmissible, because subsequent measures decide between Lot 40 and Lot 41. The price and fair
by an employee are not binding on Duke’s market value of each of those two lots was $5,000.
Bar. Parnell paid Devlin $5,000, which Devlin accepted,
(D) inadmissible, because its admission would and Devlin delivered to Parnell a deed which was
discourage the taking of such remedial properly executed, complete, and ready for recording
measures. in every detail except that the space in the deed for
the lot number was left blank. Devlin told Parnell
55. Penn offered evidence that, after the accident, to fill in either Lot 40 or Lot 41 according to his
the owner of Duke’s Bar visited him at the decision and then to record the deed. Parnell visited
hospital and, offering to pay all of Penn’s the development the next day and completely changed
medical expenses, said, “That’s the least I can his mind, selecting Lot 25. He filled in Lot 25 and
do after letting Chase leave the bar so drunk duly recorded the deed. The price of Lot 25 and its fair
last night.” The statement that Chase was drunk market value was $7,500.
when he left the bar on the night of the accident
is 57. Immediately upon learning what Parnell had
done, Devlin brought an appropriate action
(A) admissible as an admission by the owner against Parnell to rescind the transaction. If
of Duke’s Bar that Chase was drunk when Devlin loses, the most likely basis for the
he left the bar. judgment is that
(B) admissible as a factual admission made in
connection with an offer of compromise. (A) Devlin’s casual business practices created
(C) inadmissible as hearsay, not within any his loss.
exception. (B) the need for certainty in land title records
(D) inadmissible as a statement made in controls.
connection with an offer to pay medical (C) the agency implied to complete the
expenses. deed cannot be restricted by the oral
understanding.
(D) the recording of the deed precludes any
questioning of its provisions in its recorded
56. In a narcotics conspiracy prosecution against form.
Daly, the prosecutor offers in evidence a tape
recording of a telephone call allegedly made by 58. Assume the following facts for this question
Daly. A lay witness is called to testify that the only. Before Devlin had time to learn of
voice on the recording is Daly’s. Her testimony Parnell’s actions, Parnell sold Lot 25 to Caruso
to which of the following would be the LEAST for $6,000 by a duly and properly executed,
sufficient basis for admitting the recording? delivered, and recorded warranty deed. Caruso
knew that Devlin had put a price of $7,500 on

14
Lot 25, but he knew no other facts regarding the Questions 60–64 are based on the following fact
Devlin-Parnell transaction. Caruso’s attorney situation.
accurately reported Parnell’s record title to be
good, marketable, and free of encumbrances. While negligently driving his father’s uninsured
Neither Caruso nor his attorney made any further automobile, 25-year-old Arthur crashed into an
investigation outside the record. Devlin brought automobile driven by Betty. Both Arthur and Betty
an appropriate action against Caruso to recover were injured. Charles, Arthur’s father, erroneously
title to Lot 25. If Devlin loses, the most likely believing that he was liable because he owned the
basis for the judgment is that automobile, said to Betty, “I will see to it that you are
reimbursed for any losses you incur as a result of the
(A) the Statute of Frauds prevents the accident.” Charles also called Physician and told him
introduction of any evidence of Devlin’s to take care of Betty, and that he, Charles, would pay
and Parnell’s agreement. the bill.
(B) recording of the deed from Devlin to
Parnell precludes any question of its Arthur, having no assets, died as a result of his
genuineness. injuries. Dodge, one of Arthur’s creditors, wrote to
(C) as between Devlin and a bona fide Charles stating that Arthur owed him a clothing bill
purchaser, Devlin is estopped. of $200 and that he was going to file a claim against
(D) the clean hands doctrine bars Devlin from Arthur’s estate. Charles replied, “If you don’t file a
relief. claim against Arthur’s estate, I will pay what he owed
you.”

60. In an action by Betty against Charles for wages
59. Ohner owns the Acme Hotel. When the lost while she was incapacitated as a result of
International Order of Badgers came to town for the accident, which of the following would be
its convention, its members rented 400 of the Charles’s best defense?
500 rooms, and the hotel opened its convention
facilities to them. Badgers are a rowdy group, (A) Lack of consideration
and during their convention they littered both the (B) Mistake of fact as to basic assumption
inside and the outside of the hotel with debris (C) Statute of Frauds
and bottles. The hotel manager knew that objects (D) Indefiniteness of Charles’s promise
were being thrown out of the hotel windows.
At his direction, hotel employees patrolled the 61. Which of the following, if true, would be
hallways telling the guests to refrain from such significant in determining whether or not there
conduct. Ohner was out of town and was not was bargained-for consideration to support
aware of the problems which were occurring. Charles’s promise to Physician?
During the convention, as Smith walked past
the Acme Hotel on the sidewalk, he was hit and I. Physician had not begun treating Betty
injured by an ashtray thrown out of a window in before Charles called him.
the hotel. Smith sued Ohner for damages for his II. Charles had a contract with Betty.
injuries.
(A) I only
Will Smith prevail in his claim against Ohner? (B) II only
(C) Both I and II
(A) Yes, because a property owner is strictly (D) Neither I nor II
liable for acts on his premises if such acts
cause harm to persons using the adjacent 62. If Physician discontinued treating Betty before
public sidewalks. she had fully recovered and Betty brought an
(B) Yes, if the person who threw the ashtray action against Physician for breach of contract,
cannot be identified. which of the following arguments, if any,
(C) No, because Ohner had no personal by Physician would probably be effective in
knowledge of the conduct of the hotel defense?
guests.
(D) No, if the trier of fact determines that the I. Betty furnished no consideration, either
hotel employees had taken reasonable express or implied.
precautions to prevent such an injury. II. Physician’s contract was with Charles and
not with Betty.

15
III. Whatever contract Physician may have had denies admission to all non-Caucasians. Stone School
with Betty was discharged by novation on is a private school that offers religious instruction.
account of the agreement with Charles.
65. Which of the following is the strongest argument
(A) I only against the constitutionality of free distribution
(B) I and II only of textbooks to the students at the Little White
(C) II and III only School?
(D) Neither I nor II nor III
(A) No legitimate educational function is
63. If Dodge did not file action against Arthur’s served by the free distribution of textbooks.
estate, would Dodge succeed in an action against (B) The state may not in any way aid private
Charles for $200? schools.
(C) The Constitution forbids private bias of
(A) Yes, because Dodge had detrimentally any kind.
relied on Charles’s promise. (D) Segregation is furthered by the distribution
(B) Yes, because Charles’s promise was of textbooks to these students.
supported by a bargained-for exchange.
(C) No, because Dodge’s claim against 66. Which of the following is the strongest
Arthur’s estate was worthless. argument in favor of the constitutionality of free
(D) No, because Charles at most had only a distribution of textbooks to the students at Stone
moral obligation to pay Arthur’s debts. School?

64. Assume that Charles, honestly believing that he (A) Private religious schools, like public
owed Dodge nothing, refused to pay anything to nonsectarian schools, fulfill an important
Dodge, who honestly believed that Charles owed educational function.
him $200. If Dodge then accepts $150 from (B) Religious instruction in private schools is
Charles in settlement of the claim, will Dodge not constitutionally objectionable.
succeed in an action against Charles for the (C) The purpose and effect of the free
remaining $50? distribution of these textbooks is secular
and does not entangle church and state.
(A) Yes, because Arthur’s debt of $200 was (D) The free exercise clause requires identical
liquidated and undisputed. treatment by the state of students in public
(B) Yes, because Dodge honestly believed that and private schools.
he had a legal right against Charles for the
full $200.
(C) No, because Charles honestly believed that
Dodge did not have a legal right against 67. Defendant is charged with assault and battery.
him for the $200. The state’s evidence shows that Victim was
(D) No, because Charles was not contractually struck in the face by Defendant’s fist. In which
obligated to pay Dodge $200 in the first of the following situations is Defendant most
place. likely to be not guilty of assault and battery?

(A) Defendant had been hypnotized at a party


and ordered by the hypnotist to strike the
Questions 65–66 are based on the following fact person he disliked the most.
situation. (B) Defendant was suffering from an epileptic
seizure and had no control over his
Until 1954, the state of New Atlantic required motions.
segregation in all public and private schools, but all (C) Defendant was heavily intoxicated and
public schools are now desegregated. Other state laws, was shadowboxing without realizing that
enacted before 1954 and continuing to the present, Victim was near him.
provide for free distribution of the same textbooks on (D) Defendant, who had just awakened from
secular subjects to students in all public and private a deep sleep, was not fully aware of what
schools. In addition, the state accredits schools and was happening and mistakenly thought
certifies teachers. Victim was attacking him.

Little White School, a private school that offers 68. Leader is a labor leader in Metropolis. Ten years
elementary and secondary education in the state, ago he was divorced. Both he and his first wife
16
have since married other persons. Recently, than half the average size of this crop over the five
News, a newspaper in another city, ran a feature years immediately preceding the construction of the
article on improper influences it asserted had plant. It can be established that the fumes caused the
been used by labor officials to secure favorable crop reduction.
rulings from government officials. The story said
that in 1960 Leader’s first wife, with Leader’s Farmer’s hay fever, from which he had long suffered,
knowledge and concurrence, gave sexual favors became worse in 1976. Physicians advised him that
to the mayor of Metropolis and then persuaded the lignite fumes were affecting it and that serious
him to grant concessions to Leader’s union, with lung disease would soon result unless he moved
which Metropolis was then negotiating a labor away from the plant. He did so, selling his farm at its
contract. The story named Leader and identified reasonable market value, which was then $10,000 less
his first wife by her former and current surnames. than before the construction of the plant.
The reporter for News believed the story to be
true, since it had been related to him by two very 69. If Farmer asserts a claim based on nuisance
reliable sources. against Utility for damages for personal injuries,
will Farmer prevail?
Leader’s first wife suffered emotional distress
and became very depressed. If she asserts a (A) No, because there is no practicable way for
claim based on defamation against News, Utility to reduce the fumes.
she will (B) No, because Utility’s acts constituted a
public nuisance.
(A) prevail, because the story concerned her (C) Yes, because Farmer’s personal injuries
personal, private life. were within the scope of the liability
(B) prevail if the story was false. imposed on Utility.
(C) not prevail, because News did not print the (D) Yes, because the generation of electricity
story with knowledge of its falsity or with is an ultrahazardous activity.
reckless disregard for its truth or falsity.
(D) not prevail if News exercised ordinary 70. If Farmer asserts a claim based on negligence
care in determining if the story was true or against Utility for crop damages, will Farmer
false. prevail?

(A) No, because Utility was not negligent.


Questions 69–70 are based on the following fact (B) No as to 1976 crop damage, because
situation. Farmer did not mitigate damages by selling
his farm in 1975.
In 1976, Utility constructed a new plant for the (C) Yes as to 20 percent of his crop damage,
generation of electricity. The plant burns lignite, a because use of available equipment would
low-grade fuel which is available in large quantities. have reduced the fumes by 20 percent.
(D) Yes, because operation of the plant
Although the plant was constructed in accordance constitutes a nuisance.
with the best practicable technology, the plant emits a
substantial quantity of invisible fumes. The only way
Utility can reduce the fumes is by the use of scrubbing
equipment that would cost $50,000,000 to install and Questions 71–76 are based on the following fact
would increase the retail price of generated electricity situation.
by 50 percent while reducing the volume of fumes
by only 20 percent. Because of the expense of such On March 1, Computer Programs, Inc. (CP) orally
equipment and its relative ineffectiveness, no other agreed with Holiday Department Store (HDS) to
generating plants burning lignite use such equipment. write a set of programs for HDS’s computer and to
coordinate the programs with HDS’s billing methods.
The plant is located in a sparsely settled rural area, A subsequent memo, signed by both parties, provided
remote from the large city served by Utility. in its entirety:

Farmer owned a farm adjacent to the plant. He had HDS will pay CP $20,000 in two equal
farmed the land for 40 years and had lived on the installments within one month of completion if
premises. The prevailing winds carry fumes from the CP is successful in shortening by one-half the
new plant over Farmer’s land. His 1976 crop was less

17
processing time for the financial transactions (D) No, because “within one month of
now handled on HDS’s Zenon 747 computer; completion” would, in these circumstances,
CP to complete by July 1. This agreement may be interpreted to mean “within one month
be amended only by a signed writing. after completion.”

On June 6, CP demanded $10,000, saying the job 73. Was the escrow agreement a valid modification?
was one-half done. After HDS denied liability, the
parties orally agreed that HDS should deposit $20,000 (A) Yes, because it was the compromise of an
in escrow, pending completion to the satisfaction honest dispute.
of HDS’s computer systems manager. The escrow (B) Yes, because the Statute of Frauds does
deposit was thereupon made. On July 5, CP completed not apply to subsequent oral modifications.
the programs, having used an amount of time in which (C) No, because it was oral.
it could have earned $18,000 had it devoted that (D) No, because it was not supported by
time to other jobs. Tests by CP and HDS’s computer consideration.
systems manager then showed that the computer
programs, not being perfectly coordinated with HDS’s 74. Assume for this question only that the programs
billing methods, cut processing time by only 47 completed on July 5 had cut processing time by
percent. They would, however, save HDS $12,000 a one-half for all of HDS’s financial transactions.
year. Further, if HDS would spend $5,000 to change Is HDS entitled to renounce the contract because
its invoice preparation methods, as recommended by of CP’s delay in completion?
CP, the programs would cut processing time by a total
of 58 percent, saving HDS another $8,000 a year. (A) Yes, because “CP to complete by July 1”
HDS’s computer systems manager refused in good is an express condition.
faith to certify satisfactory completion. HDS asked the (B) Yes, because the doctrine of substantial
escrow agent to return the $20,000 and asserted that performance does not apply to commercial
nothing was owed to CP even though HDS continued contracts.
to use the programs. (C) No, because both parties manifested an
understanding that time was not of the
71. If HDS denies liability on the ground that CP essence.
had orally agreed to coordinate with HDS’s (D) No, because the contract did not contain
methods of accounting, and CP seeks in a liquidated damages clause dealing with
litigation to bar introduction of that agreement delay in completion.
because of the parol evidence rule, HDS’s most
effective argument is that 75. Assume for this question only that CP’s delay
in completion did not give HDS the right to
(A) the parol evidence rule does not bar the renounce the contract and that the parties’
introduction of evidence for the purpose of escrow agreement was enforceable. Is CP
interpreting a written agreement. entitled to recover damages for breach of the
(B) the memorandum was not a completely contract?
integrated agreement.
(C) HDS detrimentally relied on the oral (A) Yes, because CP had substantially
promise of coordination in signing the performed.
memorandum. (B) Yes, because the program would save
(D) the memorandum was not a partially HDS $12,000 a year.
integrated agreement. (C) No, because shortening the processing
time by one-half was an express condition
72. If CP in fact had half-completed the job on June 6, subsequent.
would it then have been entitled to $10,000? (D) No, because HDS’s computer systems
manager did not certify satisfactory
(A) Yes, because June 6 was within one month completion of the programs.
of completion.
(B) Yes, because CP had done one-half the 76. Assume for this question only that CP was in
job. breach of contract because of its four-day delay
(C) No, because of a constructive condition in completion and that an express condition
precedent requiring at least substantial precedent to HDS’s duty to pay the contract
completion of the work before HDS
would have a duty to pay.

18
price has failed. Can CP nevertheless recover the At trial, Donna’s motion to prevent the
reasonable value of its service? introduction of the statement she made while
being fingerprinted will most probably be
(A) Yes, because continued use of the
programs by HDS would save at least (A) granted, because Donna’s request to
$12,000 a year. contact her attorney by telephone was
(B) Yes, because HDS was continuing to use reasonable and should have been granted
programs created by CP for which, as HDS immediately.
knew, CP expected to be paid. (B) granted, because of the “fruit of the
(C) No, because failure of an express condition poisonous tree” doctrine.
precedent excused HDS from any duty to (C) denied, because the statements were
compensate CP. volunteered and not the result of
(D) No, because such a recovery by CP would interrogation.
be inconsistent with a claim by HDS (D) denied, because fingerprinting is not a
against CP for breach of contract. critical stage of the proceeding requiring
the assistance of counsel.

77. Police Officer stopped Dexter for speeding late Questions 79–80 are based on the following fact
one night. Noting that Dexter was nervous, situation.
Police Officer ordered him from the car and
placed him under arrest for speeding. By state Barnes was hired as an assistant professor of
law, Police Officer was empowered to arrest mathematics at Reardon State College and is now
Dexter and take him to the nearest police station in his third consecutive one-year contract. Under
for booking. Police Officer searched Dexter’s state law, he cannot acquire tenure until after five
person and discovered a package of heroin in consecutive annual contracts. In his third year, Barnes
his jacket pocket. was notified that he was not being rehired for the
following year. Applicable state law and college rules
Dexter is charged with possession of heroin. At did not require either a statement of reasons or a
trial, Dexter’s motion to prevent introduction of hearing, and in fact neither was offered to Barnes.
the heroin into evidence, on the ground that the
search violated his federal constitutional rights, 79. Which of the following, if established, sets forth
will most probably be the strongest constitutional argument Barnes
could make to compel the college to furnish him
(A) denied, because the search was incident to a statement of reasons for the failure to rehire
a valid custodial arrest. him and an opportunity for a hearing?
(B) denied, because Police Officer acted under
a reasonable suspicion and legitimate (A) There is no evidence that tenured teachers
concern for his own personal safety. are any more qualified than he is.
(C) granted, because there was no reasonable (B) He leased a home in reliance on an oral
or proper basis upon which to justify promise of reemployment by the college
conducting the search. president.
(D) granted if Police Officer was not in fear (C) He was the only teacher at the college
and had no suspicion that Dexter was whose contract was not renewed that year.
transporting narcotics. (D) In the expectation of remaining at the
college, he had just moved his elderly
78. Donna was arrested and taken to police parents to the town in which the college is
headquarters, where she was given her Miranda located.
warnings. Donna indicated that she wished to
telephone her lawyer and was told that she could 80. Which of the following, if established, most
do so after her fingerprints had been taken. strongly supports the college in refusing to give
While being fingerprinted, however, Donna Barnes a statement of reasons or an opportunity
blurted out, “Paying a lawyer is a waste of for a hearing?
money because I know you have me.”
(A) Barnes’ academic performance had been
substandard.

19
(B) A speech he made that was critical of residuary clause. Assuming that the action was
administration policies violated a college properly brought with all necessary parties and
regulation concerning teacher behavior. with a guardian ad litem appointed to represent
(C) Barnes worked at the college for less than the interests of unborn and infant grandchildren,
five years. the decision should be that
(D) Barnes could be replaced with a more
competent teacher. (A) the attempted gifts to grandchildren are
void under the Rule Against Perpetuities.
(B) the attempted gifts to grandchildren are
void as unlawful restraints on alienation.
81. National regulation of predatory wild animals on (C) the provisions concerning grandchildren
federal lands is most likely are valid and will be upheld according to
their terms.
(A) constitutional, because the protection of (D) even if the provisions against sale or
wild animals is important to the general mortgage by the grandchildren are void,
welfare. the remainders to grandchildren are
(B) constitutional, because Congress has otherwise valid and will be given effect.
authority to make regulations respecting
federal property. 83. Seth was an elderly widower who lived alone
(C) unconstitutional, because wild animals on a small farm which he owned. Except for the
as defined by state common law are not farm, including the house and its furnishings,
federal property. and the usual items of personal clothing and
(D) unconstitutional, because regulation and similar things, Seth owned substantially no
control of wild animals is retained by the property. Under proper management, the farm
states under the Tenth Amendment. was capable of producing an adequate family
income. Because of the usual deterioration
82. By her validly executed will, Sallie devised accompanying old age, Seth was unable to
a certain tract of land to her son, Ben, for his do farm work or even to provide for his own
life with remainder to such of Ben’s children personal needs. Seth entered into an oral contract
as should be living at his death, “Provided, with his nephew, Jim, by which Seth agreed to
however, that no such child of Ben shall convey the farm to Jim and Jim agreed to move
mortgage or sell, or attempt to mortgage or into the house with Seth, operate the farm, and
sell, his or her interest in the property prior to take care of Seth for the rest of his life. The
attaining 25 years of age; and, if any such child oral contract was silent as to when the land was
of Ben shall violate this provision, then upon to be conveyed. Jim, who lived about 50 miles
such violation his or her interest shall pass away where he was operating a small business
to and become the property of the remaining of his own, terminated his business and moved
children of Ben then living, share and share in with Seth. With the assistance of his wife, Jim
alike.” gave Seth excellent care until Seth died intestate
about five years after the date of the contract. In
Sallie’s will included an identical provision for his final years Seth was confined to his bed and
each of her four other children concerning four required much personal service of an intimate
other tracts of land. The residuary clause of the and arduous sort. Seth was survived by his only
will gave the residuary estate to Sallie’s five son, Sol, who was also Seth’s sole heir and next
children equally. Sallie died and was survived of kin. Sol resided in a distant city and gave his
by the five children named in her will and by 11 father no attention in his father’s final years. Sol
grandchildren. Several additional grandchildren showed up for Seth’s funeral and demanded that
have since been born. Jim vacate the farm immediately. Upon Jim’s
refusal to do so, Sol brought an appropriate
In an action for a declaration of rights, it was action for possession. Jim answered by way
claimed that the attempted gifts to Sallie’s of a counterclaim to establish Jim’s right to
grandchildren were entirely void and that the possession and title to the farm.
interests following the life estates to Sallie’s
children passed to the children absolutely by the

20
If the court’s decision is in favor of Jim, it will (B) denied, because Wade has absolute title to
be because the property.
(C) granted, because the tenancy by the
(A) the land is located in a state where the entirety that was created by the deed was
Statute of Frauds will not be applied if severed when Wade abandoned Mary.
there has been such part performance as (D) granted, because the estate created by the
will result in an irreparable hardship if the deed was not a tenancy by the entirety.
contract is not performed.
(B) the land is located in a state where the
Statute of Frauds will not be applied if Questions 85–87 are based on the following fact
there has been part performance that is by situation.
its very nature unequivocally referable to
the contract. Sand Company operated an installation for distributing
(C) Sol is precluded by the “clean hands” sand and gravel. The installation was adjacent to a
doctrine from enforcing his claim against residential area. On Sand’s grounds there was a chute
Jim. with polished metal sides for loading sand and gravel
(D) the blood relationship of uncle-nephew is into trucks. The trucks being loaded stopped on the
sufficient to remove the necessity for any public street below the chute.
writing to satisfy the Statute of Frauds.
After closing hours, a plywood screen was placed
84. The following events took place in a state that in the chute and the ladder used for inspection was
does not recognize common law marriage. The removed to another section of the installation. For
state does recognize the common law estate of several months, however, a number of children,
tenancy by the entirety and has no statute on the eight to ten years of age, had been playing on Sand’s
subject. property and the adjoining street after closing hours.
The children found the ladder and also discovered that
Wade Sloan and Mary Isaacs, who were never they could remove the plywood screen from the chute
formally married, lived together over a seven- and slide down to the street below. Sand knew of this
year period. During this time Mary identified activity.
herself as “Mrs. Sloan” with the knowledge and
consent of Wade. Wade and Mary maintained One evening, the children were using the chute as a
several charge accounts at retail stores under play device. As an automobile driven by Commuter
the names “Mr. and Mrs. Wade Sloan,” and approached the chute, Ladd, an eight-year-old boy,
they filed joint income tax returns as Mr. and slid down just in front of the automobile. Commuter
Mrs. Sloan. Within this period Wade decided applied her brakes, but they suddenly failed, and she
to buy a home. The deed was in proper form hit and injured Ladd. Commuter saw the child in time
and identified the grantees as “Wade Sloan and to have avoided hitting him if her brakes had worked
Mary Sloan his wife, and their heirs and assigns properly. Two days previously, Commuter had taken
forever as tenants by the entirety.” Wade made her car to Garage to have her brakes inspected. Garage
a down payment of $10,000 and gave a note and inspected the brakes and told her that the brakes
mortgage for the unpaid balance. Both Wade were in perfect working order. Claims were asserted
and Mary signed the note and mortgage for the on behalf of Ladd by his proper legal representative
unpaid balance. Both Wade and Mary signed the against Sand, Commuter, and Garage.
note and mortgage as husband and wife. Wade
made the monthly payments as they became due 85. On Ladd’s claim against Sand, will Ladd prevail?
until he and Mary had a disagreement and he
abandoned her and the house. Mary then made (A) Yes, if Sand could have effectively secured
the payments for three months. She then brought the chute at moderate cost.
an action against Wade for partition of the land (B) Yes, because Sand is strictly liable for
in question. The prayer for partition should be harm resulting from an artificial condition
on its property.
(A) denied, because a tenant by the entirety has (C) No, if Commuter had the last clear chance
no right to partition. to avoid the injury.
(D) No, because Ladd was a trespasser.

21
86. On Ladd’s claim against Commuter, (A) not prevail, because Bill was not an
Commuter’s best defense is that aggressor.
(B) not prevail unless Bill intended his gesture
(A) her conduct was not the cause in fact of the as a threat.
harm. (C) prevail if David honestly believed that Bill
(B) she used reasonable care in the would attack him.
maintenance of her brakes. (D) prevail only if a reasonable person under
(C) she could not reasonably foresee Ladd’s the circumstances would have believed that
presence in the street. Bill would attack him.
(D) she did not act willfully and wantonly.

87. On Ladd’s claim against Garage, will Ladd Questions 90–91 are based on the following fact
prevail? situation.

(A) Yes, because Garage is strictly liable in Section 1 of the Vehicle Code of State makes it illegal
tort. to cross a street in a central business district other
(B) Yes, if Garage was negligent in inspecting than at a designated crosswalk. Section 2 of the Code
Commuter’s brakes. prohibits parking any motor vehicle so that it blocks
(C) No, if Ladd was in the legal category of a any part of a designated crosswalk. Ped wanted to
bystander. cross Main Street in the central business district of
(D) No, because Sand’s conduct was an City, located in State, but a truck parked by Trucker
independent and superseding cause. was blocking the designated crosswalk. Ped stepped
out into Main Street and carefully walked around the
back of the truck. Ped was struck by a motor vehicle
negligently operated by Driver.
88. Driving down a dark road, Defendant accidentally
ran over a man. Defendant stopped and found 90. If Ped asserts a claim against Driver, Ped’s
that the victim was dead. Defendant, fearing that failure to be in the crosswalk will have which of
he might be held responsible, took the victim’s the following effects?
wallet, which contained a substantial amount of
money. He removed the identification papers and (A) It is not relevant in determining the right of
put the wallet and money back into the victim’s Ped.
pocket. Defendant is not guilty of (B) It may be considered by the trier of fact
on the issue of Driver’s liability.
(A) larceny, because he took the papers only to (C) It will bar Ped’s recovery unless Driver
prevent identification and not for his own saw Ped in time to avoid the impact.
use. (D) It will bar Ped’s recovery as a matter of
(B) larceny, because he did not take anything law.
from a living victim.
(C) robbery, because he did not take the papers 91. If Ped asserts a claim against Trucker, the most
by means of force or putting in fear. likely result is that Ped will
(D) robbery, because he did not take anything
of monetary value. (A) prevail, because Trucker’s violation of a
state statute makes him strictly liable for
89. Al and Bill are identical twins. Al, angry at all injuries caused thereby.
David, said, “You’d better stay out of my (B) prevail, because the probable purpose of
way. The next time I find you around here, Section 2 of the Vehicle Code of State
I’ll beat you up.” Two days later, while in the was to safeguard pedestrians in using
neighborhood, David saw Bill coming toward crosswalks.
him. As Bill came up to David, Bill raised his (C) not prevail, because Ped assumed the risk
hand. Thinking Bill was Al and fearing bodily of injury when he crossed the street outside
harm, David struck Bill. the crosswalk.
(D) not prevail, because Driver’s conduct was
If Bill asserts a claim against David and David the actual cause of Ped’s harm.
relies on the privilege of self-defense, David will

22
92. Suspecting that students were using narcotics, 94. In which of the following situations is Defendant
the president of a private college arranged for most likely to be not guilty of the charge made?
local police to place concealed microphones
in several suites of the dormitory. Using these (A) Police arrested Thief and recovered goods
microphones, the college security officers he had stolen. At the direction of the
recorded a conversation in which Green, a police, Thief took the goods to Defendant.
student, offered to sell marijuana to another Defendant, believing the goods to be
student. The tape was turned over to the local stolen, purchased them. Defendant is
police, who played it for a local judge. The charged with attempting to receive stolen
judge issued a warrant to search Green’s room. property.
The room was searched by police, and marijuana (B) Defendant misrepresented his identity to
was discovered. secure a loan from a bank. The banker was
not deceived and refused to grant the loan.
Green is charged with unlawful possession of Defendant is charged with attempting to
narcotics. At trial, Green’s motion to prevent the obtain property by false pretenses.
introduction of the marijuana into evidence will (C) Believing that state law made it a
most probably be crime to purchase codeine without a
prescription, Defendant purchased, without
(A) denied, because the college president, in a prescription, cough syrup containing
loco parentis, had the responsibility of codeine. Unknown to Defendant, the
preventing unlawful activity by students statute had been repealed and codeine
under the president’s supervision. could be legally purchased without a
(B) denied, because there was probable cause prescription. Defendant is charged with
to make the search and police obtained a attempting to purchase codeine without a
warrant before commencing the search. prescription.
(C) granted, because Green’s privacy was (D) Defendant, intending to kill Selma, shot
unreasonably invaded. at Selma. Unknown to Defendant, Selma
(D) granted, because the electronic surveillance had died of a heart attack minutes before
was “fundamentally unfair.” Defendant shot at her. Defendant is
charged with attempted murder.
93. Tom had a heart ailment so serious that
his doctors had concluded that only a heart 95. Potts sued Dobbs on a product liability claim.
transplant could save his life. They therefore Louis testified for Potts. On cross-examination,
arranged to have him flown to Big City to have which of the following questions is the trial
the operation performed. judge most likely to rule improper?

Dan, Tom’s nephew, who stood to inherit from (A) “Isn’t it a fact that you are Potts’ close
him, poisoned him. The poison produced a friend?”
reaction which required postponing the journey. (B) “Isn’t it true that you are known in the
The plane on which Tom was to have flown community as ‘Louie the Lush’ because of
crashed, and all aboard were killed. By the your addiction to alcohol?”
following day, Tom’s heart was so weakened (C) “Didn’t you fail to report some income on
by the effects of the poison that he suffered a your tax return last year?”
heart attack and died. If charged with criminal (D) “Weren’t you convicted, seven years ago
homicide, Dan should be found in this court, of obtaining money under
false pretenses?”
(A) guilty.
(B) not guilty, because his act did not hasten 96. In an action to recover for personal injuries
the deceased’s death, but instead prolonged arising out of an automobile accident, Plaintiff
it by one day. calls Bystander to testify. Claiming the privilege
(C) not guilty, because the deceased was against self-incrimination, Bystander refuses
already suffering from a fatal illness. to answer a question as to whether she was at
(D) not guilty, because the poison was not the the scene of the accident. Plaintiff moves that
sole cause of death. Bystander be ordered to answer the question.
The judge should allow Bystander to remain
silent only if

23
(A) the judge is convinced that she will Questions 98–102 are based on the following fact
incriminate herself. situation.
(B) there is clear and convincing evidence that
she will incriminate herself. During 1976 a series of arsons, one of which damaged
(C) there is a preponderance of evidence that the Humongous Store, occurred in the City of Swelter.
she will incriminate herself. In early 1977 Swelter’s City Council adopted this
(D) the judge believes that there is some resolution:
reasonable possibility that she will
incriminate herself. The City will pay $10,000 for the arrest and
conviction of anyone guilty of any of the 1976
97. Blackacre is a three-acre tract of land with a arsons committed here.
small residence. Olga, the owner of Blackacre,
rented it to Terrence at a monthly rental of The foregoing was telecast by the city’s sole television
$200. After Terrence had been in possession of station once daily for one week. Subsequently,
Blackacre for several years, Terrence and Olga Humongous, by a written memorandum to Gimlet, a
orally agreed that Terrence would purchase private detective, proposed to pay Gimlet $200 “for
Blackacre from Olga for the sum of $24,000, each day’s work you actually perform in investigating
payable at the rate of $200 a month for 10 our fire.” Thereafter, in August 1977, the Swelter City
years, and also would pay the real estate taxes Council by resolution repealed its reward offer and
and the expenses of insuring and maintaining caused this resolution to be broadcast once daily for a
Blackacre. Olga agreed to give Terrence a deed week over two local radio stations, the local television
to Blackacre after five years had passed and station having meanwhile ceased operations. In
$12,000 had been paid on account and to accept September 1977, a Humongous employee voluntarily
from Terrence a note secured by a mortgage for confessed to Gimlet to having committed all of the
the balance. Terrence continued in possession 1976 arsons. Humongous’s president thereupon paid
of Blackacre and performed his obligations as Gimlet at the proposed daily rate for his investigation
orally agreed. Terrence, without consulting Olga, and suggested that Gimlet also claim the city’s reward,
made improvements for which he paid $1,000. of which Gimlet had been previously unaware. Gimlet
When Terrence had paid $12,000, he tendered a immediately made the claim. In December 1977, as
proper note and mortgage to Olga and demanded a result of Gimlet’s investigation, the Humongous
the delivery of the deed as agreed. Olga did employee was convicted of burning the store. The city,
not deny the oral agreement but told Terrence which has no immunity to suit, has since refused to
that she had changed her mind and refused to pay Gimlet anything, although he swears that he never
complete the transaction. Terrence then brought heard of the city’s repealer before claiming its reward.
an action for specific performance. Olga pleaded
the Statute of Frauds as her defense. If Olga 98. In which of the following ways could the city
wins, it will be because reward offer be effectively accepted?

(A) nothing Terrence could have done would (A) Only by an offeree’s return promise to
have overcome the original absence of a make a reasonable effort to bring about the
written agreement. arrest and conviction of an arsonist within
(B) the actions and payments of Terrence are the scope of the offer.
as consistent with his being a tenant as (B) Only by an offeree’s making the arrest and
with an oral contract. assisting in the successful conviction of an
(C) Terrence did not secure Olga’s approval arsonist within the scope of the offer.
for the improvements that he made. (C) By an offeree’s supplying information
(D) Olga has not received any unconscionable leading to arrest and conviction of an
benefit, and therefore Terrence is not arsonist within the scope of the offer.
entitled to equitable relief. (D) By an offeree’s communication of assent
through the same medium (television) used
by the city in making its offer.

24
99. With respect to duration, the city’s reward offer (B) The city’s offer was in the nature of a
was terminable bounty, so that the elements of contract are
not essential to the city’s liability.
(A) by lapse of time, on December 31 of the (C) The fact that the city attempted to revoke
year in which it was made. its offer only a few months after making it
(B) not by lapse of time, but only by effective demonstrated that the attempted revocation
revocation. was in bad faith.
(C) not by revocation, but only by lapse of a (D) Although there was no bargained-for
reasonable time. exchange between Gimlet and the city,
(D) either by lapse of a reasonable time or Gimlet’s claim for the reward is supported
earlier by effective revocation. by a moral obligation on the part of the
city.
100. If the city’s reward offer was revocable,
revocation could be effectively accomplished
only
Questions 103–105 are based on the following fact
(A) by publication in the legal notices of a situation.
local newspaper.
(B) in the same manner as made, i.e., by local Husband and Wife, walking on a country road, were
telecast at least once daily for one week. frightened by a bull running loose on the road. They
(C) in the same manner as made or by a climbed over a fence to get onto the adjacent property,
comparable medium and frequency of owned by Grower. After climbing over the fence,
publicity. Husband and Wife damaged some of Grower’s plants
(D) by notice mailed to all residents of the which were near the fence. The fence was posted with
city and all other reasonably identifiable a large sign, “No Trespassing.”
potential offerees.
Grower saw Husband and Wife and came toward them
101. Which of the following best characterizes the with his large watchdog on a long leash. The dog
relationship between Humongous and Gimlet? rushed at Wife. Grower had intended only to frighten
Husband and Wife, but the leash broke, and before
(A) A unilateral offer of employment by Grower could restrain the dog, the dog bit Wife.
Humongous which became irrevocable for
a reasonable number of days after Gimlet 103. If Wife asserts a claim based on battery against
commenced his investigation of the store’s Grower, will Wife prevail?
arson.
(B) An employment for compensation subject (A) Yes, because Grower intended that the dog
to a condition precedent that Gimlet frighten Wife.
succeed in his investigation. (B) Yes, because the breaking of the leash
(C) A series of daily bilateral contracts, establishes liability under res ipsa loquitur.
Humongous exchanging an express (C) No, because Wife made an unauthorized
promise to pay the daily rate for Gimlet’s entry on Grower’s land.
implied promise to pursue his investigation (D) No, because Grower did not intend to
with reasonable diligence. cause any harmful contact with Wife.
(D) A series of daily unilateral contracts,
Humongous exchanging an express 104. If Husband asserts a claim based on assault
promise to pay the daily rate for Gimlet’s against Grower, will Husband prevail?
daily activity of investigating the store’s
arson. (A) Yes, because the landowner did not have a
privilege to use excessive force.
102. In a suit by Gimlet against the city to recover (B) Yes, if Husband reasonably believed that
the $10,000 reward, which of the following, in the dog might bite him.
light of the facts given, most usefully supports (C) No, if the dog did not come in contact with
Gimlet’s claim? him.
(D) No, if Grower was trying to protect his
(A) The city was benefited as a result of property.
Gimlet’s services.

25
105. If Grower asserts a claim against Wife and is not affected by the Constitution of the
Husband for damage to his plants, will Grower United States.
prevail? (B) a statute prescribing the means of
disposing of the property of intestate
(A) Yes, because Wife and Husband entered decedents does not constitute invidious
on his land without permission. discrimination.
(B) Yes, because Grower had posted his (C) inheritance under intestate succession laws
property with a “No Trespassing” sign. is a privilege, not a right, and therefore is
(C) No, because Wife and Husband were not protected as property under the due
confronted by an emergency situation. process clause.
(D) No, because Grower used excessive force (D) its interest in promoting family life and in
toward Wife and Husband. encouraging the formal acknowledgment
of paternity gives the law a rational basis.


Questions 106–107 are based on the following fact
situation. 108. Alice conveyed Twinoaks Farm “to Barbara,
her heirs and assigns, so long as the premises
Ben was the illegitimate, unacknowledged child are used for residential and farm purposes,
of Fred. Fred died intestate, leaving neither spouse then to Charles and his heirs and assigns.”
nor any children other than Ben. The state’s law of The jurisdiction in which Twinoaks Farm is
intestate succession provides that an unacknowledged located has adopted the common law Rule
illegitimate child may not inherit his father’s property. Against Perpetuities unmodified by statute. As a
The spouse, all other blood relations, and the state consequence of the conveyance, Alice’s interest
are preferred as heirs over the unacknowledged in Twinoaks Farm is
illegitimate child. Ben filed suit in an appropriate court
alleging that the state statute barring an illegitimate (A) nothing.
child from sharing in a parent’s estate is invalid, and (B) a possibility of reverter.
that he should be declared lawful heir to his father’s (C) a right of entry for condition broken.
estate. (D) a reversion in fee simple absolute.

106. In challenging the validity of the state statute, 109. Lawnacre was conveyed to Celeste and Donald
Ben’s strongest argument would be that by a deed which, in the jurisdiction in which
Lawnacre is situated, created a cotenancy in
(A) there is no rational basis for preferring equal shares and with the right of survivorship.
as heirs collateral relatives and even the The jurisdiction has no statute directly applicable
state over unacknowledged children, to any of the problems posed.
and therefore the law violates the equal
protection clause. Celeste, by deed, conveyed “my undivided one-
(B) he has been deprived of property without half interest in Lawnacre” to Paul. Celeste has
due process because his fundamental right since died. In an appropriate action between
to inherit has been compromised without a Paul and Donald in which title to Lawnacre is at
compelling state need. issue, Donald will
(C) it violates the privileges and immunities
clause of the Fourteenth Amendment. (A) prevail, because he is the sole owner of
(D) it is a denial of procedural due Lawnacre.
process because it does not give the (B) prevail if, but only if, the cotenancy
unacknowledged illegitimate child an created in Celeste and Donald was a
opportunity to prove paternity. tenancy by the entirety.
(C) not prevail if he had knowledge of the
107. The state’s strongest defense of the statute would conveyance prior to Celeste’s death.
be that (D) not prevail, because Paul and Donald own
Lawnacre as tenants in common.
(A) the authority of a state over the disposition
of decedents’ property located in the state

26
Questions 110–114 are based on the following fact (A) not recover, because economic loss from
situation. injury to an employee is not within the
scope of Storekeeper’s duty.
Storekeeper, the owner of a large hardware store, sells (B) not recover, because Storekeeper was not
power saws for both personal and commercial use. He the manufacturer of the power saw.
often takes old power saws as trade-ins on new ones. (C) recover, because Storekeeper knew that the
The old power saws are then completely disassembled power saw was to be used in Purchaser’s
and rebuilt with new bearings by Storekeeper’s cabinetmaking business.
employees and sold by Storekeeper as “reconditioned (D) recover, because the reconditioned power
saws.” Purchaser, the owner and operator of a saw was the direct cause of Purchaser’s
cabinetmaking shop, informed Storekeeper that he loss of business.
wanted to buy a reconditioned circular saw for use in
his cabinetmaking business. However, the blade that 112. If Employee was injured while cutting plywood
was on the saw he picked out had very coarse teeth because the shaft holding the saw blade came
for cutting rough lumber. Purchaser told Storekeeper loose when a bearing gave way and the shaft and
that he wanted a saw blade that would cut plywood. blade flew off the saw, and if Employee asserts
Storekeeper exchanged the coarse blade for a new one a claim based on strict liability in tort against
with finer teeth that would cut plywood smoothly. The Storekeeper, Employee probably will
new blade was manufactured by Saw-Blade Company,
which uses all available techniques to inspect its (A) not recover unless Purchaser told
products for defects. The reconditioned saw had been Storekeeper that Employee would use the
manufactured by Power Saw Company. power saw.
(B) not recover if Employee failed to notice
The week after the saw was purchased, Employee, that the shaft was coming loose.
who works for Purchaser in Purchaser’s cabinet- (C) recover unless Employee knew that the
making shop, was injured while using the saw. shaft was coming loose.
Employee’s arm was severely cut. As a result, the (D) recover unless Storekeeper used all
cabinetmaking shop was shut down for a week until a possible care in reconditioning the power
replacement for Employee could be found. saw.

110. If Employee was injured while cutting plywood 113. If Employee was cutting a sheet of plywood,
because the shaft holding the saw blade came and while he was doing so, the saw blade flew
loose when a bearing gave way and the shaft and to pieces and severely cut Employee’s arm, and
blade flew off the saw, and if Employee asserts if Employee asserts a claim against Storekeeper,
a claim based on strict liability in tort against the theory on which Employee is most likely to
Power Saw Company, Employee will probably prevail is

(A) recover if the shaft that came loose was a (A) strict liability in tort.
part of the saw when it was new. (B) express warranty.
(B) recover, because Power Saw Company was (C) negligence, relying on res ipsa loquitur.
in the business of manufacturing dangerous (D) negligence, relying on the sale of an
machines. inherently dangerous product.
(C) not recover, because Employee was not the
buyer of the power saw. 114. If Employee was cutting a sheet of hard plastic,
(D) not recover, because the saw had been and while he was doing so, the saw blade flew to
rebuilt by Storekeeper. pieces and severely cut Employee’s arm, and if
Employee asserts a claim based on strict liability
111. If Employee was injured while cutting plywood in tort against Saw-Blade Company, the defense
because the shaft holding the saw blade came most likely to prevail is
loose when a bearing gave way and the shaft and
blade flew off the saw, and if Purchaser asserts (A) Employee did not purchase the saw blade.
a claim based on strict liability in tort against (B) the blade was being put to an improper use.
Storekeeper for loss of business because of the (C) Employee was contributorily negligent in
injury to Employee, Purchaser probably will using the blade to cut hard plastic.
(D) Saw-Blade Company used every available
means to inspect the blade for defects.

27
115. In which of the following situations is Defendant (D) inadmissible, because Miller had no reason
most likely to be guilty of common law murder? to respond to the bartender’s statement.

(A) Angered because his neighbor is having a 118. At the request of police, the teller who was
noisy party, Defendant fires a rifle into the robbed prepared a sketch bearing a strong
neighbor’s house. The bullet strikes and likeness to Miller, but the teller died in an
kills a guest at the party. automobile accident before Miller was arrested.
(B) During an argument, Harry slaps Defendant. At trial the prosecution offers the sketch. The
Angered, Defendant responds by shooting sketch is
and killing Harry.
(C) Defendant drives his car through a red (A) admissible as an identification of a person
light and strikes and kills a pedestrian who after perceiving him.
is crossing the street. (B) admissible as past recollection recorded.
(D) Using his fist, Defendant punches Walter (C) inadmissible as hearsay, not within any
in the face. As a result of the blow, Walter exception.
falls and hits his head on a concrete curb, (D) inadmissible as an opinion of the teller.
suffers a concussion, and dies.
119. Miller testified on direct examination that he had
never been in the First Bank of City. His counsel
Questions 116–120 are based on the following fact asked, “What, if anything, did you tell the police
situation. when you were arrested?” If his answer was “I
told them I had never been in the bank,” this
Miller is tried for armed robbery of the First Bank of answer would be
City.
(A) admissible to prove that Miller had never
116. The prosecution, in its case in chief, offers been in the bank.
evidence that when Miller was arrested one day (B) admissible as a prior consistent statement.
after the crime, he had a quantity of heroin and (C) inadmissible as hearsay, not within any
a hypodermic needle in his possession. This exception.
evidence should be (D) inadmissible, because it was a self-serving
statement by a person with a substantial
(A) admitted to prove Miller’s motive to motive to fabricate.
commit the crime.
(B) admitted to prove Miller’s propensity to 120. On cross-examination of Miller, the prosecutor
commit crimes. asks Miller whether he was convicted the
(C) excluded, because its probative value is previous year of tax fraud. This question is
substantially outweighed by the danger of
unfair prejudice. (A) proper to show that Miller is inclined to lie.
(D) excluded, because such evidence may be (B) proper to show that Miller is inclined to
offered only to rebut evidence of good steal money.
character offered by a defendant. (C) improper, because the conviction has
insufficient similarity to the crime charged.
117. The prosecutor offers the testimony of a (D) improper, because the probative value of
bartender who will say that when he saw the the evidence is outweighed by the danger
money in Miller’s wallet, he said, “You must of unfair prejudice.
have robbed a bank,” to which Miller made no
reply. This evidence is

(A) admissible to prove that Miller’s conduct 121. In an effort to relieve serious and persistent
caused the bartender to believe that Miller unemployment in the industrialized state of
robbed the bank. Onondaga, its legislature enacted a statute
(B) admissible as a statement made in the requiring every business with annual sales
presence of the defendant. in Onondaga of over one million dollars to
(C) inadmissible, because it would purchase goods and/or services in Onondaga
violate Miller’s privilege against self- equal in value to at least half of the annual
incrimination. sales in Onondaga of the business. Which of

28
the following constitutional provisions is the adjacent one-acre parcel owned by White. Whiteacre
strongest basis on which to attack this statute? has no access to any public way except an old, poorly
developed road which is inconvenient and cannot be
(A) The due process clause of the Fourteenth used without great expense. White sold Whiteacre
Amendment. to Breyer. The description used in the deed from
(B) The equal protection clause. White to Breyer was the same as that used in prior
(C) The commerce clause. deeds except that the portion of the description which
(D) The privileges and immunities clause of formerly said, “thence by land of Black, northeasterly
the Fourteenth Amendment. a distance of 200 feet, more or less,” was changed to
“thence by College Avenue as laid out on the Plan
122. Tess occupied an apartment in a building owned of Meadowview North 46, East 201.6 feet,” with full
by Len. She paid rent of $125 in advance each reference to the plan and its recording data.
month. During the second month of occupancy,
Tess organized the tenants in the building as a Breyer now seeks a building permit which will
tenants’ association and the association made show that Breyer intends to use College Avenue for
demands of Len concerning certain repairs and access to Whiteacre. Black objects to the granting of
improvements the tenants wanted. When Tess a building permit on the grounds that he has never
tendered rent for the third month, Len notified granted any right to White or Breyer to use College
her that rent for the fourth and subsequent Avenue. There are no governing statutes or ordinances
months would be $200 per month. Tess protested relating to the problem. Black brings an appropriate
and pointed out that all other tenants paid rent action in which the right of Breyer to use College
of $125 per month. Thereupon, Len gave the Avenue without an express grant from Black is at
required statutory notice that the tenancy was issue.
being terminated at the end of the third month.
By an appropriate proceeding, Tess contests 123. The best argument for Black in this action is that
Len’s right to terminate. If Tess succeeds, it will
be because (A) Breyer’s right must await the action of
appropriate public authorities to open
(A) a periodic tenancy was created by College Avenue as a public street, since
implication. no private easements arose by implication.
(B) the doctrine prohibiting retaliatory eviction (B) the Statute of Frauds prevents the
is part of the law of the jurisdiction. introduction of evidence which might
(C) the $200 rent demanded violates the prove the necessity for Breyer to use
agreement implied by the rate charged to College Avenue.
other tenants. (C) Breyer’s right to use College Avenue is
(D) the law implies a term of one year in the restricted to the assertion of a way by
absence of any express agreement. necessity and the facts preclude the
success of such a claim.
(D) Breyer would be unjustly enriched if he
Questions 123–124 are based on the following fact were permitted to use College Avenue.
situation.
124. The best argument for Breyer in this action is
Meadowview is a large tract of undeveloped that
land. Black, the owner of Meadowview, prepared
a development plan creating 200 house lots in (A) there is a way by necessity over
Meadowview with the necessary streets and public Meadowview’s lands to gain access to a
areas. The plan was fully approved by all necessary public road.
governmental agencies and duly recorded. However, (B) the deed from White to Breyer referred to
construction of the streets, utilities, and other aspects the recorded plan and therefore created a
of the development of Meadowview has not yet right to use the streets delineated on the
begun, and none of the streets can be opened as public plan.
ways until they are completed in accordance with the (C) the sale of lots in Meadowview by
applicable ordinances of the municipality in which reference to its plan creates private
Meadowview is located. easements in the streets shown in the plan.
(D) the recording of the plan is a dedication of
College Avenue, one of the streets laid out as part of the streets shown on the plan to public use.
the Meadowview development, abuts Whiteacre, an

29
Questions 125–129 are based on the following fact (D) An organization whose purpose
situation. was “to seek judicial invalidation of
unconstitutional laws.”
John Doe, the owner of a milk container manufacturing
firm, sought to focus public attention on the milk 127. The “Capitol steps” statute is probably
packaging law of the State of Clinton in order to have
it repealed. On a weekday at 12 noon, he delivered an (A) constitutional both on its face and as
excited, animated, and loud harangue on the steps of applied to Doe.
the State Capitol in front of the main entryway. An (B) constitutional on its face, but
audience of 200 onlookers, who gathered on the steps, unconstitutional as applied to Doe.
heckled him and laughed as he delivered his tirade. (C) unconstitutional on its face, because it
Doe repeatedly stated, gesturing expressively and applies to all working days.
making faces, that “the g–ddamned milk packaging (D) unconstitutional on its face, because it
law is stupid,” and that “I will strangle every one of concerns the State Capitol.
those g–ddamned legislators I can get hold of because
this law they created proves they are all too dumb to 128. A second state statute punishes “any person who
live.” After about 15 minutes, Doe stopped speaking, shall intentionally threaten the life or safety of any
and the amused crowd dispersed. public official for any act which he performed as
part of his public office.” Which of the following
There are three relevant statutes of the State of statements is correct concerning the possible
Clinton. The first statute prohibits “all speech making, punishment of Doe under the second statute?
picketing, and public gatherings of every sort on the
Capitol steps in front of the main entryway during the (A) The statute is unconstitutional on its face.
hours of 7:45 a.m.–8:15 a.m., 11:45 a.m.–12:15 p.m., (B) The statute is constitutional on its face, but
12:45 p.m.–l:15 p.m., and 4:45 p.m.–5:15 p.m. on Doe could not constitutionally be punished
Capitol working days.” under it for this speech.
(C) Doe could constitutionally be punished
125. If Doe is prosecuted under the “Capitol steps” under the statute for his speech.
statute and defends on constitutional grounds, (D) Doe could constitutionally be punished
which of the following best describes the proper under the statute for his speech, but only
burden of proof? if one or more legislators were actually
present when he delivered it.
(A) Doe would have to prove that the state did
not have a rational basis for enacting this 129. A third state statute, enacted in 1880, makes
statute. criminal “the utterance in any public place of
(B) Doe would have to prove that the state did any blasphemy or sacrilege.” Assume that there
not have a compelling need for this statute have been only a few recorded prosecutions
or that it had less restrictive means by under the 1880 statute. Doe is charged with
which it could satisfy that need. violating its proscriptions. The charge is based
(C) The state would have to prove that it had a wholly on the speech he delivered on the
rational basis for enacting this statute. steps of the Clinton State Capitol. Which of
(D) The state would have to prove that it had the following constitutional defenses to this
a compelling need for this statute and that prosecution under the 1880 statute would be the
there were no less restrictive means by LEAST likely to succeed?
which it could satisfy that need.
(A) This statute is vague and therefore violates
126. Which of the following possible plaintiffs other the due process clause of the Fourteenth
than Doe would be most likely to obtain an Amendment.
adjudication in a federal court on the validity of (B) This statute is an establishment of religion
the “Capitol steps” statute? and therefore violates the due process
clause of the Fourteenth Amendment.
(A) A state taxpayer in the highest tax bracket. (C) Application of this statute to Doe denies
(B) A politician intending to make a campaign him equal protection of the laws in
speech on the Capitol steps during a violation of the Fourteenth Amendment.
prohibited time. (D) Application of this statute to Doe denies
(C) A legislator who voted against the statute him freedom of speech in violation of the
because he thought it unconstitutional. Fourteenth Amendment.

30
Questions 130–131 are based on the following fact in Shoe Store featured “Jumpers,” a well-known make
situation. of basketball shoes. Photo sold the picture, greatly
enlarged, to Shoe Store and told Shoe Store that
Peri sued Denucci for a libelous letter received by Photo had Player’s approval to do so and that Player
Investigator. The authenticity and contents of the had consented to Shoe Store’s showing the enlarged
letter are disputed. picture in the window. Shoe Store made no effort
to ascertain whether Player had given his consent to
130. Peri’s attorney asks Investigator to testify that, Photo. In fact, Player did not even know that Photo
a week before receiving the libelous letter, he had taken the picture. Shoe Store put the enlarged
had written to Denucci inquiring about Peri. picture in the window with the display of “Jumpers”
The testimony is shoes. The college that Player attended believed that
Player had intentionally endorsed Shoe Store and
(A) admissible provided that this inquiry “Jumpers” shoes, and the college cancelled his athletic
was made in the regular course of scholarship.
Investigator’s business.
(B) admissible without production of the 132. If Player asserts a claim based on defamation
inquiry letter or the showing of its against Shoe Store, will Player prevail?
unavailability.
(C) inadmissible unless Peri’s attorney has (A) Yes, if Shoe Store was reckless in
given Denucci notice of Investigator’s accepting Photo’s statement that Photo had
intended testimony. Player’s approval.
(D) inadmissible unless the inquiry letter itself (B) Yes, because the defamatory material was
is shown to be unavailable. in printed form.
(C) No, if Shoe Store believed Photo’s
131. Investigator, if permitted, will testify that “I statement that Photo had Player’s approval.
received a letter that I cannot now find, which (D) No, because the picture of Player was not
read: defamatory per se.

‘Dear Investigator, 133. If Player asserts a claim based on invasion of


privacy against Shoe Store, will Player prevail?
You inquired about Peri. We fired him
last month when we discovered that he (A) Yes, because Photo had no right to take
had been stealing from the stockroom. Player’s picture.
(B) Yes, because Shoe Store, without Player’s
Denucci’.” permission, used Player’s picture for profit.
(C) No, because Player was already a
The testimony should be admitted in evidence basketball star who had received much
only if the publicity in the press.
(D) No, because Shoe Store believed it had
(A) jury finds that Investigator has quoted the permission to put the picture in the
letter precisely. window.
(B) jury is satisfied that the original letter is
unavailable.
(C) judge is satisfied that Investigator has
quoted the letter precisely. 134. Landover, the owner in fee simple of Highacre,
(D) judge finds that the original letter is an apartment house property, entered into an
unavailable. enforceable written agreement with VanMeer to
sell Highacre to VanMeer. The agreement
provided that a good and marketable title was to
be conveyed free and clear of all encumbrances.
Questions 132–133 are based on the following fact However, the agreement was silent as to the risk
situation. of fire prior to closing, and there is no applicable
statute in the state where the land is located. The
Photo, a freelance photographer, took a picture of premises were not insured. The day before the
Player in front of Shoe Store. Player was a nationally scheduled closing date, Highacre was wholly
known amateur basketball star who had received much destroyed by fire. When VanMeer refused to
publicity in the press. At the time, the window display close, Landover brought an action for specific

31
performance. If Landover prevails, the most the street, picked up a brick, and broke the display
likely reason will be that window of a liquor store. As he was reaching for a
bottle, the night watchman arrived. Startled, Defendant
(A) the failure of VanMeer to insure his turned and struck the watchman on the head with the
interest as the purchaser of Highacre bottle, killing him. Only vaguely aware of what was
precludes any relief for him. happening, Defendant returned to his car, consumed
(B) the remedy at law is inadequate in actions more liquor, and then drove off at a high speed. He
concerning real estate contracts and either ran a red light and struck and killed a pedestrian who
party is entitled to specific performance. was crossing the street.
(C) equity does not permit consideration of
surrounding circumstances in actions Relevant statutes define burglary to include “breaking
concerning real estate contracts. and entering a building not used as a dwelling with
(D) the doctrine of equitable conversion the intent to commit a crime therein.” Manslaughter
applies. is defined as the “killing of a human being in a
criminally reckless manner.” Criminal recklessness
135. Realco Realtors acquired a large tract of land is “consciously disregarding a substantial and
upon which Realco developed a mobile home unjustifiable risk resulting from the actor’s conduct.”
subdivision. The tract was divided into 60 lots, Murder is defined as “the premeditated and intentional
appropriate utilities were installed, and a plat killing of another or the killing of another in the
of the entire tract, including a Declaration of commission of committing rape, robbery, burglary, or
Restrictions, was properly drawn and recorded. arson.” Another statute provides that intoxication is
The Declaration of Restrictions included the not a defense to crime unless it negates an element of
following: “3. Ownership and/or occupancy are the offense.
restricted to persons 21 years of age or over,
one family per lot.” As the separate lots were Defendant was charged with the murder of the
sold, the deed to each lot included the following watchman and manslaughter in the death of the
provision: “As shown on recorded plat [properly pedestrian. Assume that he is tried separately on each
identified by page and plat book reference] and charge.
subject to the restrictions therein contained.”
One of the lots was purchased by Dawson, 136. At Defendant’s trial for the murder of the
who now resides in a mobile home on the lot watchman, the court should in substance
together with his wife and two children, aged charge the jury on the issue of the defense of
11 and 13. Other lot owners in the subdivision intoxication that
brought action against Dawson to enjoin further
occupancy by the children under 21 years of age. (A) intoxication is a defense to the underlying
If judgment is for Dawson, the issue that most crime of burglary if Defendant, due to
likely will determine the case will be whether drunkenness, did not form an intent to
commit a crime within the building, in
(A) the mobile home is treated as personalty or which case there can be no conviction for
realty. murder unless Defendant intentionally and
(B) the restriction constitutes an unlawful with premeditation killed the watchman.
restraint on alienation. (B) voluntary intoxication is not a defense to
(C) enforcement of the restriction is considered the crime of murder.
a violation of the equal protection clause of (C) Defendant is guilty of murder despite
the Fourteenth Amendment of the United his intoxication only if the state proves
States Constitution. beyond a reasonable doubt that the killing
(D) the terms of the restriction are expressly of the watchman was premeditated and
repeated verbatim in Dawson’s deed. intentional.
(D) voluntary intoxication is a defense to
the crime of murder if Defendant would
Questions 136–138 are based on the following fact not have killed the watchman but for his
situation. intoxication.

Defendant became intoxicated at a bar. He got into 137. At Defendant’s trial on the charge of manslaughter
his car and drove away. Within a few blocks, craving in the death of the pedestrian, his best argument
another drink, he stopped his car in the middle of would be that

32
(A) he was too intoxicated to realize that he Minicar, Inc., a minor United States car manufacturer,
was creating a substantial and unjustifiable seeks to enjoin enforcement of the Commission’s
risk in the manner in which he was rules.
operating his car.
(B) when he got into the car, his acts were not 139. The best argument that Minicar can make is that
voluntary because he was too intoxicated
to know where he was or what he was (A) legislative power may not be delegated by
doing. Congress to an agency in the absence of
(C) the pedestrian was contributorily clear guidelines.
negligent in failing to see Defendant’s car (B) the commerce power does not extend to
approaching. the manufacture of automobiles not used in
(D) he was too intoxicated to form any intent interstate commerce.
to voluntarily operate the automobile. (C) Minicar is denied due process of law
because it is not represented on the
138. The state’s best argument to counter Defendant’s Commission.
argument in Question 137 on the intoxication (D) the Commission lacks authority to enforce
issue in the manslaughter death of the pedestrian its standards because not all of its members
is that were appointed by the President.

(A) intoxication is no defense to the crime 140. The appropriate decision for the court is to
charged, because manslaughter is
historically a general intent crime. (A) allow the Commission to continue
(B) intoxication is a defense only to a specific investigating automobile safety and
intent crime, and no specific intent is making recommendations to Congress.
involved in the definition of the crime of (B) allow the Commission to prosecute
manslaughter. violations of the act but not allow it to
(C) conscious risk-taking refers to Defendant’s issue rules.
entire course of conduct, including (C) forbid the Commission to take any action
drinking with the knowledge that he might under the act.
become intoxicated and seriously injure or (D) order that all members of the Commission
kill someone while driving. be appointed by the President by and with
(D) whether Defendant was intoxicated or not the advice and consent of the Senate.
is not the crucial issue here; the real issue
is whether the manner in which Defendant
was operating his car can be characterized
under the facts as criminally reckless. Questions 141–142 are based on the following fact
situation.

Johnson wanted to purchase a used motor vehicle.
Questions 139–140 are based on the following fact The used car lot of Car Company, in a remote section
situation. away from town, was enclosed by a 10-foot chain-
link fence. While Johnson and Sales Representative,
The Federal Automobile Safety Act establishes certain an employee of Car Company, were in the used car
safety and performance standards for all automobiles lot looking at cars, a security guard locked the gate
manufactured in the United States. The Act creates a at 1:30 p.m., because it was Saturday and the lot was
five-member “Automobile Commission” to investigate supposed to be closed after 1:00 p.m. Saturday until
automobile safety, to make recommendations to Monday morning. At 1:45 p.m., Johnson and Sales
Congress for new laws, to make further rules Representative discovered that they were locked in.
establishing safety and performance standards, and
to prosecute violations of the act. The chairman is There was no traffic in the vicinity and no way in
appointed by the President, two members are selected which help could be summoned. After two hours,
by the President pro tempore of the Senate, and two Johnson began to panic at the prospect of remaining
members are selected by the Speaker of the House of undiscovered and without food and water until
Representatives. Monday morning. Sales Representative decided to
wait in a car until help should come. Johnson tried to
climb over the fence and, in doing so, fell and was

33
injured. Johnson asserts a claim against Car Company with possession of the whole of the
for damages for his injuries. property for the entire term.
(C) win, because of the implied warranty on
141. If Johnson’s claim is based on negligence, is the the part of the tenant to return the demised
defense of assumption of the risk applicable? premises in the same condition at the end
of the term as they were at the beginning.
(A) Yes, if a reasonable person would have (D) win, because the relationship of landlord
recognized that there was some risk of and tenant was unaffected by the
falling while climbing the fence. condemnation, thus leaving Tanner still
(B) Yes, because Sales Representative, as Car obligated to pay rent.
Company’s agent, waited for help.
(C) No, if it appeared that there was no other
practicable way of getting out of the lot Questions 144–145 are based on the following fact
before Monday. situation.
(D) No, because Johnson was confined as the
result of a volitional act. Statutes in the jurisdiction define criminal assault as
“an attempt to commit a criminal battery” and criminal
142. If Johnson’s claim is based on false imprisonment, battery as “causing an offensive touching.”
will Johnson prevail?
As Edward was walking down the street, a gust of
(A) Yes, because he was confined against his wind blew his hat off. Edward reached out, trying to
will. grab his hat, and narrowly missed striking Margaret
(B) Yes, because he was harmed as a result of in the face with his hand. Margaret, fearful of being
his confinement. struck by Edward, pushed Edward away.
(C) No, unless the security guard was
negligent in locking the gate. 144. If charged with criminal assault, Edward should
(D) No, unless the security guard knew that be found
someone was in the lot at the time the
guard locked the gate. (A) guilty, because he caused Margaret to be
in apprehension of an offensive touching.
(B) guilty, because he should have realized
that he might strike someone by reaching
143. Lester, the owner in fee simple of a small farm out.
consisting of 30 acres of land improved with (C) not guilty, because he did not intend to hit
a house and several outbuildings, leased the Margaret.
same to Tanner for a 10-year period. After two (D) not guilty, because he did not hit Margaret.
years had expired, the government condemned
20 acres of the property and allocated the 145. If charged with criminal battery, Margaret
compensation award to Lester and Tanner should be found
according to their respective interest so taken.
It so happened, however, that the 20 acres (A) guilty, because she intentionally pushed
taken embraced all of the farm’s tillable land, Edward.
leaving only the house, outbuildings, and a small (B) guilty, because she caused the touching of
woodlot. There is no applicable statute in the Edward whether she meant to do so or not.
jurisdiction where the property is located nor any (C) not guilty, because a push is not an
provision in the lease relating to condemnation. offensive touching.
Tanner quit possession, and Lester brought suit (D) not guilty, because she was justified in
against him to recover rent. Lester will pushing Edward.

(A) lose, because there has been a frustration


of purpose which excuses Tanner from
further performance of his contract to pay 146. Police were concerned about an increase in
rent. marijuana traffic in Defendant’s neighborhood.
(B) lose, because there has been a breach of One night, Police Officers, accompanied by
the implied covenant of quiet enjoyment dogs trained to sniff out marijuana, went into
by Lester’s inability to provide Tanner the backyard of Defendant’s house and onto
his porch. Defendant and his friend were inside

34
having dinner. The dogs acted as if they smelled “Isn’t it a fact that you are Drew’s first cousin?”
marijuana. Police Officers knocked on the back The question is
door. Defendant answered the door and let them
in. Defendant was immediately placed under (A) proper, because it goes to bias.
arrest. After a brief search, Police Officers (B) proper, because a relative is not competent
confiscated a large quantity of marijuana which to give reputation testimony.
they found in Defendant’s linen closet. (C) improper, because the question goes
beyond the scope of direct examination.
Defendant’s motion to prevent introduction of (D) improper, because the evidence being
the marijuana into evidence will most probably sought is irrelevant.
be
150. Drew called Warren to testify to alibi. On cross-
(A) denied, because the search was incident to examination of Warren, the prosecutor asked,
a valid arrest. “Weren’t you on the jury that acquitted Drew of
(B) denied, because Defendant permitted another criminal charge?” The best reason for
Police Officers to enter his house. sustaining an objection to this question is that
(C) granted, because under the circumstances
the police activity violated Defendant’s (A) the question goes beyond the scope of
reasonable expectations of privacy. direct examination.
(D) granted, because this kind of detection by (B) the probative value of the answer would
trained dogs has not been scientifically be outweighed by its tendency to mislead.
verified and cannot be the basis for (C) the question is leading.
probable cause. (D) prior jury service in a case involving a
party renders the witness incompetent.

Questions 147–150 are based on the following fact


situation.
151. Redirect examination of a witness must
Drew was tried for the July 21 murder of Victor. be permitted in which of the following
circumstances?
147. In his case in chief, Drew called his first
witness, Wilma, to testify to Drew’s reputation (A) To reply to any matter raised in cross-
in his community as a “peaceable man.” The examination.
testimony is (B) Only to reply to significant new matter
raised in cross-examination.
(A) admissible as tending to prove that Drew is (C) Only to reiterate the essential elements of
believable. the case.
(B) admissible as trying to prove that Drew is (D) Only to supply significant information
innocent. inadvertently omitted on direct
(C) inadmissible, because Drew has not examination.
testified.
(D) inadmissible, because reputation is not a 152. On March 1, Mechanic agreed to repair Ohner’s
proper way to prove character. machine for $5,000, to be paid on completion
of the work. On March 15, before the work was
148. Drew called William to testify that on July 20 completed, Mechanic sent a letter to Ohner with
Drew said that he was about to leave that day to a copy to Jones, telling Ohner to pay the $5,000
visit relatives in a distant state. The testimony is to Jones, who was one of Mechanic’s creditors.
Mechanic then completed the work.
(A) admissible, because it is a declaration of Which of the following, if true, would best serve
present mental state. Ohner as a defense in an action brought against
(B) admissible, because it is not hearsay. him by Jones for $5,000?
(C) inadmissible, because it is irrelevant.
(D) inadmissible, because it is hearsay, not (A) Jones was incapable of performing
within any exception. Mechanic’s work.
(B) Mechanic had not performed his work in a
149. Drew called Wilson to testify to alibi. On cross- workmanlike manner.
examination of Wilson, the prosecution asked,

35
(C) On March 1, Mechanic had promised 154. Assume for this question only that BCD’s
Ohner that he would not assign the termination of John’s employment was not
contract. wrongful. If John sues BCD for the reasonable
(D) Jones was not the intended beneficiary of value of his services, which of the following is
the Ohner-Mechanic contract. the most likely result?

(A) John will win, because BCD benefited as a
Questions 153–155 are based on the following fact result of John’s services.
situation. (B) John will win, because BCD made an
implied-in-fact promise to pay a reasonable
BCD, a manufacturer of computers, pays its commission for services that result in
salespeople a salary of $1,000 per month and a sales.
commission of five percent on billings actually (C) John will lose, because there is an express
rendered for machines that they sell. BCD salespeople contractual provision pre-empting the
are employed at will under written agreements subject of compensation for his services.
which provide that in order to receive a commission (D) John will lose, because he cannot perform
the salesperson must be in the employment of the his agreement to assist the customer for six
company when the bill is sent to the customer. months.

In 1976, John, a salesperson for BCD, worked for 155. Which of the following additional facts, if shown
eight months to get an order from Bobb Corporation by the evidence, would support a claim by John
for a large $750,000 computer. He consulted against BCD?
extensively with Bobb’s top executives and worked
with its operating personnel to develop detailed I. BCD terminated John because Franklin is
specifications for the new equipment. He also the son of the company’s president, who
promised Bobb, with BCD’s knowledge and approval, wanted his son to have the commission
to assist Bobb for six months after installation in instead of John.
making the equipment work. II. BCD and John were mistaken; John had in
fact exceeded his sales quotas for 1975 and
On January 1, 1977, Bobb signed an order, and on 1976.
March 1, the computer was installed. On March 15, III. John had worked for BCD as a salesperson
BCD fired John on the stated ground that he had failed for 20 years.
to meet his 1975 and 1976 sales quotas. John thought
that BCD was correct in this statement. Another (A) I only
salesperson, Franklin, was thereupon assigned to (B) II only
service the Bobb account. On March 31, BCD billed (C) I and II only
Bobb for the computer. (D) I, II, and III

153. Assume for this question only that BCD’s


termination of John’s employment was not
wrongful. If John, after demand and refusal, sues 156. In 1963, Hobson was appointed to a tribunal
BCD for the Bobb sale commission, which of established pursuant to a congressional act. The
the following is the most likely to result? tribunal’s duties were to review claims made
by veterans and to make recommendations to
(A) John will win, because he had procured the the Veterans Administration on their merits.
sale of the computer. Congress later abolished the tribunal and
(B) John will win, because he had promised established a different format for review of
Bobb to assist in making the equipment such claims. Hobson was offered a federal
work. administrative position in the same bureau at a
(C) BCD will win, because Franklin is entitled lesser salary. He thereupon sued the government
to the commission on a quantum meruit on the ground that Congress may not remove
basis. a federal judge from office during good
(D) BCD will win, because John was not behavior nor diminish his compensation during
employed as a BCD salesperson when continuance in office. Government attorneys
Bobb was billed for the computer. filed a motion to dismiss the action. The court
should

36
(A) deny the motion, because of the 158. If plaintiffs assert a claim based on private
independence of the federal judiciary nuisance, plaintiffs will
constitutionally guaranteed by Article III.
(B) deny the motion, because Hobson has (A) prevail, because Cattle Company’s activity
established a property right to his federal unreasonably interferes with plaintiffs’ use
employment on the tribunal. and enjoyment of their property.
(C) grant the motion, because Hobson lacks (B) prevail, because Cattle Company’s activity
standing to raise the question. constitutes an inverse condemnation of
(D) grant the motion, because Hobson was not their property.
a judge under Article III and is not entitled (C) not prevail, because Cattle Company has
to life tenure. operated the feedlot for more than 25
years.
(D) not prevail, because Cattle Company uses
Questions 157–158 are based on the following fact the most reasonable procedures to keep
situation. down flies and odors.

In 1940, Cattle Company paid $30,000 for a 150-


acre tract of agricultural land well suited for a cattle
feedlot. The tract was 10 miles from the city of 159. Alan, who was already married, went through a
Metropolis, then a community of 50,000 people, and marriage ceremony with Betty and committed
five miles from the nearest home. By 1976, the city bigamy. Carl, his friend, who did not know
limits extended to Cattle Company’s feedlot, and the of Alan’s previous marriage, had encouraged
city had a population of 350,000. About 10,000 people Alan to marry Betty and was best man at the
lived within three miles of the cattle-feeding operation. ceremony. If Carl is charged with being an
accessory to bigamy, he should be found
The Cattle Company land is outside the city limits and
no zoning ordinance applies. The Cattle Company land (A) not guilty, because his encouragement and
is now worth $300,000, and $25,000 has been invested assistance were not the legal cause of the
in buildings and pens. Cattle Company, conscious crime.
of its obligations to its neighbors, uses the best and (B) not guilty, because he did not have the
most sanitary feedlot procedures, including chemical mental state required for aiding and
sprays, to keep down flies and odors, and frequently abetting.
removes manure. Despite these measures, residents (C) guilty, because he encouraged Alan, and
of Metropolis complain of flies and odors. An action his mistake as to the existence of a prior
has been filed by five individual homeowners who marriage is not a defense to a charge of
live within half a mile of the Cattle Company feedlot. bigamy.
The plaintiffs’ homes are valued currently at $25,000 (D) guilty, because he was present when the
to $40,000 each. Flies in the area are five to ten times crime occurred and is thus a principal in
more numerous than in other parts of Metropolis, and the second degree.
extremely obnoxious odors are frequently carried by
the wind to the plaintiffs’ homes. The flies and odors 160. Darlene was arrested on a murder charge. She
are a substantial health hazard. was given Miranda warnings and refused to talk
further with the police. At trial, she testified in
157. If plaintiffs assert a claim based on public her own defense. She recounted in some detail
nuisance, plaintiffs will her whereabouts on the day of the crime and
explained why she could not have committed
(A) prevail if plaintiffs sustained harm different the crime. On cross-examination and over
from that suffered by the public at large. defense objection, the prosecution emphasized
(B) prevail if Cattle Company’s acts interfered the fact that she did not tell the police this story
with any person’s enjoyment of his following her arrest. The prosecution thereby
property. suggested that her testimony was false.
(C) not prevail, because only the state may
bring an action based on public nuisance. Darlene was convicted. On appeal, she claims
(D) not prevail, because plaintiffs came to the error in the prosecutor’s cross-examination. Her
nuisance. conviction will most probably be

37
(A) affirmed, because Darlene’s silence at to recover for damages to the lumberyard. The
time of arrest is tantamount to a prior decision should be for
inconsistent statement, giving rise to an
inference that the story was fabricated. (A) Maria, because the owner of an easement
(B) affirmed, because Darlene’s silence was has a duty to so maintain the easement as
not used as direct evidence but only for to avoid unreasonable interference with the
impeachment, a purpose consistent with use of the servient tenement by its lawful
legitimate cross-examination. possessor.
(C) reversed, because post-arrest silence (B) Maria, because the owner of an easement
constituted Darlene’s exercise of her is absolutely liable for any damage caused
Miranda rights and use of that silence to the servient tenement by the exercise of
against her at trial violated due process. the easement.
(D) reversed, because to require the defense (C) Reliable Electric Company, because the
to acquaint the prosecution with Darlene’s possessor of the servient tenement has a
testimony prior to trial would constitute duty to give the easement holder notice of
unconstitutional pretrial discovery. defective conditions.
(D) Reliable Electric Company, because an
161. Alice was held up at the point of a gun, an easement holder’s right to repair is a
unusual revolver with a red-painted barrel, right for his own benefit, and is therefore
while she was clerking in a neighborhood inconsistent with any duty to repair for the
grocery store. Dennis is charged with armed benefit of another.
robbery of Alice.

The prosecutor calls Winthrop to testify that, a Questions 163–166 are based on the following fact
week after the robbery of Alice, he was robbed situation.
by Dennis with a pistol that had red paint on the
barrel. Winthrop’s testimony is Sartorial, Inc., a new business enterprise about to
commence the manufacture of clothing, entered into
(A) admissible as establishing an identifying a written agreement to purchase all of its monthly
circumstance. requirements of a certain elasticized fabric for a period
(B) admissible as showing that Dennis was of three years from the Stretch Company at a specified
willing to commit robbery. unit price and agreed delivery and payment terms. The
(C) inadmissible, because it is improper agreement also provided:
character evidence.
(D) inadmissible, because its probative value is 1. The parties covenant not to assign this
substantially outweighed by the danger of contract.
unfair prejudice. 2. Payments coming due hereunder for the
first two months shall be made directly by
162. Maria is the owner and possessor of Goodacre, Sartorial to Virginia Wear and Son, Inc.,
on which there is a lumberyard. Maria conveyed a creditor of Stretch.
to Reliable Electric Company the right to
construct and use an overhead electric line Stretch promptly made an “assignment of the contract”
across Goodacre to serve other properties. to Finance Company as security for a $100,000 loan.
The conveyance was in writing, but the writing Sartorial subsequently ordered, took delivery of, and
made no provision concerning the responsibility paid Stretch the agreed price ($5,000) for Sartorial’s
for repair or maintenance of the line. Reliable requirement of the fabric for the first month of its
installed the poles and erected the electric line in operation.
a proper and workmanlike manner. Neither
Maria nor Reliable took any steps toward the 163. Which of the following accurately states the
maintenance or repair of the line after it was legal effect of the covenant not to assign the
built. Neither party complained to the other contract?
about any failure to repair. Because of the failure
to repair or properly maintain the line, it fell to (A) The covenant made the assignment to
the ground during a storm. In doing so, it caused Finance Company ineffective.
a fire in the lumberyard and did considerable (B) The covenant had no legal effect.
damage. Maria sued Reliable Electric Company (C) Stretch’s assignment was a breach
of its contract with Sartorial but was

38
nevertheless effective to transfer to (A) Stretch’s rights under its agreement with
Finance Company Stretch’s rights Sartorial were personal and therefore
against Sartorial. nonassignable.
(D) By normal interpretation, a covenant (B) Stretch’s “assignment of the contract” to
against assignment in a sale-of-goods Finance Company to secure a loan would
agreement applies to only the buyer, not normally be interpreted as a delegation
the seller. of Stretch’s duties under the contract as
well as an assignment of its rights; and its
164. Assume for this question only that the duties, owed to Sartorial, were personal
assignment from Stretch to Finance Company and therefore nondelegable.
was effective, and that Sartorial was unaware of (C) The original contract between Sartorial
the assignment when it paid Stretch the $5,000. and Stretch was unenforceable by either
Which of the following is correct? party for want of a legally sufficient
consideration for Stretch’s promise to
(A) Sartorial is liable to Finance Company for supply Sartorial’s requirements of the
$5,000. elasticized fabric.
(B) Stretch is liable to Finance Company for (D) Sartorial ceased in good faith to have any
$5,000. further requirements for elasticized fabric.
(C) Sartorial and Stretch are each liable to
Finance Company for $2,500.
(D) Neither Sartorial nor Stretch is liable to
Finance Company for any amount. 167. Seth owned a vacant lot known as Richacre.
Seth entered into a written contract with Bob
165. Assume for this question only that the to build a house of stated specifications on
assignment from Stretch to Finance Company Richacre and to sell the house and lot to Bob.
was effective, and that Virginia Wear and Son, The contract provided for an “inside date” of
Inc., did not become aware of the original April 1, 1977, and an “outside date” of May 1,
agreement between Sartorial and Stretch until 1977, for completion of the house and delivery
after Stretch’s acceptance of the $5,000 payment of a deed. Neither party tendered performance on
from Sartorial. Which of the following, if any, the dates stated. On May 3, 1977, Bob notified
is/are correct? Seth in writing of Bob’s election to cancel the
contract because of Seth’s failure to deliver title
I. Virginia Wear and Son, Inc., was an by May 1. On May 12, Seth notified Bob that
incidental beneficiary of the Sartorial- some unanticipated construction difficulties had
Stretch agreement. been encountered but that Seth was entitled to a
II. Virginia Wear and Son, Inc., has a prior reasonable time to complete in any event. The
right to Sartorial’s $5,000 payment as notification also included a promise that Seth
against either Stretch or Finance Company. would be ready to perform by May 29 and that
he was setting that date as an adjourned closing
(A) I only date. Seth obtained a certificate of occupancy
(B) II only and appropriate documents of title, and he
(C) Both I and II tendered performance on May 29. Bob refused.
(D) Neither I nor II Seth brought an action to recover damages for
breach of contract. The decision in the case will
166. Assume for this question only that, two weeks most likely be determined by whether
after making the $5,000 payment to Stretch,
Sartorial by written notice to Stretch terminated (A) Seth acted with due diligence in
the agreement for purchase of the elasticized completing the house.
fabric because market conditions had in fact (B) Bob can prove actual “undue hardship”
forced Sartorial out of the clothing manufacture caused by the delay.
business. In an immediate suit by Finance (C) the expressions “inside date” and “outside
Company against Sartorial for total breach, date” are construed to make time of the
which of the following would be useful in essence.
Sartorial’s defense? (D) there is a showing of good faith in Bob’s
efforts to terminate the contract.

39
168. Metterly, the owner in fee simple of Brownacre, (A) recover the full amount of his damages,
by quitclaim deed conveyed Brownacre to her because Motorist himself was not at fault.
daughter, Doris, who paid no consideration for (B) recover only a proportion of his damages,
the conveyance. The deed was never recorded. because Spouse was also at fault.
About a year after the delivery of the deed, (C) not recover, because Spouse was negligent
Metterly decided that this gift had been ill- and a wife’s negligence is imputed to her
advised. She asked Doris to destroy the deed, husband.
which Doris dutifully and voluntarily did. (D) not recover, because the failure of the
Within the month following the destruction brakes was the immediate cause of the
of the deed, Metterly and Doris were killed collision.
in a common disaster. Each of the successors
in interest claimed title to Brownacre. In an 170. If the jurisdiction has adopted “pure”
appropriate action to determine the title to comparative negligence and Spouse asserts a
Brownacre, the probable outcome will be that claim against Cross, Spouse will

(A) Metterly was the owner of Brownacre, (A) recover in full for her injuries, because
because Doris was a donee and therefore Motorist, who was driving the car in which
could not acquire title by quitclaim deed. she was riding, was not himself at fault.
(B) Metterly was the owner of Brownacre, (B) recover a proportion of her damages based
because title to Brownacre reverted to her on the respective degrees of her negligence
upon the voluntary destruction of the deed and that of Cross.
by Doris. (C) not recover, because but for the failure of
(C) Doris was the owner of Brownacre, the brakes the collision would not have
because her destruction of the deed to occurred.
Brownacre was under the undue influence (D) not recover, because she was negligent and
of Metterly. her negligence continued until the moment
(D) Doris was the owner of Brownacre, of impact.
because the deed was merely evidence
of her title, and its destruction was 171. If Motorist asserts a claim against Owner, will
insufficient to cause title to pass back to Motorist prevail?
Metterly.
(A) Yes, in negligence, because Owner knew
the brakes were faulty and failed to tell
Questions 169–171 are based on the following fact Motorist.
situation. (B) Yes, in strict liability in tort, because the
car was defective and Owner lent it to
Motorist arranged to borrow his friend Owner’s car Motorist.
to drive for one day while Motorist’s car was being (C) No, because Owner was a gratuitous
repaired. Owner knew that the brakes on his car lender, and thus his duty of care was slight.
were faulty and might fail in an emergency. Owner (D) No, because the failure of Spouse to tell
forgot to tell Motorist about the brakes when Motorist Motorist about the brakes was the cause in
picked up the car, but Owner did telephone Spouse, fact of Motorist’s harm.
Motorist’s wife, and told her about them. Spouse,
however, forgot to tell Motorist.

Motorist was driving Owner’s car at a reasonable 172. In which of the following situations is Defendant
rate of speed and within the posted speed limit, with most likely to be guilty of larceny?
Spouse as a passenger. Another car, driven by Cross,
crossed in front of Motorist at an intersection and in (A) Defendant took Sue’s television set, with
violation of the traffic signal. Motorist tried to stop, the intention of returning it the next day.
but the brakes failed, and the two cars collided. If the However, he dropped it and damaged it
brakes had been in proper working order, Motorist beyond repair.
could have stopped in time to avoid the collision. (B) Defendant went into Tom’s house and took
Motorist and Spouse were injured. $100 in the belief that Tom had damaged
Defendant’s car to that amount.
169. If Motorist asserts a claim against Cross,
Motorist will

40
(C) Mistakenly believing that larceny does in the course of legislative business and are immune
not include the taking of a dog, Defendant from scrutiny.
took his neighbor’s dog and sold it.
(D) Unreasonably mistaking George’s car for 174. Which of the following is the strongest
his own, Defendant got into George’s car constitutional argument supporting Kane?
in a parking lot and drove it home.
(A) Because of doctrines of federalism, federal
173. Acting on an anonymous telephone call, police law generally cannot be applied to state
went to Desmond’s apartment, knocked on the legislators acting in the course of their
door, and demanded to search the apartment official duties.
for narcotics. When Desmond refused, the (B) State legislators enjoy the protection of
police forced the door open and placed him the speech and debate clause of the United
under arrest. As they were removing him from States Constitution.
the apartment, Desmond offered to give the (C) A federal court must follow state law
officers “valuable information” in exchange for respecting the scope of legislative
his release. Before he could say anything else, immunity.
Desmond was given Miranda warnings by the (D) To apply the Federal Securities Act to state
police. Thereafter he told the police that he had legislators would violate the due process
stored some heroin in his friend’s apartment clause.
and that he and his friend had been going to
sell it. The heroin was recovered, and Desmond 175. Which of the following is the strongest argument
was prosecuted for conspiracy to sell narcotics against Kane’s constitutional defense?
and for possession of narcotics. At his trial,
Desmond moved to suppress his statements. (A) Congress has plenary power under the
Which of the following is Desmond’s best commerce clause.
argument in support of the motion to suppress? (B) Congress may impose liability on state
legislators as a means of guaranteeing a
(A) Desmond is entitled to know the identity republican form of government.
of his accuser, and the state cannot supply (C) Congress does not significantly interfere
this information. with state government by applying this law
(B) The police should have given Desmond to state legislators.
Miranda warnings prior to entry into the (D) Congress may impose liability on state
apartment, and the warnings were ineffectual legislators by virtue of the necessary and
once Desmond offered to give the police proper clause.
information.
(C) Desmond was intimidated by the forced
entry into the apartment, and since the
statements were involuntary and coerced, 176. Jim watched a liquor store furtively for some
their use against him would violate due time, planning to hold it up. He bought a
process of law. realistic-looking toy gun for the job. One night,
(D) The statements were fruits of an just before the store’s closing time, he drove to
unlawful arrest, and though the Miranda the store, opened the front door, and entered.
warnings may have been sufficient to He reached into his pocket for the toy gun, but
protect Desmond’s right against self- he became frightened and began to move back
incrimination, they were not sufficient to toward the front door. However, the shopkeeper
purge the taint of the illegal arrest. had seen the butt of the gun. Fearing a holdup,
the shopkeeper produced a gun from under the
counter, pointed it at Jim, and yelled, “Stop!”
Questions 174–175 are based on the following fact Jim ran to the door and the toy gun fell from his
situation. pocket. The shopkeeper fired. The shot missed
Jim, but struck and killed a passerby outside the
Kane, a member of the legislature of State, is store.
prosecuted in federal court for a violation of the
Federal Securities Act arising out of the activities of A statute in the jurisdiction defines burglary as
a state-owned corporation. Kane’s defense includes a “breaking and entering any building or structure
claim that the alleged wrongful acts were committed with the intent to commit a felony or to steal

41
therein.” On a charge of burglary, Jim’s best (A) be held strictly liable in tort for serving
defense would be that spoiled tuna.
(B) be held liable only if she was negligent.
(A) the intent required was not present. (C) not be held liable unless her conduct was
(B) the liquor store was open to the public. in reckless disregard of the safety of Guest.
(C) he had a change of heart and withdrew (D) not be held liable, because Guest was a
before committing any crime inside the store. social visitor.
(D) he was unsuccessful, and so at most could
be guilty of attempted burglary. 179. If Guest asserts a claim against Supermart, the
most likely result is that Guest will

Questions 177–179 are based on the following fact (A) recover on the theory of res ipsa loquitur.
situation. (B) recover on the theory of strict liability in tort.
(C) not recover, because Supermart gave
A water pipe burst in the basement of Supermart, a proper warning.
grocery store, flooding the basement and damaging (D) not recover, because Guest was not the
cases of canned goods on the floor. The plumbing purchaser of the cans.
contractor’s workmen, in repairing the leak, knocked
over several stacks of canned goods in cases, denting
the cans. After settling its claims against the landlord
for the water leak and against the plumbing contractor Questions 180–181 are based on the following fact
for the damage done by his workmen, Supermart put situation.
the goods on special sale.
Addle, who has been in the painting and contracting
Four weeks later, Dotty was shopping in Supermart. business for 10 years and has a fine reputation,
Several tables in the market were covered with contracts to paint Boone’s barn. Boone’s barn is a
assorted canned foods, all of which were dirty and standard red barn with a loft. The contract has no
dented. A sign on each of the tables read: “Damaged provision regarding assignment.
Cans—Half Price.”
180. If Addle assigns the contract to Coot, who has
Dotty was having Guest for dinner that evening and comparable experience and reputation, which of
purchased two dented cans of tuna, packed by Canco, the following statements is correct?
from one of the tables displaying the damaged cans.
Before Guest arrived, Dotty prepared a tuna casserole (A) Addle is in breach of contract.
which she and Guest ate. Both became ill, and the (B) Boone may refuse to accept performance
medical testimony established that the illness was by Coot.
caused by the tuna’s being unfit for consumption. The (C) Boone is required to accept performance
tuna consumed by Dotty and Guest came from the by Coot.
case that was at the top of one of the stacks knocked (D) There is a novation.
over by the workmen. The tuna in undamaged cans
from the same Canco shipment was fit for consumption. 181. If Addle assigns the contract to Coot and
thereafter Coot does not meet the contract
177. If Dotty asserts a claim against Canco based on specifications in painting Boone’s barn, Boone
negligence, the doctrine of res ipsa loquitur is
(A) has a cause of action against Addle for
(A) applicable, because the tuna was packed in damages.
a sealed can. (B) has a cause of action only against Coot for
(B) applicable, because Canco as the packer is damages.
strictly liable. (C) has a cause of action against Addle for
(C) not applicable, because the case of tuna damages only after he has first exhausted
had been knocked over by the workmen. his remedies against Coot.
(D) not applicable, because of the sign on the (D) does not have a cause of action against
table from which Dotty purchased the tuna. Addle for damages, because he waived his
rights against Addle by permitting Coot to
178. If Guest asserts a claim against Dotty, Dotty perform the work.
most likely will

42
182. Jackson and Brannick planned to break into a Questions 184–185 are based on the following fact
federal government office to steal food stamps. situation.
Jackson telephoned Crowley one night and asked
whether Crowley wanted to buy some “hot” food Rider, a bus passenger, sued Transit Company
stamps. Crowley, who understood that “hot” for injuries to his back from an accident caused
meant stolen, said, “Sure, bring them right over.” by Transit’s negligence. Transit denies that Rider
Jackson and Brannick then successfully executed received any injury in the accident.
their scheme. That same night they delivered the
food stamps to Crowley, who bought them for 184. Rider’s counsel seeks to introduce an affidavit
$500. Crowley did not ask when or by whom the he obtained in preparation for trial from Dr.
stamps had been stolen. All three were arrested. Bond, who has since died. The affidavit avers
Jackson and Brannick entered guilty pleas in that Dr. Bond examined Rider two days after
federal court to a charge of larceny in connection the Transit Company accident and found him
with the theft. Crowley was brought to trial in suffering from a recently incurred back injury.
the state court on a charge of conspiracy to steal The judge should rule the affidavit
food stamps.
(A) admissible, as a statement of present
On the evidence stated, Crowley should be found bodily condition made to a physician.
(B) admissible, as prior recorded testimony.
(A) guilty, because, when a new confederate (C) inadmissible, because it is irrelevant.
enters a conspiracy already in progress, he (D) inadmissible, because it is hearsay, not
becomes a party to it. within any exception.
(B) guilty, because he knowingly and willingly
aided and abetted the conspiracy and is 185. Transit Company calls Observer to testify that
chargeable as a principal. right after the accident, Rider told him that he
(C) not guilty, because, although Crowley had recently suffered a recurrence of an old
knew the stamps were stolen, he neither back injury. The judge should rule Observer’s
helped to plan nor participated or assisted testimony
in the theft.
(D) not guilty, because Jackson and Brannick (A) admissible, as an admission of a party
had not been convicted of or charged with opponent.
conspiracy, and Crowley cannot be guilty (B) admissible, as a spontaneous declaration.
of conspiracy by himself. (C) inadmissible, because it is irrelevant.
(D) inadmissible, because it is hearsay, not
183. Owens contracted to sell a tract of land, Overlea, within any exception.
to Painter by general warranty deed. However,
at the closing Painter did not carefully examine
the deed and accepted a quitclaim deed without
covenants of title. Painter later attempted to sell Questions 186–188 are based on the following fact
Overlea to Thompson, who refused to perform situation.
because Owens had conveyed an easement for
a highway across Overlea before Painter bought Johnston bought 100 bolts of standard blue wool, No.
the property. 1 quality, from McHugh. The sales contract provided
that Johnston would make payment prior to inspection.
Painter sued Owens for damages. Which of the The 100 bolts were shipped, and Johnston paid
following arguments will most likely succeed in McHugh. Upon inspection, however, Johnston
Owens’ defense? discovered that the wool was No. 2 quality. Johnston
thereupon tendered back the wool to McHugh and
(A) The existence of the easement does not demanded return of his payment. McHugh refused on
violate the contract. the ground that there is no difference between No. 1
(B) The mere existence of an easement which quality wool and No. 2 quality wool.
is not being used does not give rise to a
cause of action. 186. Which of the following statements regarding the
(C) Painter’s cause of action must be based on contract provision for preinspection payment is
the deed and not on the contract. correct?
(D) The proper remedy is rescission of the
deed. (A) It constitutes an acceptance of the goods.

43
(B) It constitutes a waiver of the buyer’s food and drink are dispensed for consumption on
remedy of private sale in the case of the premises.” Diner is the first drive-in in Little
nonconforming goods. City. For seven years Mr. and Mrs. Householder
(C) It does not impair a buyer’s right of have owned and lived in their single-family
inspection or his remedies. residence, which is across the street from Diner.
(D) It is invalid.
On opening day a brass band played in the
187. What is Johnston’s remedy because the wool was parking lot of Diner until midnight, and the
nonconforming? noise of cars and the unusual activity as a result
of the new business prevented the Householders
(A) Specific performance. from getting to sleep until well after midnight,
(B) Damages measured by the difference long after their usual time. Diner is heavily
between the value of the goods delivered patronized during the day and night by high-
and the value of conforming goods. school students. The noise of cars, the lights of
(C) Damages measured by the price paid plus the cars, the lights illuminating the parking lot at
the difference between the contract price Diner, and the noise from the loudspeaker of the
and the cost of buying substitute goods. ordering system prevent the Householders from
(D) None, since he waived his remedies by sleeping before midnight. Paper cups, napkins,
agreeing to pay before inspection. and other items from the drive-in are regularly
blown into the Householders’ front yard by the
188. Can Johnston resell the wool? prevailing wind. The traffic to and from Diner is
so heavy on the street in front of their house that
(A) Yes, in a private sale. the Householders are afraid to allow their small
(B) Yes, in a private sale but only after giving children to play in the front yard.
McHugh reasonable notice of his intention
to resell. The Householders have asserted a claim against
(C) Yes, but only at a public sale. Diner based on private nuisance. The most likely
(D) No. effect of the fact that Householders were in the
area before Diner is that it

(A) requires that the Householders’ interest be
189. Lord leased a warehouse building and the lot on given priority.
which it stood to Taylor for a term of 10 years. (B) is irrelevant because of the zoning
The lease contained a clause prohibiting Taylor ordinance.
from subletting his interest. Can Taylor assign (C) is irrelevant because conforming economic
his interest under the lease? uses are given priority.
(D) is some, but not controlling, evidence.
(A) Yes, because restraints on alienation of
land are strictly construed.
(B) Yes, because disabling restraints on Questions 191–192 are based on the following fact
alienation of land are invalid. situation.
(C) No, because the term “subletting” includes
“assignment” when the term is employed Jack and Paul planned to hold up a bank. They drove
in a lease. to the bank in Jack’s car. Jack entered while Paul
(D) No, because, even in the absence of an remained as lookout in the car. After a few moments,
express prohibition on assignment, a tenant Paul panicked and drove off.
may not assign without the landlord’s
permission. Jack looked over the various tellers, approached one,
and whispered nervously, “Just hand over the cash.
190. Diner, a drive-in hamburger and ice cream stand, Don’t look around, don’t make a false move—or it’s
recently opened for business in the suburban town your life.” The teller looked at the fidgeting Jack,
of Little City. Diner’s business hours are from laughed, flipped him a dollar bill, and said, “Go on,
9:00 a.m. to midnight. It is in an area that for 15 beat it.” Flustered, Jack grabbed the dollar and left.
years has been zoned for small retail businesses,
apartment buildings, and one- and two-family Soon after leaving the scene, Paul was stopped by the
residences. The zoning code specifies that “small police for speeding. Noting his nervous condition, the
retail businesses” include “businesses where police asked Paul if they might search the car. Paul

44
agreed. The search turned up heroin concealed in the 2. In 1950 Bob died, devising his interest to
lid of the trunk. his only child, “Charles, for life, and then
to Charles’ son, Sam, for life, and then to
191. Paul’s best defense to a charge of robbery would Sam’s children, their heirs and assigns.”
be that 3. In 1970 Bill died, devising his interest “to
my friend, Frank, his heirs and assigns.”
(A) Jack alone entered the bank. 4. In 1972 Frank conveyed by quitclaim deed
(B) Paul withdrew before commission of the “to Paul, his heirs and assigns whatever
crime when he fled the scene. right, title, and interest I own.”
(C) Paul had no knowledge of what Jack
whispered to the teller. Paul has never married. Paul has contracted to
(D) the teller was not placed in fear by Jack. convey marketable record title in the land to
Patrick. Can Paul do so?
192. The prosecution’s best argument to sustain the
validity of the search of Jack’s car would be that (A) Yes, without joinder of any other person in
the conveyance.
(A) the search was reasonable under the (B) Yes, if Charles, Sam, and Sam’s only
circumstances, including Paul’s nervous child (Gene, aged 25) will join in the
condition. conveyance.
(B) the search was incident to a valid arrest. (C) No, regardless of who joins in the
(C) Paul had, under the circumstances, conveyance, because Sam may have
sufficient standing and authority to consent additional children whose interests cannot
to the search. be defeated.
(D) exigent circumstances, including the (D) No, regardless of who joins in the
inherent mobility of a car, justified the conveyance, because a title acquired
search. by quitclaim deed is impliedly
unmerchantable.

193. Pace sued Def Company for injuries suffered Questions 195–196 are based on the following fact
when Pace’s car collided with Def Company’s situation.
truck. Def’s general manager prepared a report
of the accident at the request of the company’s Peter was rowing a boat on a mountain lake when a
attorney in preparation for the trial and delivered storm suddenly arose. Fearful that the boat might sink,
the report to the attorney. Pace now demands Peter rowed to a boat dock on shore and tied the boat
that the report be produced. Will production of to the dock. The shore property and dock were the
the report be required? private property of Owner.
(A) Yes, because business reports are not
generally privileged. While the boat was tied at the dock, Owner came
(B) No, because it is a privileged down and ordered Peter to remove the boat, because
communication from the client to the the action of the waves was causing the boat to rub
attorney. against a bumper on the dock. When Peter refused,
(C) No, because such reports contain hearsay. Owner untied the boat and cast it adrift. The boat sank.
(D) No, because such reports are self-serving.
Peter was wearing only a pair of swimming trunks.
194. The following facts concern a tract of land in a He had a pair of shoes and a parka in the boat,
state which follows general United States law. but they were lost when Owner set the boat adrift.
Each instrument is in proper form and recorded, Peter was staying at a cabin one mile from Owner’s
marital property rights were waived when property. The only land routes back were a short,
necessary, and each person named was adult and rocky trail that was dangerous during the storm, and
competent at the time of the named transaction. a 15-mile road around the lake. The storm continued
with heavy rain and hail, and Peter, having informed
1. In 1940 Oleg, the owner, conveyed his Owner of the location of his cabin, asked Owner
interest in fee simple “to my brothers Bob to take him back there in Owner’s car. Owner said,
and Bill, their heirs and assigns as joint “You got here by yourself and you’ll have to get back
tenants with right of survivorship.” home yourself.” After one hour the storm stopped,
and Peter walked home over the trail.

45
195. A necessary element in determining if Peter is failed to object to Alex’s statement and remained
liable for a trespass is whether silent after Alex had implicated him in the crime
should be ruled
(A) Owner had clearly posted his property
with a sign indicating that it was private (A) admissible, because his silence was an
property. implied admission by Sam that he had
(B) Peter knew that the property belonged to a participated in the crime.
private person. (B) admissible, because a statement of a
(C) Peter had reasonable grounds to believe participant in a crime is admissible against
that the property belonged to a private another participant.
person. (C) inadmissible, because, under the
(D) Peter had reasonable grounds to believe circumstances, there was no duty or
that his boat might be swamped and sink. responsibility on Sam’s part to respond.
(D) inadmissible, because whatever Alex may
196. If Peter asserts a claim against Owner for loss of have said has no probative value in a trial
the boat, the most likely result is that Owner will against Sam.

(A) have no defense under the circumstances.


(B) prevail, because Peter was a trespasser ab Questions 199–201 are based on the following fact
initio. situation.
(C) prevail, because the boat might have
damaged the dock. Duffer and Slicker, who lived in different suburbs
(D) prevail, because Peter became a trespasser 20 miles apart, were golfing acquaintances at the
when he refused to remove the boat. Interurban Country Club. Both were traveling
salesmen—Duffer for a pharmaceutical house and
Slicker for a widget manufacturer. Duffer wrote
Slicker by United States mail on Friday, October 8:
197. Park brought an action against Dan for injuries
received in an automobile accident, alleging I need a motorcycle for transportation to the
negligence in that Dan was speeding and country club, and will buy your Sujocki for
inattentive. Park calls White to testify that Dan $1,200 upon your bringing it to my home
had a reputation in the community of being a address above [stated in the letterhead] on or
reckless driver and was known as “daredevil before noon, November 12 next. This offer is not
Dan.” White’s testimony is subject to countermand.

(A) admissible as habit evidence. Sincerely,


(B) admissible, because it tends to prove that
Dan was negligent at the time of this [signed] Duffer
collision.
(C) inadmissible, because Dan has not offered Slicker replied by mail the following day:
testimony of his own good character.
(D) inadmissible to show negligence. I accept your offer, and promise to deliver the
bike as you specified.
198. Alex and Sam were arrested for holding up a gas
station. They were taken to police headquarters Sincerely,
and placed in a room for interrogation. As
a police officer addressing both started to [signed] Slicker
give them the Miranda warnings prior to the
questioning, Alex said, “Look, Sam planned This letter, although properly addressed, was
the damned thing and I was dumb enough to go misdirected by the postal service and not received by
along with it. We robbed the place—what else Duffer until November 10. Duffer had bought another
is there to say?” Sam said nothing. Sam was Sujocki bike from Koolcat for $1,050 a few hours
escorted into another room and a full written before.
confession was then obtained from Alex.
Koolcat saw Slicker at the Interurban Country Club on
If Sam is brought to trial on an indictment November 11 and said, “I sold my Sujocki to Duffer
charging him with robbery, the fact that Sam yesterday for $1,050. Would you consider selling me

46
yours for $950?” Slicker replied, “I’ll let you know in buyer (Duffer) rather than a prospective
a few days.” seller.
(D) Koolcat’s conversation with Slicker on
On November 12, Slicker took his Sujocki to Duffer’s November 11 terminated Duffer’s original
residence; he arrived at 11:15 a.m. Duffer was asleep offer and operated as an offer by Koolcat
and did not answer Slicker’s doorbell rings until 12:15 to buy Slicker’s Sujocki for $950.
p.m. Duffer then rejected Slicker’s bike on the ground
that he had already bought Koolcat’s.

199. In Duffer’s letter of October 8, what was the 202. In a suit attacking the validity of a deed
legal effect of the language: “This offer is not executed 15 years ago, Plaintiff alleges mental
subject to countermand”? incompetency of Joe, the grantor, and offers
in evidence a properly authenticated affidavit
(A) Under the Uniform Commercial Code of Harry, Joe’s brother. The affidavit, which
the offer was irrevocable until noon, was executed shortly after the deed, stated that
November 12. Harry had observed Joe closely over a period
(B) Such language prevented an effective of weeks, that Joe had engaged in instances of
acceptance by Slicker prior to noon, unusual behavior (which were described), and
November 12. that Joe’s appearance had changed from one
(C) At common law, such language created a of neatness and alertness to one of disorder
binding option in Slicker’s favor. and absentmindedness. The judge should rule
(D) Such language did not affect the offerer’s Harry’s affidavit
power of revocation of the offer.
(A) inadmissible as opinion.
200. In a lawsuit by Slicker against Duffer for breach (B) inadmissible as hearsay, not within any
of contract, what would the court probably exception.
decide regarding Slicker’s letter of October 9? (C) admissible as an official document.
(D) admissible as an ancient document.
(A) The letter bound both parties to a unilateral
contract as soon as Slicker mailed it.
(B) Mailing of the letter by Slicker did not, Questions 203–204 are based on the following fact
of itself, prevent a subsequent, effective situation.
revocation by Duffer of his offer.
(C) The letter bound both parties to a bilateral Oscar, the owner in fee simple, laid out a subdivision
contract, but only when received by Duffer of 325 lots on 150 acres of land. He obtained
on November 10. governmental approval (as required by applicable
(D) Regardless of whether Duffer’s offer had ordinances) and, between 1968 and 1970, he sold
proposed a unilateral or a bilateral contract, 140 of the lots, inserting in each of the 140 deeds the
the letter was an effective acceptance upon following provision:
receipt, if not upon dispatch.
“The Grantee, for himself and his heirs, assigns
201. What is the probable legal effect of Koolcat’s and successors, covenants and agrees that the
conversation with Slicker and report that he premises conveyed herein shall have erected
(Koolcat) had sold his Sujocki to Duffer on thereon one single-family dwelling and that no
November 10? other structure (other than a detached garage,
normally incident to a single-family dwelling)
(A) This report had no legal effect because shall be erected or maintained; and, further, that
Duffer’s offer was irrevocable until no use shall ever be made or permitted to be
November 12. made other than occupancy by a single family
(B) Unless a contract had already been formed for residential purposes only.”
between Slicker and Duffer, Koolcat’s
report to Slicker operated to terminate Because of difficulty encountered in selling the
Slicker’s power of accepting Duffer’s remaining lots for single-family use, in January 1971,
offer. Oscar advertised the remaining lots with prominent
(C) This report has no legal effect because emphasis: “These lots are not subject to any restriction
the offer had been made by a prospective and purchasers will find them adaptable to a wide
range of uses.”

47
203. Payne had purchased one of the 140 lots and (D) The facts do not establish a common
brought suit against Oscar to establish that the building or development scheme for the
remaining 185 lots, as well as the 140 sold entire subdivision.
previously, can be used only for residential
purposes by single families. Assuming that
procedural requirements have been met to 205. Adam and Bailey, brothers, operated an illicit
permit adjudication of the issue Payne has still. They customarily sold to anyone unless
tendered, which of the following is the most they suspected the person of being a revenue
appropriate comment? agent or an informant. One day when Adam
was at the still alone, he was approached by
(A) Oscar should win because the provision Mitchell, who asked to buy a gallon of liquor.
binds only the grantee. Mitchell was in fact a revenue officer. After
(B) The outcome turns on whether a common Adam had sold him the liquor, Mitchell revealed
development scheme had been established his identity. Adam grabbed one of the rifles
for the entire subdivision. that the brothers kept handy in case of trouble
(C) The outcome turns on whether there are with the law, and shot and wounded Mitchell.
sufficient land areas devoted to multiple- Other officers, hiding nearby, overpowered and
family uses within the municipality to arrested Adam.
afford reasonable opportunity for all
economic classes to move into the area Shortly thereafter, Bailey came on the scene.
so as to satisfy the standards of equal The officers in hiding had been waiting for him.
protection of the law. One of them approached him and asked to buy
(D) Payne should win under an application of liquor. Bailey was suspicious and refused to sell.
the doctrine which requires construction The officers nevertheless arrested him.
of deeds to resolve any doubt against the
grantor. Adam and Bailey were charged with conspiracy
to violate revenue laws, illegal selling of liquor,
204. Suppose that Oscar sold 50 lots during 1971 and battery of the officer.
without inserting in the deeds any provisions
relating to structures or uses. Doyle purchased On the charge of battery, which statement
one of the 50 lots and proposes to erect a service concerning Adam and Bailey is true?
station and to conduct a retail business for the
sale of gasoline, etc. Pringle purchased a lot (A) Neither is guilty.
from Boyer. Boyer had purchased from Oscar (B) Both are guilty.
in 1968 and the deed had the provision that is (C) Adam is guilty but Bailey is not, because
quoted in the fact situation. Pringle brings suit to the conspiracy had terminated with the
prevent Doyle from erecting the service station arrest of Adam.
and from conducting a retail business. In the (D) Adam is guilty but Bailey is not, because
litigation between Pringle and Doyle, which of Adam’s act was outside the scope of the
the following constitutes the best defense for conspiracy.
Doyle?
206. Odum owned Brightacre (a tract of land) in fee
(A) Oscar’s difficulty in selling with simple. He conveyed it “to Pike, his heirs and
provisions relating to use establishes a assigns; but if Farley shall be living 30 years
change in circumstances which renders any from the date of this deed, then to Farley, his
restrictions which may once have existed heirs and assigns.” The limitation “to Farley, his
unenforceable. heirs and assigns” is
(B) Enforcement of the restriction, in view of
the change of circumstances, would be an (A) valid, because Farley’s interest is a
unreasonable restraint on alienation. reversion.
(C) Since the proof (as stated) does not (B) valid, because the interest will vest, if at
establish a danger of monetary loss to all, within a life in being.
Pringle, Pringle has failed to establish one (C) valid, because Farley’s interest is vested
of the necessary elements in a cause of subject to divestment.
action to prevent Doyle from using his lot (D) invalid.
for business purposes.

48
207. Defendant was driving his automobile at a legal was a $20,000 mortgage on the home requiring
speed in a residential zone. A child darted out in monthly payments, each covering interest to
front of him and was run over and killed before date plus a portion of the principal. Which of
Defendant could prevent it. Defendant’s driver’s the following statements about the monthly
license had expired three months previously; payment is correct?
Defendant had neglected to check when it was
due to expire. Driving without a valid license is (A) Wanda must pay the full monthly payment.
a misdemeanor in the jurisdiction. On a charge (B) Wanda must pay a portion of the monthly
of manslaughter, Defendant should be found payment based on an apportionment of
the value between Wanda’s life estate and
(A) guilty under the misdemeanor-manslaughter Dixie’s remainder.
rule. (C) Wanda must pay the portion of the monthly
(B) guilty, because the licensing requirements payment which represents interest.
are to protect life, and failure to obey is (D) Dixie must pay the full monthly payment.
negligence.
(C) not guilty, because the offense was not the 210. The police, answering a complaint about noise,
proximate cause of the death. arrived at Sam’s apartment and found Sam’s
(D) not guilty, because there was no criminal wife dead on the living room floor. One of
intent. the officers turned to Sam and asked, “What
happened?” Sam replied, “She was a bitch and
208. Pauline, an unmarried female, was prominent in I took care of her.” At Sam’s trial, his statement
the women’s liberation movement. She recently should be ruled
gave birth to a baby and publicly announced
that she had no intention of marrying the father (A) admissible, because the statement was part
or disclosing his identity. The local newspaper, of the res gestae.
Journal, decided to do a series of articles on (B) admissible, because the statement was
Pauline entitled “The Perils of Pauline.” made at the scene, was essentially
volunteered, and was not a product of a
The first article about Pauline discussed her custodial interrogation.
parents. The article correctly stated that Mary, (C) inadmissible, because the statement
her mother, had died recently and that Frank, her is ambiguous and not necessarily
father, was still living. The article referred to the incriminatory.
fact that at the time of Pauline’s birth there were (D) inadmissible, because Sam was effectively
rumors that she had been born six months after in police custody and should have been
the marriage of Mary and Frank, that Frank was given the Miranda warnings.
not in fact her father, and that a person identified
as Albert, who had played minor roles in two
motion pictures, was her real father. Albert has Questions 211–213 are based on the following fact
lived in retirement for the last 10 years. situation.

If Pauline asserts a claim based on invasion of An ordinance of City makes it unlawful to park a
privacy against Journal for the statements in the motor vehicle on a City street within 10 feet of a fire
first article about her birth, and it is established hydrant. At 1:55 p.m., Parker, realizing that he must
that the statements are true, the most likely result be in Bank before it closed at 2:00 p.m., and finding
is that Pauline will no other space available, parked his automobile in
front of a fire hydrant on a City street. Parker then
(A) not prevail, because truth is a complete hurried into the bank, leaving his aged neighbor, Ned,
defense. as a passenger in the rear seat of the car. About five
(B) not prevail, because of her announcement minutes later, and while Parker was still in Bank,
concerning the birth of her own child. Driver was driving down the street. Driver swerved
(C) prevail, because the statements hold her up to avoid what he mistakenly thought was a hole in the
to ridicule and contempt. street and sideswiped Parker’s car. Parker’s car was
(D) prevail, because the statements are turned over on top of the hydrant, breaking the hydrant
embarrassing to her. and causing a small flood of water. Parker’s car was
severely damaged and Ned was badly injured. There is
209. Homer conveyed his home to his wife, Wanda, no applicable guest statute.
for life, remainder to his daughter, Dixie. There

49
211. If Ned asserts a claim against Parker, the most expert testimony and the teacher does not,
likely result is that Ned will per se, qualify as an expert.
(B) sustain the objection on the ground that
(A) recover, because Parker’s action was the best evidence of Smith’s handwriting
negligence per se. would be testimony by a person who had
(B) recover, because Parker’s action was a examined his writing more recently than
continuing wrong which contributed to 10 years ago.
Ned’s injuries. (C) overrule the objection on the ground that a
(C) not recover, because a reasonably prudent schoolteacher qualifies as an expert witness
person could not foresee injury to Ned as a for the purpose of identifying handwriting.
result of Parker’s action. (D) overrule the objection on the ground that
(D) not recover, because a violation of a city a layman may identify handwriting if he
ordinance does not give rise to a civil has seen the person in question write and
cause of action. has an opinion concerning the writing in
question.
212. If Parker asserts a claim against Driver for
damage to Parker’s automobile, the most likely
result is that Parker will Questions 215–216 are based on the following fact
situation.
(A) recover, because the purpose of the
ordinance is to provide access to the fire Paula sued for injuries she sustained in a fall in a
hydrant. hotel hallway connecting the lobby of the hotel with
(B) recover, because Driver’s negligence was a restaurant located in the hotel building. The hallway
later in time than Parker’s act of parking. floor was covered with vinyl tile. The defendants were
(C) not recover, because Parker was Horne, owner of the hotel building, and Lee, lessee of
contributorily negligent as a matter of law. the restaurant. The evidence was that the hallway floor
(D) not recover, because Parker’s action in had been waxed approximately an hour before Paula
parking unlawfully was a continuing wrong. slipped on it, and although the wax had dried, there
appeared to be excessive dried wax caked on several
213. If City asserts a claim against Driver for the of the tiles. Horne’s defense was that the hallway was
damage to the fire hydrant and Driver was a part of the premises leased to Lee over which he
negligent in swerving his car, his negligence is retained no control, and Lee denied negligence and
alleged contributory negligence.
(A) a cause in fact and a legal cause of City’s
harm. 215. Lee offered to prove by Marks, the restaurant
(B) a cause in fact, but not a legal cause, of manager, that in the week immediately preceding
City’s harm because Parker parked illegally. Paula’s fall at least 1,000 people had used the
(C) a legal cause, but not a cause in fact, of hallway in going to and from the restaurant, and
City’s harm because Parker’s car struck the Marks had neither seen anyone fall nor received
hydrant. reports that anyone had fallen. The trial judge
(D) neither a legal cause nor a cause in fact of should rule this evidence
City’s harm.
(A) admissible, because it tends to prove that
Paula did not use the care exercised by
reasonably prudent people.
214. John Smith has denied his purported signature (B) admissible, because it tends to prove that
on a letter which has become critical in a breach Lee was generally careful in maintaining
of contract suit between Smith and Miller. At the floor.
trial, Miller’s counsel calls Alice, a teacher, who (C) inadmissible, because Marks’ testimony is
testifies that she taught John Smith mathematics self-serving.
in school 10 years earlier, knows his signature, (D) inadmissible, because it does not bear on
and proposes to testify that the signature to the the issue of Lee’s exercise of due care on
letter is that of John Smith. this specific occasion.

Smith’s counsel objects. The trial judge should 216. If Paula offered to prove that the day after
she fell Horne had the vinyl tile taken up and
(A) sustain the objection on the ground that replaced with a new floor covering, the trial
identification of handwriting requires judge should rule the evidence

50
(A) admissible, because it is relevant to the (C) Defendant is charged with armed robbery.
issue of whether Horne retained control of He claims he was so drunk he did not
the hallway. know if the gun was loaded.
(B) admissible, because it is relevant to the (D) Defendant is charged with statutory rape
issue of awareness of the unsafe condition after he had sexual intercourse with a girl
of the hallway at the time of Paula’s fall. aged 15 in a jurisdiction where the age of
(C) inadmissible, because there was no consent is 16. Defendant claims he was
showing that the new floor covering would so drunk he did not realize the girl was a
be any safer than the old. minor.
(D) inadmissible, because to admit such would
discourage a policy of making repairs to
prevent further injury, regardless of fault. Questions 219–221 are based on the following fact
situation.

On May 1, Ohner telegraphed Byer, “Will sell you any
217. A newly enacted state criminal statute provides, or all of the lots in Grover subdivision at $5,000 each.
in its entirety, “No person shall utter to Details will follow in letter.” The letter contained all
another person in a public place any annoying, the necessary details concerning terms of payment,
disturbing, or unwelcome language.” Smith insurance, mortgages, etc., and provided, “This offer
followed an elderly woman for three blocks remains open until June 1.”
down a public street, yelling in her ear offensive
four-letter words. The woman repeatedly asked On May 2, after he had received the telegram but
Smith to leave her alone, but he refused. before he had received the letter, Byer telegraphed
Ohner, “Accept your offer with respect to lot 101.”
In the subsequent prosecution of Smith, the first Both parties knew that there were 50 lots in the Grove
under this statute, Smith subdivision and that they were numbered 101 through
150.
(A) can be convicted.
(B) cannot be convicted, because speech of the 219. For this question only, assume that Ohner and
sort described here may not be punished Byer were bound by a contract for the sale
by the state because of the First and of lot 101 for $5,000, that on May 3 Ohner
Fourteenth Amendments. telephoned Byer to say that because he had just
(C) cannot be convicted, because, though his discovered that a shopping center was going to
speech here may be punished by the state, be erected adjacent to the Grove subdivision,
the state may not do so under this statute. he would “have to have $6,000 for each of the
(D) cannot be convicted, because the average lots including lot 101,” that Byer thereupon
user of a public street would think his agreed to pay him $6,000 for lot 101, and that
speech/action here was amusing and on May 6 Byer telegraphed, “Accept your offer
ridiculous rather than “annoying,” etc. with respect to the rest of the lots.” Assuming
that two contracts were formed and that there is
no controlling statute, Byer will most likely be
218. In which of the following situations is required to pay
Defendant’s claim of intoxication most likely to
result in his being found not guilty? (A) only $5,000 for each of the 50 lots.
(B) only $5,000 for lot 101, but $6,000 for the
(A) Defendant is charged with manslaughter remaining 49 lots.
for a death resulting from an automobile (C) $6,000 for each of the 50 lots.
accident. Defendant, the driver, claims (D) $6,000 for lot 101, but only $5,000 for the
he was so drunk he was unable to see the remaining 49 lots.
other car involved in the accident.
(B) Defendant is charged with assault 220. For this question only, assume that on May 5
with intent to kill Watts as a result of Ohner telephoned Byer to say that he had sold
his wounding Watts by shooting him. lots 102 through 150 to someone else on May
Defendant claims he was so drunk he did 4, and that Byer thereafter telegraphed Ohner,
not realize anyone else was around when “Will take the rest of the lots.” Assume further
he fired the gun. that there is no controlling statute. In an action
by Byer against Ohner for breach of contract,
Byer probably will

51
(A) succeed, because Ohner had promised (A) Whether the press on which Philip was
him that the offer would remain open injured was visible from a public way.
until June 1. (B) Whether the maintenance of the area for
(B) succeed, because Ohner’s attempted the storage of discarded machinery was a
revocation was by telephone. private nuisance.
(C) not succeed, because Byer’s power of (C) Whether the maintenance of the area for
acceptance was terminated by Ohner’s sale the storage of discarded machinery was a
of the lots to another party. public nuisance.
(D) not succeed, because Byer’s power of (D) Whether Macco could have eliminated the
acceptance was terminated by an effective risk of harm without unduly interfering
revocation. with Macco’s normal operations.

221. For this question only, assume that on May 6


Byer telegraphed Ohner, “Will take the rest of Questions 223–224 are based on the following fact
the lots,” and that on May 8 Ohner discovered situation.
that he did not have good title to the remaining
lots. Which of the following would provide the Si was in the act of siphoning gasoline from Neighbor’s
best legal support for Ohner’s contention that car in Neighbor’s garage and without his consent
he was not liable for breach of contract as to the when the gasoline exploded and a fire followed.
remaining 49 lots? Rescuer, seeing the fire, grabbed a fire extinguisher
from his car and put out the fire, saving Si’s life and
(A) Impossibility of performance. Neighbor’s car and garage. In doing so, Rescuer was
(B) Unilateral mistake as to basic assumption. badly burned.
(C) Termination of the offer by Byer’s having
first contracted to buy lot 101. 223. If Rescuer asserts a claim against Si for personal
(D) Excuse by failure of an implied condition injuries, Rescuer will
precedent.
(A) prevail, because he saved Si’s life.
(B) prevail, because Si was at fault in causing
the fire.
222. Philip was a 10-year-old boy. Macco was a (C) not prevail, because Rescuer knowingly
company that sold new and used machinery. assumed the risk.
Macco stored discarded machinery, pending sale (D) not prevail, because Rescuer’s action was
for scrap, on a large vacant area it owned. This not a foreseeable consequence of Si’s
area was unfenced and was one-quarter mile conduct.
from the housing development where Philip
lived. Macco knew that children frequently 224. If Rescuer asserts a claim against Neighbor for
played in the area and on the machinery. Philip’s personal injuries, Rescuer will
parents had directed him not to play on the
machinery because it was dangerous. (A) prevail, because he saved Neighbor’s
property.
One day Philip was playing on a press in (B) prevail, because he acted reasonably in an
Macco’s storage area. The press had several emergency.
wheels, each geared to the next. Philip climbed (C) not prevail, because Neighbor was not at
on the largest wheel, which was about five feet fault.
in diameter. Philip’s weight caused the wheel to (D) not prevail, because Rescuer knowingly
rotate, his foot was caught between two wheels assumed the risk.
that were set in motion, and he was severely
injured.

A claim for relief was asserted by Philip through 225. Defendant was tried for robbery. Victim and
a duly appointed guardian. Macco denied Worth were the only witnesses called to testify.
liability and pleaded Philip’s contributory fault Victim testified that Defendant threatened her
as a defense. with a knife, grabbed her purse, and ran off with
it. Worth testified that he saw Defendant grab
In determining whether Macco breached a duty Victim’s purse and run away with it but that he
to Philip, which of the following is the most neither saw a knife nor heard any threats. On this
significant? evidence the jury could properly return a verdict
of guilty of

52
(A) robbery only. Questions 229–230 are based on the following fact
(B) larceny only. situation.
(C) either robbery or larceny.
(D) both robbery and larceny. Price sued Derrick for injuries Price received in an
automobile accident. Price claims that Derrick was
226. Congressional legislation authorizing marriages negligent in (a) exceeding the posted speed limit of 35
and divorces as a matter of federal law on m.p.h., (b) failing to keep a lookout, and (c) crossing
prescribed terms and conditions could most the center line.
easily be upheld if it
229. Bystander, Price’s eyewitness, testified on cross-
(A) applied only to marriages and divorces examination that Derrick was wearing a green
in which at least one of the parties was a sweater at the time of the accident. Derrick’s
member of the armed forces. counsel calls Wilson to testify that Derrick’s
(B) applied only to marriages performed by sweater was blue. Wilson’s testimony is
federal judges and to divorces granted by
federal courts. (A) admissible as substantive evidence of a
(C) implemented an executive agreement material fact.
seeking to define basic human rights. (B) admissible as bearing on Bystander’s
(D) applied only to marriages and divorces in truthfulness and veracity.
the District of Columbia. (C) inadmissible, because it has no bearing on
the capacity of Bystander to observe.
(D) inadmissible, because it is extrinsic
Questions 227–228. Assume for the purposes of these evidence of a collateral matter.
questions that you are counsel to the state legislative
committee that is responsible for real estate laws in 230. Derrick testified on his own behalf that he was
your state. going 30 m.p.h. On cross-examination, Price’s
counsel did not question Derrick with regard
227. The committee wants you to draft a statute to his speed. Subsequently, Price’s counsel
governing the recording of deeds that fixes calls Officer to testify that, in his investigation
priorities of title, as reflected on the public following the accident, Derrick told him that he
record, as definitely as possible. Which of was driving 40 m.p.h. Officer’s testimony is
the following, divorced from other policy
considerations, would best accomplish this (A) admissible as a prior inconsistent
particular result? statement.
(B) admissible as an admission.
(A) Eliminate the requirement of witnesses to (C) inadmissible, because it lacks a foundation.
deeds.
(B) Make time of recording the controlling (D) inadmissible, because it is hearsay, not
factor. within any exception.
(C) Make irrebuttable the declarations in the
deeds that valuable consideration was paid.
(D) Make the protection of bona fide
purchasers the controlling factor. Questions 231–232 are based on the following fact
situation and the diagram on the next page.
228. The committee wants you to draft legislation
to make all restrictions on land use imposed by In City of State Y, Maple Street is a local public
deeds (now or hereafter recorded) unenforceable thoroughfare, designated as a one-way street for
in the future so that public land-use planning northbound traffic. Pine Street is a public thoroughfare,
through zoning will have exclusive control in designated as a one-way street for eastbound traffic.
matters of land use. Which of the following Maple and Pine Streets intersect at right angles. The
is LEAST likely to be a consideration in the intersection is controlled by traffic lights. There are
drafting of such legislation? two sets of lights for traffic on Maple Street, one at
the northeast corner and one at the northwest corner.
(A) Compensation for property rights taken by There are two sets of lights for traffic on Pine Street,
public authority. one at the northeast corner and one at the southeast
(B) Impairment of contract. corner.
(C) Sovereign immunity.
(D) Police power. Trucker was making a delivery to a market on the east
side of Maple Street, just north of its intersection

53
(D) neither an actual nor a legal cause of
Driver’s injuries.

232. If Driver asserts a claim against City, the theory


on which he has the best chance of prevailing is
that City

(A) is strictly liable for harm caused by a


defective traffic signal.
(B) was negligent in not replacing the broken
pole within 72 hours.
(C) has an absolute duty to maintain installed
traffic signals in good operating order.
(D) created a dangerous trap by not promptly
replacing the broken pole.

Questions 233–235 are based on the following fact


situation.

Congress provides by statute that any state that fails to


with Pine Street. There being insufficient space for his prohibit automobile speeds of over 55 miles per hour
truck and enclosed trailer, he parked it with the rear on highways within the state shall be denied all federal
of the trailer extending entirely across the crosswalk highway construction funding. The state of Atlantic,
on the north side of the intersection. The height of the one of the richest and most highway-oriented states in
trailer was such that it entirely obscured the traffic the country, refuses to enact such a statute.
light on the northeast corner from the view of traffic
moving east on Pine Street. Unknown to Trucker, the 233. Which of the following potential plaintiffs is most
traffic light at the southeast corner was not functioning, likely to be able to obtain a judicial determination
because a collision 72 hours earlier had knocked down of the validity of this federal statute?
the pole from which the light was suspended.
(A) A taxpayer of the United States and the
Visitor, on his first trip to City, was driving east on state of Atlantic who wants his state to
Pine Street. Not seeing any traffic light or pole, he get its fair share of federal tax monies for
entered the intersection at a time when the light was highways, and who fears that, if it does
red for eastbound traffic and green for northbound not, his state taxes will be increased to pay
traffic. Driver, proceeding north on Maple Street and for the highway construction in the state
seeing the green light, entered the intersection without of Atlantic that federal funds would have
looking for any cross traffic and struck Visitor’s car. financed.
Driver received personal injuries, and Visitor’s car (B) Contractors who have been awarded
was damaged severely as a result of the impact. contracts by the state of Atlantic for
specific highway construction projects,
Statutes of State Y make it a misdemeanor (1) to which contracts are contingent on payment
park a motor vehicle so that any part projects into to the state of the federal highway
a crosswalk and (2) to enter an intersection contrary construction funds to which it would
to a traffic signal. otherwise be entitled.
(C) An automobile owner who lives in the
231. If Driver asserts a claim against Trucker and state of Atlantic and regularly uses its
establishes that Trucker was negligent, the likely highway system.
result is that Trucker’s negligence is (D) An organization dedicated to keeping the
federal government within the powers
(A) a legal but not an actual cause of Driver’s granted it by the Constitution.
injuries.
(B) an actual but not a legal cause of Driver’s 234. The strongest argument that can be made in
injuries. support of the constitutionality of this federal
(C) both an actual and a legal cause of Driver’s statute is that
injuries.

54
(A) the states ceded their authority over (C) both burglary and larceny.
highways to the national government when (D) neither burglary nor larceny.
the states accepted federal grants to help
finance their highways. 237. Henry hated Wanda, his former wife, for
(B) the federal government can regulate the divorcing him and marrying John a short time
use of state highways without limitation thereafter. About a month after Wanda married
because the federal government paid for John, Henry secretly entered Wanda and John’s
some of their construction costs. rented apartment during their absence by using a
(C) Congress could reasonably believe that the master key. Henry placed a microphone behind
55-m.p.h. speed limit will assure that the the nightstand in the bedroom of the apartment,
federal money spent on highways results in drilled a hole in the nearby wall, and poked the
greater benefit than harm to the public. wires from the microphone through the hole into
(D) a recent public opinion survey the space in the wall, with the result that the
demonstrated that 90 percent of the people microphone appeared to be connected to wires
in this country support a 55-m.p.h. speed going into the adjoining apartment. Actually the
limit. microphone was not connected to anything.
Henry anticipated that Wanda would discover
235. The federal statute relating to disbursement of the microphone in a few days and would be
highway funds conditioned on the 55-m.p.h. upset by the thought that someone had been
speed limit is probably listening to her conversations with John in their
bedroom.
(A) unconstitutional.
(B) constitutional only on the basis of the Shortly thereafter, John noticed the wires
spending power. behind the nightstand and discovered the hidden
(C) constitutional only on the basis of the microphone. He then called Wanda and showed
commerce power. her the microphone and wires. Wanda fainted
(D) constitutional on the basis of both the and, in falling, struck her head on the nightstand
spending power and the commerce power. and suffered a mild concussion. The next day
John telephoned Henry and accused him of
planting the microphone. Henry laughingly
admitted it. Because of his concern about Wanda
236. Defendant visited a fellow college student, and his anger at Henry, John is emotionally
James, in James’s dormitory room. They drank upset and unable to go to work.
some beer. James produced a box containing
marijuana cigarettes and asked if Defendant If Wanda asserts a claim against Henry based on
wanted one. Defendant, afraid of being caught, infliction of mental distress, the fact that John
declined and urged James to get rid of the was the person who showed her the microphone
marijuana. James refused. will

Shortly thereafter, both went out to get more (A) relieve Henry of liability, because John
beer, leaving the door to James’s room standing was careless in so doing.
open. Making an excuse about having dropped (B) relieve Henry of liability, because John’s
his pen, Defendant went back into James’s room. conduct was the immediate cause of
Still apprehensive about their being caught with Wanda’s harm.
the marijuana cigarettes, he took the cigarettes (C) not relieve Henry of liability, because
and flushed them down the toilet. He was sure Henry’s goal was achieved.
James was too drunk to notice that the cigarettes (D) not relieve Henry of liability, because the
were missing. conduct of a third person is irrelevant in
emotional distress cases.
Defendant is charged with larceny and burglary
(the latter defined in the jurisdiction as breaking
and entering the dwelling of another with intent Questions 238–239 are based on the following fact
to commit any felony or theft). He should be situation.
found guilty of
Trease owned Hilltop in fee simple. By his will,
(A) burglary only. he devised as follows: “Hilltop to such of my
(B) larceny only. grandchildren who shall reach the age of 21; and by

55
this provision I intend to include all grandchildren After receiving a bill from Doctor for $1,000, Victim
whenever born.” At the time of his death, Trease had immediately wrote Doctor explaining that he was
three children and two grandchildren. unable to pay and enclosing a copy of his contract
with First.
238. Courts hold such a devise valid under the
common law Rule Against Perpetuities. What is Victim then asked First about payment of this bill,
the best explanation of that determination? but First requested a release from their employment
contract, stating that he would like to refer Victim’s
(A) All of Trease’s children would be claim to attorney Second and that Second was willing
measuring lives. to represent Victim in the pending lawsuit. Victim
(B) The rule of convenience closes the class of wrote a letter to First releasing him from their contract
beneficiaries when any grandchild reaches and agreeing to Second’s representation. A copy of
the age of 21. this letter was sent to Doctor. Second subsequently
(C) There is a presumption that Trease promised First to represent Victim and soon negotiated
intended to include only those a settlement of Victim’s claim against Driver which
grandchildren born prior to his death. netted $1,000, all of which was paid by Victim to
(D) There is a subsidiary rule of construction creditors other than Doctor. Victim remains insolvent.
that dispositive instruments are to be
interpreted so as to uphold interests rather 240. In an action by Doctor against Victim to recover
than to invalidate them under the Rule $1,000, Doctor’s best theory of recovery is that
Against Perpetuities. Doctor

239. Which of the following additions to or changes (A) is a creditor beneficiary of the employment
in the facts of the preceding question would contract between Victim and First.
produce a violation of the common law Rule (B) is a donee beneficiary of the employment
Against Perpetuities? contract between Victim and First.
(C) provided services essential to the
(A) A posthumous child was born to Trease. preservation of Victim’s health.
(B) Trease’s will expressed the intention to (D) has a claim based upon an implied-in-fact
include all afterborn grandchildren in the contract with Victim.
gift.
(C) The instrument was an inter vivos 241. In an action by Doctor against First upon First’s
conveyance rather than a will. employment contract with Victim, First is likely
(D) Trease had no grandchildren living at the to argue in defense that
time of his death.
(A) the anti-assignment clause in First’s
contract with Victim is void as against
public policy.
Questions 240–243 are based on the following fact (B) First has relied to his detriment on
situation. Victim’s letter of release.
(C) third parties cannot acquire valid claims
Victim, injured by Driver in an auto accident, under an attorney-client contract.
employed attorney First to represent him in the matter. (D) Doctor has not materially changed
Victim was chronically insolvent and expressed doubt his position in reliance upon First’s
whether he could promptly get necessary medical employment contract.
treatment. Accordingly, First wrote into their contract
his promise to Victim “to pay from any settlement 242. In an action by Doctor against First upon First’s
with Driver compensation to any physician who employment contract with Victim, if First
provides professional services for Victim’s injuries.” attempts to use Victim’s release as a defense,
The contract also provided that First’s duties were Doctor is likely to argue that
“nonassignable.” First immediately filed suit against
Driver. Victim then sought and received medical (A) the release was ineffective, because Doctor
treatment, reasonably valued at $1,000, from Doctor, had impliedly assented to the Victim-First
but failed to inform Doctor of First’s promise. contract.
(B) the release was ineffective, because Victim
would thereby be unjustly enriched.

56
(C) there was no consideration for Victim’s (D) inadmissible, because of the Dead Man’s
release of First. Statute.
(D) First’s contract duties were too personal to
be effectively delegated to Second.

243. In an action by Doctor against Second, Second is Questions 246–247 are based on the following fact
most likely to argue on these facts that situation.

(A) Second made only a gratuitous promise to Owner and his employee, Driver, consult Attorney
First. about a motor vehicle collision resulting in a suit by
(B) at the time Second promised to represent Litigant against Owner and Driver as joint defendants.
Victim, Doctor was only a member of an Attorney calls Irving, his investigator, into the
unidentified class of beneficiaries. conference to make notes of what is said, and those
(C) there is insufficient evidence to support a present discuss the facts of the collision and Owner’s
finding that Doctor was either a creditor or insurance. Owner thereafter files a cross-claim against
donee beneficiary of Second’s promise to Driver for indemnity for any damages obtained by
First. Litigant.
(D) there is insufficient evidence to support a
finding that Doctor substantially changed 246. Litigant calls Driver to testify in Litigant’s case
his position in reliance on Second’s in chief to admissions made by Owner in the
promise. conference. On objection by Owner, the court
should rule that Driver’s testimony is

Questions 244–245 are based on the following fact (A) admissible, because of the presence of
situation. persons in the conference other than
Attorney and Owner.
Peters sued Davis for $100,000 for injuries received (B) admissible, because Driver is an adverse
in a traffic accident. Davis charged Peters with party in the lawsuit.
contributory negligence and alleged that Peters failed (C) inadmissible, because of the attorney-client
to have his lights on at a time when it was dark privilege.
enough to require them. (D) inadmissible, because the best evidence is
Irving’s notes of the conference.
244. Davis calls Bystander to testify that Passenger,
who was riding in Peters’s automobile and who 247. Driver calls Irving in his defense against the
also was injured, confided to Bystander at the cross-claim. He seeks to have Irving testify to an
scene of the accident that “we should have had admission made by Owner in the conference. On
our lights on.” Bystander’s testimony is objection by Owner, the court should rule that
Irving’s testimony is
(A) admissible as an admission of a party
opponent. (A) admissible, because the attorney-client
(B) admissible as a declaration against interest. privilege does not apply, in suits between
(C) inadmissible, because it is hearsay not those conferring with him, to joint
within any exception. consultations with an attorney.
(D) inadmissible, because it is opinion. (B) admissible, because the attorney-client
privilege does not apply to testimony by
245. Davis offers to have Bystander testify that one who does not stand in a confidential
he was talking to Witness when he heard relationship with the person against whom
the crash and heard Witness, now deceased, the evidence is offered.
exclaim, “That car doesn’t have any lights on.” (C) admissible, because the conference was
Bystander’s testimony is not intended to be confidential, since it
concerned anticipated testimony in open
(A) admissible as a statement of present sense court.
impression. (D) inadmissible, because Owner has not
(B) admissible, because Witness is not waived the attorney-client privilege.
available to testify.
(C) inadmissible as hearsay, not within any
exception.

57
Questions 248–249 are based on the following fact for breach of contract, which of the following
situation. can Brown successfully use as a defense?

Dave is a six-year-old boy who has a well-deserved I. The Clark-Green agreement permitting the
reputation for bullying younger and smaller children. digging of the channel across Clark’s lot
His parents have encouraged him to be aggressive and was not in writing.
tough. Dave, for no reason, knocked down, kicked, II. The Green-Brown agreement was not in
and severely injured Pete, a four-year-old boy. A writing.
claim for relief has been asserted by Pete’s parents for
their medical and hospital costs and for Pete’s injuries. (A) I only
(B) II only
248. If the claim is asserted against Dave’s parents, (C) Both I and II
the most likely result is that they will be (D) Neither I nor II

(A) liable, because parents are strictly liable 251. Assume that Green paid the $2,500 on March
for the torts of their children. 15 and that Brown completed the boathouse
(B) liable, because Dave’s parents encouraged according to specifications, but that Green
him to be aggressive and tough. then refused to pay the second installment and
(C) not liable, because a six-year-old cannot repudiated the contract. Assume further that the
commit a tort. absence of a writing is not raised as a defense.
(D) not liable, because parents cannot be held Which of the following is/are correct?
liable for the tort of a child. I. Brown has a cause of action against Green
and his damages will be $2,500.
249. If the claim is asserted against Dave, the most II. Brown can refuse to dig the channel and
likely result is that Dave will be will not be liable for breach of contract.

(A) liable, because he intentionally harmed Pete. (A) I only


(B) liable, because, as a six-year-old, he (B) II only
should have known that his conduct was (C) Both I and II
wrongful. (D) Neither I nor II
(C) not liable, because a child under seven is
not liable in tort. 252. Assume that Green paid the $2,500 on March
(D) not liable, because he is presumed to be 15, that Brown completed the boathouse, that
under his parents’ control and they have Green paid the second installment of $2,500, and
the sole responsibility. that Brown completed the digging of the channel
but not until July 1. Assume further that the
absence of a writing is not raised as a defense.
Which of the following is/are correct?
Questions 250–252 are based on the following fact
situation. I. Green has a cause of action against Brown
for breach of contract.
On March 1, Green and Brown orally agreed that II. Green is excused from paying the $5,000.
Brown would erect a boathouse on Green’s lot and
would dig a channel from the boathouse across (A) I only
Clark’s lot to a lake. Clark had already orally agreed (B) II only
with Green to permit the digging of the channel (C) Both I and II
across Clark’s lot. Brown agreed to begin work on (D) Neither I nor II
the boathouse on March 15, and to complete all the
work before June 1. The total price of $10,000 was
to be paid by Green in three installments: $2,500 on
March 15, $2,500 when the boathouse was completed, 253. Customer, aged 20, went into Store at
and $5,000 when Brown finished the digging of the approximately 6:45 p.m. to look at some suits
channel. that were on sale. The clerks were busy, and one
of them told Customer that he should wait on
250. Assume that Green tendered the $2,500 on himself. Customer selected three suits from a
March 15, and that Brown refused to accept it or rack and went into the dressing room to try them
to perform. In an action by Green against Brown on. Signs posted on the walls of Store stated that

58
closing time was 9:00 p.m.; however, because If Paul asserts a claim against Dock based
of a special awards banquet for employees, on defamation and it is proved that Dock’s
Store was closed at 7:00 p.m. on this day. The statement was true, such proof will be
employees, in a hurry to get to the banquet, did
not check the dressing rooms or turn off the lights (A) a defense by itself.
before leaving. When Customer emerged from (B) a defense only if Dock was not actuated by
the dressing room a few minutes after 7:00 p.m., malice.
he was alone and locked in. Customer tried the (C) a defense only if Dock reasonably believed
front door, but it was secured on the outside by it to be true.
a bar and padlock, so he went to the rear door. (D) no defense by itself.
Customer grabbed the doorknob and vigorously
shook the door. It did not open, but the activity
set off a mechanism that had been installed Questions 255–257 are based on the following fact
because of several recent thefts committed by situation.
persons who had hidden in the store until after
closing time. The mechanism sprayed a chemical Pemberton and three passengers, Able, Baker, and
mist in Customer’s face, causing him to become Charley, were injured when their car was struck by
temporarily blind. The mechanism also activated a truck owned by Mammoth Corporation and driven
an alarm carried by Store’s employee, Watchman, by Edwards. Helper, also a Mammoth employee, was
who was just coming to work. Watchman riding in the truck. The issues in Pemberton v.
unlocked the front door, ran into the store, Mammoth include the negligence of Edwards in
and grabbed Customer. Customer, who was still driving too fast and failing to wear glasses, and of
unable to see, struck out at this person and hit Pemberton in failing to yield the right of way.
a metal rack, injuring his hand. Watchman then
identified himself, and Customer did the same. 255. Pemberton’s counsel proffers evidence showing
After assuring himself that Customer was telling that shortly after the accident Mammoth put
the truth, Watchman allowed him to leave. a speed governor on the truck involved in the
accident. The judge should rule the proffered
If Customer is to prevail on a claim against Store evidence
based on battery from the use of the chemical
spray, Customer must establish that (A) admissible as an admission of a party.
(B) admissible as res gestae.
(A) he suffered severe bodily harm. (C) inadmissible for public policy reasons.
(B) the spray mist was an offensive or harmful (D) inadmissible, because it would lead to the
contact. drawing of an inference on an inference.
(C) he suffered severe emotional distress.
(D) his conduct was not a factual cause of the 256. Pemberton’s counsel seeks to introduce Helper’s
chemical’s being sprayed on him. written statement that Edwards, Mammoth’s
driver, left his glasses (required by his operator’s
254. Dock was the unsuccessful suitor of Mary, who license) at the truck stop when they left it five
recently announced her engagement to Paul. minutes before the accident. The judge should
Angered by her engagement, Dock sent Mary rule the statement admissible only if
the following letter: “I hope you know what you
are doing. The man you think you love wears (A) Pemberton first proves that Helper was an
women’s clothes when at home. A Friend.” agent of Mammoth and that the statement
concerned a matter within the scope of his
The receipt of this letter caused Mary great agency.
emotional distress. She hysterically telephoned (B) Pemberton produces independent evidence
Paul, read him the letter, and told him that she that Edwards was not wearing corrective
was breaking their engagement. The contents lenses at the time of the accident.
of the letter were not revealed to others. (C) Helper is shown to be beyond the process
Paul, who was a young attorney in the state of the court and unavailable to testify.
attorney’s office, suffered serious humiliation (D) the statement was under oath in affidavit
and emotional distress as a result of the broken form.
engagement.
257. Mammoth’s counsel seeks to have Sheriff testify
that while he was investigating the accident he

59
was told by Pemberton, “This was probably a warrant and using the landlord’s passkey, entered
our fault.” The judge should rule the proffered the apartment with the landlord’s consent. The lease
evidence to these premises gave the landlord a right of entry,
at any reasonable hour, for the purpose of making
(A) admissible as an admission of a party. repairs. The police officer found a large trunk in
(B) admissible, because it is a statement made the bedroom which seemed to be the source of the
to a police officer in the course of an odor. Upon breaking it open, he found the remains of
official investigation. Rosette, Defendant’s former mistress.
(C) inadmissible, because it is a mixed
conclusion of law and fact. 259. The landlord’s consent to the police officer’s
(D) inadmissible, because it is hearsay, not search of Defendant’s apartment is
within any exception.
(A) a waiver of Defendant’s Fourth Amendment
rights, because a landlord has implied
consent to enter a tenant’s apartment.
258. The city of Newtown adopted an ordinance (B) a waiver of Defendant’s Fourth
providing that street demonstrations involving Amendment rights, because the lease gave
more than 15 persons may not be held in the landlord express authority to enter the
commercial areas during “rush” hours. premises.
“Exceptions” may be made to the prohibition (C) not a waiver of Defendant’s Fourth
“upon 24-hour advance application to an Amendment rights, because the landlord
approval by the police department.” The lacked probable cause to believe that
ordinance also imposes sanctions on any person a crime was then in the process of
“who shall, without provocation, use to or of commission.
another, and in his presence, opprobrious words (D) not a waiver of Defendant’s Fourth
or abusive language tending to cause a breach of Amendment rights, because the landlord
the peace.” The ordinance has not yet had either had neither actual nor apparent authority to
judicial or administrative interpretation. Which permit the entry.
of the following is the strongest argument for the
unconstitutionality of both parts of the ordinance 260. If Defendant undertakes to challenge the search
on their face? of his apartment, he has

(A) No type of prior restraint may be imposed (A) standing, because the items seized in the
on speech in public places. search were incriminating in nature.
(B) Laws regulating, by their terms, expressive (B) standing, because he still has a sufficient
conduct or speech may not be overbroad or interest in the apartment even while in jail.
unduly vague. (C) no standing, because his landlord
(C) The determination as to whether public authorized the search.
gatherings may be lawfully held cannot be (D) no standing, because he was out of the
vested in the police. apartment when the search occurred and
(D) The right of association in public places had not paid his rent.
without interference is assured by the First
and Fourteenth Amendments.

Questions 261–262 are based on the following fact


Questions 259–260 are based on the following fact situation.
situation.
Seller and Buyer execute an agreement for the sale of
While Defendant was in jail on a procuring charge, real property on September 1, 1971. The jurisdiction in
his landlord called the police because rent had not which the property is located recognizes the principle
been paid and because he detected a disagreeable odor of equitable conversion and has no statute pertinent to
coming from Defendant’s apartment into the hallways. this problem.

The police officer who responded to the call knew 261. Assume for this question only that Seller dies
that Defendant was in jail. He recognized the stench before closing and his will leaves his personal
coming from Defendant’s apartment as that of property to Perry and his real property to Rose.
decomposing flesh and, without waiting to obtain

60
There being no breach of the agreement by July, August, and September 1971, without requiring
either party, which of the following is correct? a certificate. Sawtooth worked through October,
but no work was done from November 1, 1971, to
(A) Death, an eventuality for which the parties the end of February 1972, because of bad weather,
could have provided, terminates the and Farquart made no payments during that period.
agreement if they did not so provide. Sawtooth did not object. On March 1, 1972, Sawtooth
(B) Rose is entitled to the proceeds of the demanded payment of $2,000; but Farquart refused on
sale when it closes, because the doctrine the grounds that no construction work had been done
of equitable conversion does not apply to for four months and Builders had issued no certificate.
these circumstances. Sawtooth thereupon abandoned work and repudiated
(C) Perry is entitled to the proceeds of the sale the agreement.
when it closes.
(D) Title was rendered unmarketable by 263. Assuming that Sawtooth committed a total
Seller’s death. breach on March 1, 1972, what would be the
probable measure of Farquart’s damages in an
262. Assume for this question only that Buyer action against Sawtooth for breach of contract?
dies before closing, there being no breach of
the agreement by either party. Which of the (A) Restitution of the three monthly
following is appropriate in most jurisdictions? installments paid in August, September,
(A) Buyer’s heir may specifically enforce the and October.
agreement. (B) What it would cost to get the house
(B) Seller has the right to return the down completed by another contractor, minus
payment and cancel the contract. installments not yet paid to Sawtooth.
(C) Death terminates the agreement. (C) The difference between the market value
(D) Any title acquired would be unmarketable of the partly built house, as of the time of
by reason of Buyer’s death. Sawtooth’s breach, and the market value
of the house if completed according to
specifications.
(D) In addition to other legally allowable
Questions 263–265 are based on the following fact damages, an allowance for Farquart’s
situation. mental distress if the house cannot be
completed in time for Junior’s wedding on
Farquart had made a legally binding promise to June 10, 1972.
furnish his son Junior and the latter’s fiancée a house
on their wedding day, planned for June 10, 1972. 264. Assuming that Sawtooth committed a total
Pursuant to that promise, Farquart telephoned his old breach on March 1, 1972, and assuming further
contractor-friend Sawtooth on May 1, 1971, and made that he was aware when the agreement was
the following oral agreement­—each making full and made of the purpose for which Farquart wanted
accurate written notes thereof: the completed house, which of the following,
if true, would best support Farquart’s claim
Sawtooth was to cut 30 trees into fireplace for consequential damages on account of delay
logs from a specified portion of a certain one- beyond June 10, 1972, in getting the house
acre plot owned by Farquart, and Farquart was finished?
to pay therefor $20 per tree. Sawtooth agreed
further to build a house on the plot conforming (A) Junior and his bride, married on June 10,
to the specifications of Plan OP5 published 1972, had to pay storage charges on their
by Builders, Inc., for a construction price of wedding gifts and new furniture until the
$18,000. Farquart agreed to make payments house was completed.
of $2,000 on the first of every month for nine (B) Junior’s fiancée jilted Junior on June 10,
months beginning August 1, 1971, upon monthly 1972, and ran off with another man who
presentation of a certificate by Builders, Inc., had a new house.
that the specifications of Plan OP5 were being (C) Farquart was put to additional expense in
met. providing Junior and his bride, married on
June 10, 1972, with temporary housing.
Sawtooth delivered the cut logs to Farquart in July (D) On June 10, 1972, Farquart paid a $5,000
1971, when he also began building the house. Farquart judgment obtained against him in a suit
made three $2,000 payments for the work done in filed March 15, 1972, by an adjoining

61
landowner on account of Farquart’s (A) recover, because Construction Company
negligent excavation, including blasting, in left the open trench unprotected.
an attempt to finish the house himself after (B) recover, because construction companies
Sawtooth’s repudiation. are strictly liable for inherently dangerous
conditions.
265. What was the probable legal effect of the (C) not recover, because Tommy was a
following? trespasser.
(D) not recover, because Tommy’s death was
I. Sawtooth’s failure to object to Farquart’s a result of the collapse of the trench, an
making no payments on November 1, independent intervening cause.
December 1, January 1, and February 1.
II. Farquart’s making payments in August 267. Doctor, a licensed physician, resided in her own
through October without requiring a home. The street in front of the home had a
certificate from Builders. gradual slope. Doctor’s garage was on the street
level, with a driveway entrance from the street.
(A) Estoppel-type waiver as to both I and II.
(B) Waiver of delay in payment as to I and At two in the morning, Doctor received an
revocable waiver as to II. emergency call. She dressed and went to the
(C) Mutual rescission of the contract by I garage to get her car and found a car parked in
combined with II. front of her driveway. That car was occupied
(D) Discharge of Farquart’s duty to make the by Parker, who, while intoxicated, had driven
four payments as to I and estoppel-type to that place and now was in a drunken stupor
waiver as to II. in the front seat. Unable to rouse Parker,
Doctor pushed him into the passenger’s side of
the front seat and got in on the driver’s side.
Doctor released the brake and coasted the car
266. Construction Company contracted to build down the street, planning to pull into a parking
a laundry for Wash Company on the latter’s space that was open. When Doctor attempted
vacant lot in a residential area. As a part of its to stop the car, the brakes failed to work, and
work, Construction Company dug a trench from the car crashed into the wall of Owner’s home,
the partially completed laundry to the edge damaging Owner’s home and Parker’s car and
of a public sidewalk; water pipes were to be injuring Doctor and Parker.
installed in the trench. Because of the contour of
the land, the trench was dug to a depth ranging Subsequent examination of the car disclosed
from seven to nine feet. Construction Company that the brake linings were badly worn. A
did not place any barriers around the trench and state statute prohibits the operation of a motor
permitted it to lie open for almost a week while vehicle unless the brakes are capable of
Construction Company waited for delivery of the stopping the vehicle within specified distances
water pipes. This was known to Wash Company, at specified speeds. The brakes on Parker’s car
but it raised no objection. were incapable of stopping the vehicle within
the limits required by the statute. Another
During the time the trench was open, a series state statute makes it a criminal offense to be
of heavy rains fell, causing five feet of surface intoxicated while driving a motor vehicle.
water to gather in the bottom of the trench.
While this condition existed, five-year-old If Parker asserts a claim against Doctor for his
Tommy, who was playing on the vacant lot with injuries, Parker will probably
friends, stumbled and fell into the trench. Robert,
an adult passerby, saw this and immediately (A) recover, because Doctor was negligent as a
lowered himself into the trench to rescue matter of law.
Tommy. However, his doing so caused the rain- (B) recover, because Doctor had no right to
soaked walls of the trench to collapse, killing move the car.
both him and Tommy. (C) not recover, because his brakes were
defective.
In a claim for wrongful death by Tommy’s (D) not recover, because he was in a drunken
administrator against Construction Company, the stupor when injured.
most likely result is that the administrator will

62
268. In a contract suit between Terrell and Ward, (A) not guilty, because mere presence, coupled
Ward testifies that he recalls having his first with silent approval and intent, is not
conversation with Terrell on January 3. When sufficient.
asked how he remembers the date, he answers, (B) not guilty, because he did not tell Bill
“In the conversation, Terrell referred to a ahead of time that he hoped Bill would
story in that day’s newspaper announcing my murder Vic.
daughter’s engagement.” Terrell’s counsel (C) guilty, because he had a duty to stop the
moves to strike the reference to the newspaper killing and made no attempt to do so.
story. The judge should (D) guilty, because he was present and
approved of what occurred.
(A) grant the motion on the ground that the
best evidence rule requires production of
the newspaper itself.
(B) grant the motion, because the reference to 271. Leonard was the high priest of a small cult of
the newspaper story does not fit within any Satan worshippers living in New Arcadia. As
established exception to the hearsay rule. a part of the practice of their religious beliefs,
(C) deny the motion on the ground that the a cat was required to be sacrificed to the glory
court may take judicial notice of local of Satan after a live dissection of the animal in
newspapers and their contents. which it endured frightful pain. In the course of
(D) deny the motion on the ground that a such a religious sacrifice, Leonard was arrested
witness may refer to collateral documents on the complaint of the local humane society
without providing the documents and charged under a statute punishing cruelty
themselves. to animals. On appeal, a conviction of Leonard
probably will be

Questions 269–270 are based on the following fact (A) sustained on the grounds that belief
situation. in or worship of Satan does not enjoy
constitutional protection.
Bill and Chuck hated Vic and agreed to start a fight (B) sustained on the grounds that sincere
with Vic and, if the opportunity arose, to kill him. religious belief is not an adequate defense
on these facts.
Bill and Chuck met Vic in the street outside a bar and (C) overturned on the grounds that the
began to push him around. Ray, Sam, and Tom, who constitutionally guaranteed freedom of
also hated Vic, stopped to watch. Ray threw Bill a religion and its expression was violated.
knife. Sam told Bill, “Kill him.” Tom, who made no (D) overturned on the grounds that the beliefs
move and said nothing, hoped that Bill would kill Vic of the cult members in the need for the
with the knife. Chuck held Vic while Bill stabbed and sacrifice might be reasonable and that
killed him. their act was religious.

269. On a charge of murdering Vic, Sam is 272. Drew is charged with the murder of Pitt. The
prosecutor introduced testimony of a police
(A) not guilty, because his words did not create officer that Pitt told a priest, administering
a “clear and present danger” not already the last rites, “I was stabbed by Drew. Since I
existing. am dying, tell him I forgive him.” Thereafter,
(B) not guilty, because mere presence and oral Drew’s attorney offers the testimony of Wall
encouragement, whether or not he had the that the day before, when Pitt believed he would
requisite intent, do not make him guilty as live, he stated that he had been stabbed by Jack,
an accomplice. an old enemy. The testimony of Wall is
(C) guilty, because, with the intent to have Bill
kill Vic, he shouted encouragement to Bill. (A) admissible under an exception to the
(D) guilty, because he aided and abetted the hearsay rule.
murder through his mere presence plus his (B) admissible to impeach the dead declarant.
intent to see Vic killed. (C) inadmissible, because it goes to the
ultimate issue in the case.
270. On a charge of murdering Vic, Tom is (D) inadmissible, because it is irrelevant to any
substantive issue in the case.

63
273. An appropriations act passed by Congress 3. Each lot owner must have an unambiguous
over the President’s veto directs that one right to transfer the lot to a purchaser with
billion dollars “shall be spent” by the federal all original benefits.
government for the development of a new 4. Each lot owner must be obligated to pay
military weapons system, which is available annual dues to a pro rata share (based on
only from the Arms Corporation. On the order the number of lots) of the club’s annual
of the President, the Secretary of Defense operating deficit (whether or not such
refuses to authorize a contract for purchase of owner desires to make use of club and
the weapons system. The Arms Corporation sues course facilities).
the Secretary of Defense alleging an unlawful
withholding of these federal funds. 274. In the context of all aspects of the scheme,
which of the following will offer the best chance
The strongest constitutional argument for the of implementing the requirement that each lot
Arms Corporation is that owner pay annual dues to support the club and
golf course?
(A) passage of an appropriation over a veto
makes the spending mandatory. (A) Covenant.
(B) Congress’s power to appropriate funds (B) Easement.
includes the power to require that the funds (C) Mortgage.
be spent as directed. (D) Personal contractual obligation by each
(C) the President’s independent constitutional purchaser.
powers do not specifically refer to
spending. 275. Of the following, the greatest difficulty that will
(D) the President’s power to withhold such be encountered in establishing the scheme is that
funds is limited to cases where foreign
affairs are directly involved. (A) any judicial recognition will be construed
as state action which, under current
doctrines, raises a substantial question as
Questions 274–275 are based on the following fact to whether such action would be in conflict
situation. with the Fourteenth Amendment.
(B) the scheme, if effective, renders title
Ohner holds title in fee simple to a tract of 1,500 unmarketable.
acres. He wishes to develop the entire tract as a golf (C) one or more of the essential aspects
course, country club, and residential subdivision. outlined by Ohner will result in a restraint
He contemplates forming a corporation to own and on alienation.
operate the golf course and country club; the stock (D) there is a judicial reluctance to recognize
in the corporation will be distributed to the owners an affirmative burden to pay money in
of lots in the residential portions of the subdivision, installments and over an indefinite period
but no obligation to issue the stock is to ripen until as a burden which can be affixed to bind
all the residential lots are sold. The price of the lots future owners of land.
is intended to return enough money to compensate
Ohner for the raw land, development costs (including
the building of the golf course and the country club
facilities), and developer’s profit, if all of the lots 276. Rogers gave Mitchell a power of attorney
are sold. Ohner’s market analyses indicate that he containing the following provision:
must create a scheme of development that will offer
prospective purchasers (and their lawyers) a very high My attorney, Mitchell, is specifically
order of assurance that several aspects will be clearly authorized to sell and convey any part or
established: all of my real property.

1. Aside from the country club and golf Mitchell conveyed part of Rogers’s land to
course, there will be no land use other than Stone by deed in the customary form containing
for residential use and occupancy in the covenants of title. Stone sues Rogers for breach
1,500 acres. of a covenant. The outcome of Stone’s suit will
2. The residents of the subdivision will have be governed by whether
unambiguous rights of access to the club
and golf course facilities.

64
(A) deeds without covenants are effective to (C) not recover, because Peter was a licensee.
convey realty. (D) not recover, because Peter assumed the
(B) the jurisdiction views the covenants as risk.
personal or running with the land.
(C) Stone is a bona fide purchaser.
(D) the power to “sell and convey” is Questions 278–279 are based on the following fact
construed to include the power to execute situation.
the usual form of deed used to convey
realty. In a trial between Jones and Smith, an issue arose
about Smith’s ownership of a horse that had caused
277. Auto Company, a corporation, was a small damage to Jones’s crops.
dealer in big new cars and operated a service
department. Peter wanted to ask Mike, the 278. Jones offered to testify that he looked up Smith’s
service manager, whether Auto Company would telephone number in the directory, that he called
check the muffler on his small foreign car. Peter that number, and that a voice answered, “This is
parked on the street near the service department Smith speaking.” At this, Jones asked, “Was that
with the intention of entering that part of the your horse that tramped across my cornfield this
building by walking through one of the three afternoon?” The voice replied, “Yes.” The judge
large entrances designed for use by automobiles. should rule the testimony
There was no street entrance to the service
department for individuals, and customers as (A) admissible, because the answering
well as company employees often used the speaker’s identification of himself,
automobile entrances. together with the usual accuracy of the
telephone directory and transmission
As Peter reached the building, he glanced behind system, furnishes sufficient authentication.
him to be sure no vehicle was approaching that (B) admissible, because judicial notice may
entrance. Seeing none, he walked through the be taken of the accuracy of telephone
entrance, but immediately he was struck on the directories.
back of the head and neck by the large overhead (C) inadmissible unless Jones can further
door which was descending. The blow knocked testify that he was familiar with Smith’s
Peter unconscious and caused permanent damage. voice and that it was in fact Smith to
whom he spoke.
Peter did not know how the door was raised (D) inadmissible unless Smith has first been
and lowered; however, the overhead door was asked whether or not the conversation took
operated by the use of either of two switches place and has been given the opportunity
in the building. One switch was located in the to admit, deny, or explain.
office of the service manager and the other
was located near the door in the service work 279. Jones seeks to introduce into evidence a
area for the convenience of the mechanics. On photograph of his cornfield in order to depict the
this occasion, no one was in the service work nature and extent of the damage done. The judge
area except three Auto Company mechanics. should rule the photograph
Mike, who had been in his office, and the three
mechanics denied having touched a switch (A) admissible if Jones testifies that it fairly
that would have lowered the door. Subsequent and accurately portrays the condition of the
investigation showed, however, that the switches cornfield after the damage was done.
were working properly and that all of the (B) admissible if Jones testifies that the
mechanisms for moving the door were in good photograph was taken within a week after
working order. the alleged occurrence.
(C) inadmissible if Jones fails to call the
If Peter asserts a claim based on negligence photographer to testify concerning the
against Auto Company, Peter probably will circumstances under which the photograph
was taken.
(A) recover, because Auto Company is strictly (D) inadmissible if it is possible to describe the
liable under the circumstance. damage to the cornfield through direct oral
(B) recover, because an employee of Auto testimony.
Company was negligent.

65
Questions 280 and 281 each describe an offense. 283. Householder hired Contractor to remodel
Select from the choices (A–D) the most serious Householder’s kitchen. She had learned of
offense of which the defendant could be properly Contractor through a classified advertisement
convicted. he placed in the local newspaper. During the
telephone conversation in which she hired him,
(A) Involuntary manslaughter Contractor stated that he was experienced and
(B) Voluntary manslaughter qualified to do all necessary work. Because
(C) Murder of Contractor’s low charge for his work,
(D) None of the above he and Householder agreed in writing that
on acceptance of his job by Householder,
280. Defendant, an avid fan of his hometown football Contractor would have no further liability to
team, shot at the leg of a star player for a rival her or to anyone else for any defects in materials
team, intending to injure his leg enough to or workmanship, and that she would bear all
hospitalize him for a few weeks, but not to kill such costs.
him. The victim died of loss of blood. Householder purchased a dishwasher
manufactured by Elex Company from Dealer,
281. Defendant, a worker in a metalworking shop, who was in the retail electrical appliance
had long been teasing Vincent, a young business. The dishwasher was sold by Dealer
colleague, by calling him insulting names and with only the manufacturer’s warranty and with
ridiculing him. One day Vincent responded no warranty by Dealer; Elex Company restricted
to the teasing by picking up a metal bar and its warranty to ninety days on parts and labor.
attacking Defendant. Defendant could have Contractor installed the dishwasher.
escaped from the shop. He parried the blow
with his left arm, and with his right hand struck Two months after Householder accepted the
Vincent a blow on his jaw from which the young entire job, she was conversing in her home with
man died. Accountant, an acquaintance who had agreed
to prepare her income tax return gratuitously.
As they talked, they noticed that the dishwasher
was operating strangely, repeatedly stopping
282. Patty sued Mart Department Store for personal and starting. At Householder’s request,
injuries, alleging that while shopping she was Accountant gave it a cursory examination
knocked to the floor by a merchandise cart being and, while inspecting it, received a violent
pushed by Handy, a stock clerk, and that as a electrical shock which did him extensive harm.
consequence her back was injured. The dishwasher had an internal wiring defect
which allowed electrical current to be carried
Handy testified that Patty fell near the cart but into the framework and caused the machine
was not struck by it. Thirty minutes after Patty’s to malfunction. The machine had not been
fall, Handy, in accordance with regular practice adequately grounded by Contractor during
at Mart, filled out a printed form, “Employee’s installation; if it had been, the current would
Report of Accident—Mart Department Store,” in have been led harmlessly away. The machine
which he stated that Patty had been leaning over carried instructions for correct grounding, which
to spank her young child and in so doing had Contractor had not followed.
fallen near his cart. Counsel for Mart offers in
evidence the report, which was given to him by If Accountant asserts a claim based on strict
Handy’s supervisor. liability against Elex Company for damages, the
probable result is that Accountant will
The judge should rule the report offered by Mart
(A) recover, because the dishwasher was
(A) admissible as res gestae. defectively made.
(B) admissible as a business record. (B) recover, because Elex Company is
(C) inadmissible, because it is hearsay, not vicariously liable for the improper
within any exception. installation.
(D) inadmissible, because Handy is available (C) not recover, because he assumed the risk
as a witness. by inspecting the machine.
(D) not recover, because he was not the
purchaser.

66
Questions 284–285 are based on the following fact In appropriate action to construe the will, the
situation. court will determine that the remainder to
children is
The State of Missoula has enacted a new election
code designed to increase voter responsibility in (A) indefeasibly vested.
the exercise of the franchise and to enlarge citizen (B) contingent.
participation in the electoral process. None of its (C) vested subject to partial defeasance.
provisions conflicts with federal statutes. (D) vested subject to complete defeasance.

284. Which of the following is the strongest reason


for finding unconstitutional a requirement in the Questions 287–288 are based on the following fact
Missoula election code that each voter must be situation.
literate in English?
(A) The requirement violates Article I, Section Alpha and Beta made a written contract pursuant
2 of the Constitution, which provides that to which Alpha promised to convey a specified
representatives to Congress be chosen “by apartment house to Beta in return for Beta’s promise
the People of the several States.” (1) to convey a 100-acre farm to Alpha and (2) to pay
(B) The requirement violates Article I, Alpha $1,000 in cash six months after the exchange
Section 4 of the Constitution, which gives of the apartment house and the farm. The contract
Congress the power to “make or alter” contained the following provision: “It is understood
state regulations providing for the “Times” and agreed that Beta’s obligation to pay the $1,000 six
and “Manner” of holding elections for months after the exchange of the apartment house and
senators and representatives. the farm shall be voided if Alpha has not, within three
(C) the requirement violates the due process months after the aforesaid exchange, removed the
clause of the Fourteenth Amendment. existing shed in the parking area in the rear of the said
(D) The requirement violates the equal apartment house.”
protection of the laws clause of the
Fourteenth Amendment. 287. Which of the following statements concerning
the order of performances is LEAST accurate?
285. The Missoula election code provides that in a
special-purpose election for directors of a state (A) Alpha’s tendering of good title to the
watershed improvement district, the franchise is apartment house is a condition precedent
limited to landowners within the district, because to Beta’s duty to convey good title to the
they are the only ones directly affected by the farm.
outcome. Each vote is weighted according to the (B) Beta’s tendering of good title to the farm
proportion of the holding of that individual in is a condition precedent to Alpha’s duty to
relation to the total affected property. The best convey good title to the apartment house.
argument in support of the statute and against (C) Beta’s tendering of good title to the farm is
the application of the “one man, one vote” a condition subsequent to Alpha’s duty to
principle in this situation is that the principle convey good title to the apartment house.
(D) Alpha’s tendering of good title to the
(A) applies only to elections of individuals to apartment house and Beta’s tendering
statewide public office. of good title to the farm are concurrent
(B) does not apply where property rights are conditions.
involved.
(C) does not apply, because the actions of such 288. Alpha’s removal of the shed from the parking
a district principally affect landowners. area of the apartment house is
(D) does not apply, because of rights reserved
to the states by the Tenth Amendment. (A) a condition subsequent in form but
precedent in substance to Beta’s duty to
pay the $1,000.
(B) a condition precedent in form but
286. Testator devised his farm “to my son, Selden, for subsequent in substance to Beta’s duty to
life, then to Selden’s children and their heirs and pay the $1,000.
assigns.” Selden, a widower, had two unmarried (C) a condition subsequent to Beta’s duty to
adult children. pay the $1,000.

67
(D) not a condition, either precedent or In an action by Brown against Green on the
subsequent, to Beta’s duty to pay the promissory note, which of the following, if true,
$1,000. would afford Green the best defense?

(A) Although Brown honestly believed that


$6,000 was owed by Green, Green knew
Questions 289–290 are based on the following fact that it was not owed.
situation. (B) Although Brown knew that the debt was
not owed, Green honestly was in doubt as
Johnson took a diamond ring to a pawnshop and to whether it was owed.
borrowed $20 on it. It was agreed that the loan was (C) The original claim was based on an oral
to be repaid within 60 days and if it was not, the agreement, which the Statute of Frauds
pawnshop owner, Defendant, could sell the ring. A required to be in writing.
week before expiration of the 60 days, Defendant had (D) The original claim was an action on
an opportunity to sell the ring to a customer for $125. a contract, which was barred by the
He did so, thinking it unlikely that Johnson would applicable statute of limitations.
repay the loan and that if he did, Defendant would be
able to handle him somehow, even by paying him for
the ring if necessary. Two days later, Johnson came Questions 292–293 are based on the following fact
in with the money to reclaim his ring. Defendant situation.
told him that it had been stolen when his shop was
burglarized one night and that therefore he was not Mrs. Ritter, a widow, recently purchased a new
responsible for its loss. uncrated electric range for her kitchen from Local
Retailer. The range has a wide oven with a large
Larceny, embezzlement, and false pretenses are oven door. The crate in which Stove Company, the
separate crimes in the jurisdiction. manufacturer, shipped the range to Local Retailer
carried a label with a warning that the stove would tip
289. It is most likely that Defendant has committed over with a weight of 25 pounds or more on the oven
which of the following crimes? door. Mrs. Ritter has one child—Brenda, age three.
Recently, at about 5:30 p.m., Brenda was playing
(A) Larceny. on the floor of the kitchen while Mrs. Ritter was
(B) Embezzlement. heating water in a pan on the stove. The telephone
(C) Larceny by trick. rang and Mrs. Ritter went into the living room to
(D) Obtaining by false pretenses. answer it. While she was gone, Brenda decided to
find out what was cooking. She opened the oven
290. Suppose in the case above, instead of denying door and climbed on it to see what was in the pan.
liability, Defendant told Johnson the truth—that Brenda’s weight (25 pounds) on the door caused the
he sold the ring because he thought Johnson stove to tip over forward. Brenda fell to the floor and
would not reclaim it—and offered to give the hot water spilled over her, burning her severely.
Johnson $125. Johnson demanded his ring. Brenda screamed. Mrs. Ritter ran to the kitchen and
Defendant said, “Look, buddy, that’s what I got immediately gave her first aid treatment for burns.
for it, and it’s more than it’s worth.” Johnson Brenda thereafter received medical treatment.
reluctantly took the money. Defendant could
most appropriately be found guilty of Brenda’s burns were painful. They have now healed
and do not bother her, but she has ugly scars on her
(A) larceny. legs and back. Brenda’s claim is asserted on her behalf
(B) embezzlement. by the proper party.
(C) false pretenses.
(D) none of the above. 292. If Brenda asserts a claim based on strict liability
against Stove Company, she must establish that

(A) the defendant negligently designed the
291. Brown contended that Green owed him $6,000. stove.
Green denied that he owed Brown anything. (B) stoves made by other manufacturers do not
Tired of the dispute, Green eventually signed a turn over with a 25-pound weight on the
promissory note by which he promised to pay oven door.
Brown $5,000 in settlement of their dispute.

68
(C) the defendant failed to warn the Ritters that (C) The statute will help assure a continuously
the stove would turn over easily. available and stable workforce for the
(D) the stove was defective and unreasonably execution of public contracts.
dangerous to her. (D) The statute will help assure that only the
most qualified individuals work on public
293. If Brenda asserts a claim based on strict liability contracts.
against Local Retailer, she must establish that
296. Suppose the state supreme court declares the
(A) Local Retailer did not inform Mrs. Ritter statute to be unconstitutional on the grounds that
of the warning on the crate. it violates the privileges and immunities clause
(B) the stove was substantially in the same of the Fourteenth Amendment to the federal
condition at the time it tipped over as when Constitution and the equal protection clause
it was purchased from Local Retailer. of the state constitution. If the state seeks review
(C) Local Retailer made some change in the in the United States Supreme Court, which of
stove design or improperly assembled it so the following statements is most accurate?
that it tipped over more easily.
(D) Local Retailer knew or should have known (A) The United States Supreme Court may
that the stove was dangerous because of properly review that decision by certiorari
the ease with which it tipped over. only.
(B) The United States Supreme Court may
properly review the decision by appeal
only.
Questions 294–296 are based on the following fact (C) The United States Supreme Court may
situation. properly review that decision by appeal or
certiorari.
The State of Yuma provides by statute, “No person (D) The United States Supreme Court may not
may be awarded any state construction contract properly review that decision.
without agreeing to employ only citizens of the
state and of the United States in performance of the
contract.” 297. Allen and Barker are equal tenants in common
of a strip of land 10 feet wide and 100 feet
294. In evaluating the constitutionality of this state deep which lies between the lots on which their
statute under the supremacy clause, which of the respective homes are situated. Both Allen and
following would be most directly relevant? Barker need the use of the 10-foot strip as a
driveway, and each fears that a new neighbor
(A) The general unemployment rate in the might seek partition and leave him with an
nation. unusable five-foot strip. The best advice about
(B) The treaties and immigration laws of the how to solve their problem is
United States.
(C) The need of the state for this particular (A) a covenant against partition.
statute. (B) an indenture granting cross easements in
(D) The number of aliens currently residing in the undivided half interest of each.
Yuma. (C) partition into two separate five-foot-wide
strips and an indenture granting cross
295. If the Yuma statute is attacked as violating easements.
the commerce clause, which of the following (D) a trust to hold the strip in perpetuity.
defenses is the WEAKEST?
298. Mrs. Dennis’s 12-year-old daughter, Gala, had
(A) The statute will help protect the workers some difficulty getting along with other children
of the State of Yuma from competition by in the neighborhood, especially with the younger
foreign workers. ones. Thinking the experience would be good
(B) The statute will help assure that for her, Mrs. Dennis recommended Gala to Mr.
workers with jobs directly affecting Parrent as a babysitter for his five-year-old boy,
the performance of public contracts are Robby, but did not mention Gala’s difficulties or
dedicated to their jobs. her lack of prior experience as a babysitter. The
Dennises and the Parrents were longstanding
social acquaintances. On the evening Gala was

69
to sit, the Parrents told Gala that she should treat (A) “When” and “Wouldn’t that be nice”
Robby firmly, but that it would be preferable not implied a promise to type the manuscript.
to spank him since he did not take kindly to such (B) James relied on Mary Digit’s statement by
treatment. They did not tell Gala that they had bringing the manuscript to X-L.
experienced trouble retaining babysitters because (C) X-L had done good work for James in the
of Robby’s temper tantrums. past.
(D) James had forgone the services of another
Later in the evening, when Robby became angry secretarial service.
upon being told to go to his room for being
naughty, Gala spanked him, but only moderately 300. In 1967 Owen held Blackacre, a tract of land,
hard. Robby then threw a hardback book at in fee simple absolute. In that year he executed
Gala, hitting her in the eye. As Gala tried to and delivered to Price a quitclaim deed which
catch Robby to take him to his room, Robby purported to release and quitclaim to Price
fled around the house and out the back door, all of the right, title, and interest of Owen in
knocking over and breaking an expensive lamp. Blackacre. Price accepted the quitclaim and
placed the deed in his safe deposit box.
The backyard was completely dark. Gala heard
Robby screaming and banging at the back door, Owen was indebted to Crider in the amount of
which had closed and locked automatically, $35,000. In September 1971, Owen executed and
but she did nothing. After twenty minutes had delivered to Crider a warranty deed, purporting
passed, she heard a banging and crying at the to convey the fee simple to Blackacre, in
front door, but still she did nothing. Then the exchange for a full release of the debt he owed
noise stopped. In a few minutes Gala went to Crider. Crider immediately recorded his deed.
outside and found Robby lying on the steps
unconscious and injured. In December 1971, Price caused his quitclaim
deed to Blackacre to be recorded and notified
If a claim is asserted on behalf of Robby against Crider that he (Price) claimed title.
Mrs. Dennis for damages based on Gala’s
conduct, Mrs. Dennis will probably be liable, Assume that there is no evidence of occupancy
because of Blackacre and assume, further, that the
jurisdiction where Blackacre is situated has
(A) parents are vicariously liable for the a recording statute which requires good faith
intentional torts of their children. and value as elements of the junior claimant’s
(B) she has a nondelegable duty to control the priority. Which of the following is the best
actions of her child. comment concerning the conflicting claims of
(C) respondeat superior applies. Price and Crider?
(D) she was negligent.
(A) Price cannot succeed, because the
299. Professor James said to Mary Digit, president quitclaim through which he claims
of the X-L Secretarial Service, “Since you folks prevents him from being bona fide (in
have done good typing work for me in the past, I good faith).
promise to bring you the manuscript for my new (B) The outcome will turn on the view taken
book.” as to whether Crider paid value within
the meaning of the statute requiring this
“When?” asked Mary Digit. element.
(C) The outcome will turn on whether
“First chapter next Monday,” replied James. Price paid value (a fact not given in the
statement).
“Wouldn’t that be nice,” said Mary Digit. (D) Price’s failure to record until December
1971 estops him from asserting title
The following Monday, James, forgoing the against Crider.
services of another secretarial service, brought
the first chapter to the X-L office, but Mary 301. Hank owned a secondhand goods store. He
Digit refused to take it, saying that they were all often placed merchandise on the sidewalk,
booked up for three weeks. sometimes for short intervals, sometimes from 7
a.m. until 6 p.m. Pedestrians from time to time
Which of the following facts or inferences would stopped and gathered to look at the merchandise.
be most helpful in an action by James against X-L? Fred had moved into an apartment which was

70
situated immediately above Hank’s store; a (2) All that part of my farm, being a square
street-level stairway entrance was located about with 200-foot sides, the southeast corner of
20 feet to the east. On several occasions, Fred which is in the north line of my neighbor,
had complained to Hank about the situation Julia Brown.
because not only were his view and peace of
mind affected, but his travel on the sidewalk The deed contained covenants of general warranty,
was made more difficult. Fred owned and quiet enjoyment, and right to convey.
managed a restaurant two blocks to the west Pat handed the deed to Marian, who immediately
of his apartment and made frequent trips back returned it to her father for safekeeping. Her father
and forth. There was a back entrance to his kept it in his safe deposit box. The deed was not
apartment through a parking lot; this entrance recorded.
was about 200 feet farther in walking distance
from his restaurant. Once Fred complained to The property at 44 Main Street covered 7/8 of an
the police, whereupon Hank was arrested under acre of land, had a dwelling and a garage situated
a local ordinance which prohibited the placing thereon, and was subject to a right of way, described
of goods or merchandise on public sidewalks in prior deeds, in favor of Jack, a neighbor. Pat owned
and imposed, as its sole sanction, a fine for its no other land on Main Street. Jack had not used the
violation. right of way for 10 years, and it was not visible on
inspection of the property.
One day, the sidewalk in front of Hank’s store
was unusually cluttered because he was cleaning 302. The description of 44 Main Street was
and mopping the floor of his shop. Fred and his
15-year-old son, Steve, saw a bus they wished (A) sufficient, because the discrepancy in area
to take, and they raced down the stairs and onto
the cluttered sidewalk in front of Hank’s store, is not fatal.
Fred in the lead. While dodging merchandise (B) not sufficient, because it contained no
and people, Fred fell. Steve tripped over him and metes and bounds.
suffered a broken arm. Fred also suffered broken (C) not sufficient, because the acreage given
bones and was unable to attend to his duties for was not correct.
six weeks. (D) not sufficient, because a deed purporting to
convey more than a grantor owns is void
If, prior to the day of his personal injuries, Fred ab initio.
had asserted a claim based on public nuisance
for injunctive relief against Hank for his 303. The description of part of Pat’s farm
obstruction of the sidewalk in violation of the
ordinance, the defense on which Hank would (A) is sufficient if consideration has been paid.
have most likely prevailed is that (B) is sufficient because no ambiguity therein
appears on the face of the deed.
(A) Fred consented to the obstruction by (C) could be enforced if the deed contained a
continuing to rent his apartment. covenant of seisin.
(B) the violation of the ordinance was not (D) is insufficient because of vagueness.
unreasonable.
(C) remedy of abatement by self-help was
adequate.
(D) there was no claim for special damage. Questions 304–306 are based on the following fact
situation.
Questions 302–303 are based on the following fact Dann, who was charged with the crime of assaulting
situation. Smith, admitted striking Smith but claimed to have
acted in self-defense when he was attacked by Smith,
By way of a gift, Pat executed a deed naming his who was drunk and belligerent after a football game.
daughter, Marian, as grantee. The deed contained
descriptions as follows: 304. Dann offered the testimony of Employer, who
would say that he had known and employed
(1) All of my land and dwelling known as Dann for 12 years and knew Dann’s reputation
44 Main Street, Midtown, United States, among the people with whom he lived and
being one acre. worked to be that of a peaceful, law-abiding,

71
nonviolent person. The trial judge should rule as conditions for state receipt of federal funds:
this testimony (1) Whenever textbooks are provided to students
without charge, they must include no religious
(A) admissible, because it is relevant to instruction and must be made available on the same
show the improbability of Dann’s having terms to students in all public and private schools
committed an unprovoked assault. accredited by the state educational authority. (2) Salary
(B) admissible, because it is relevant to a supplements can be paid to teachers in public and
determination of the extent of punishment private schools, up to 10 percent of existing salary
if Dann is convicted. schedules, where present compensation is less than
(C) inadmissible, because whether Dann is the average salary for persons of comparable training
normally a person of good character is and experience, provided that no such supplement is
irrelevant to the specific charge. paid to any teacher who instructs in religious subjects.
(D) inadmissible, because it is irrelevant (3) Construction grants can be made toward the cost
without a showing that Employer was one of physical plant at private colleges and universities,
of the persons among whom Dann lived provided that no part of the grant is used for buildings
and worked. in which instruction in religious subject matters is
offered.
305. On cross-examination of Employer, the state’s
attorney asked Employer if he had heard that 307. Federal taxpayer Allen challenges the provision
Dann often engaged in fights and brawls. The that allows the distribution of free textbooks
trial judge should rule the question to students in a private school where religious
instruction is included in the curriculum. On the
(A) not objectionable, because evidence of question of the adequacy of Allen’s standing to
Dann’s previous fights and brawls may be raise the constitutional question, the most likely
used to prove his guilt. result is that standing will be
(B) not objectionable, because it tests
Employer’s knowledge of Dann’s (A) sustained, because any congressional
reputation. spending authorization can be challenged
(C) objectionable, because it seeks to put into by any taxpayer.
evidence separate, unrelated offenses. (B) sustained, because the challenge to the
(D) objectionable, because no specific times or exercise of congressional spending power
incidents are specified and inquired about. is based on a claimed violation of specific
constitutional limitations on the exercise of
306. Dann’s friend Frank was called to testify that such power.
Smith had a reputation among the people with (C) denied, because there is insufficient nexus
whom he lived and worked for law-breaking and between the taxpayer and the challenged
frequently engaging in brawls. The trial judge expenditures.
should rule the testimony (D) denied, because, in the case of private
schools, no state action is involved.
(A) admissible to support Dann’s theory of
self-defense, touching on whether Dann or 308. Federal taxpayer Bates challenges the salary
Smith was the aggressor. supplements for teachers in private schools
(B) admissible if Frank testifies further as to where religious instruction is included in the
specific acts of misconduct on Smith’s part curriculum. On the substantive constitutional
of which Frank has personal knowledge. issue, the most likely result is that the salary
(C) inadmissible on the question of Dann’s supplements will be
guilt because Dann, not Smith, is on trial.
(D) inadmissible, because Frank failed to lay a (A) sustained, because the statute provides
proper foundation. that no supplements will be made to
teachers who are engaged in any religious
instruction.
(B) sustained, because to distinguish between
Questions 307–309 are based on the following fact private and public school teachers would
situation. violate the religious freedom clause of the
First Amendment.
As part of a comprehensive federal aid-to-education (C) held unconstitutional, because some
program, Congress included the following provisions religions would benefit disproportionately.

72
(D) held unconstitutional, because the policing The faculty regrets that our offer regarding
of the restriction would amount to an the National Obscenity Law Competition
excessive entanglement with religion. must be withdrawn.

309. Federal taxpayer Bates also challenges the Student’s paper was submitted through the Dean’s
construction grants to church-operated private office on April 15. On May 1, it was announced that
colleges and universities. The most likely result Student had won the National Obscenity Law
is that the construction grants will be Competition and the prize of $1,000. The law faculty
refused to pay anything.
(A) sustained, because aid to one aspect of
an institution of higher education not 310. Assuming that the faculty’s notice of November
shown to be pervasively sectarian does 1 was posted on a bulletin board or other
not necessarily free it to spend its other conspicuous place commonly viewed by
resources for religious purposes. all persons in the law school, such notice
(B) sustained, because bricks and mortar constituted a
do not aid religion in a way forbidden
by the establishment clause of the First (A) preliminary invitation to deal, analogous to
Amendment. newspaper advertisements for the sale of
(C) held unconstitutional, because any goods by merchants.
financial aid to a church-operated school (B) contractual offer, creating a power of
strengthens the religious purposes of the acceptance.
institution. (C) preliminary invitation, because no offeree
(D) held unconstitutional, because the was named therein.
grants involve or cause an excessive (D) promise to make a conditional, future gift
entanglement with religion. of money.

311. As to Student, was the offer effectively revoked?



(A) Yes, by the faculty’s second notice.
Questions 310–313 are based on the following fact (B) No, because it became irrevocable after a
situation. reasonable time had elapsed.
(C) No, because of Student’s reliance, prior to
On November 1, the following notice was posted in a April 1, on the offer.
privately operated law school: (D) No, unless Student became aware of
the April 1 posting and removal before
The faculty, seeking to encourage legal submitting the paper.
research, offers to any student at this
school who wins the current National 312. The offer proposed a
Obscenity Law Competition the additional
prize of $500. All competing papers must (A) unilateral contract only.
be submitted to the Dean’s office before (B) bilateral contract only.
May 1. (C) unilateral contract or bilateral contract at
the offeree’s option.
(The National Competition was conducted by an (D) unilateral contract which ripened into a
outside agency unconnected with any law school.) bilateral contract, binding on both parties,
as soon as Student intensified his effort in
Student read this notice on November 2, and response to the offer.
thereupon intensified his effort to make his paper on
obscenity law, which he had started in October, a 313. The promise of the faculty on November 1 was
winner. Student also left on a counter in the Dean’s
office a signed note saying, “I accept the faculty’s (A) enforceable on principles of promissory
$500 Obscenity Competition offer.” This note was estoppel.
inadvertently placed in Student’s file and never (B) enforceable by Student’s personal
reached the Dean or any faculty member personally. representative even if Student had been
killed in an accident on April 16.
On the following April 1, the above notice was (C) not enforceable on policy grounds because
removed and the following substituted therefor: it produced a noncommercial agreement

73
between a student and his teachers, owner of a lot in Royal Oaks residential
analogous to intramural family agreements subdivision.
and informal social commitments. (D) any use consistent with zoning will be
(D) not enforceable, because Student, after permitted but that such uses so permitted
entering the National Competition in as are in conflict with the restrictions in the
October, was already under a duty to deeds will give rise to a right to damages
perform to the best of his ability. from Owner or Owner’s successor.

315. For this question only, assume that Owner


now desires to open his remaining 100 acres as
Questions 314–315 are based on the following fact a residential subdivision of 125 lots (with
situation. appropriate streets, etc.). He has, as an essential
element of his scheme, the feature that the
Owner held 500 acres in fee simple absolute. In 1960 restrictions should be identical with those he
Owner platted and obtained all required governmental planned for the original Royal Oaks residential
approvals of two subdivisions of 200 acres each. subdivision and, further, that lot owners in Royal
Oaks should be able to enforce (by lawsuits)
In 1960 and 1961 commercial buildings and parking restrictions on the lots in the 100 acres. The
facilities were constructed on one subdivision, Royal zoning for the 100 acres is identical with that
Center, in accordance with the plans disclosed by the for the 200 acres of Royal Oaks residential
plat for that subdivision. Royal Center continues to be subdivision. Which of the following best states
used for commercial purposes. the chance of success for his scheme?

The plat of the other subdivision, Royal Oaks, showed (A) He can restrict use only to the extent
250 lots, streets, and utility and drainage easements. of that imposed by zoning (that is, to
All of the lots in Royal Oaks were conveyed during residential use by not more than four
1960 and 1961. The deeds contained provisions, dwelling units per lot).
expressly stated to be binding upon the grantee and (B) He cannot restrict the 100 acres to
the grantee’s heirs and assigns, requiring the lots to be residential use because of the conflicting
used only for single-family, residential purposes until use for retail commercial purposes in the
1985. The deeds expressly stated that these provisions 200 acres composing the shopping center.
were enforceable by the owner of any lot in the Royal (C) He cannot impose any enforceable
Oaks subdivision. restriction to residential use only.
(D) Any chance of success depends upon the
At all times since 1949, the 200 acres of Royal Center 100 acres being considered by the courts as
have been zoned for shopping center use, and the 200 a part of a common development scheme
acres in Royal Oaks have been zoned for residential which also includes the 200 acres of Royal
use in a classification which permits both single- Oaks.
family and multiple-family use.

314. In an appropriate attack upon the limitation to
residential use by single families, if the evidence 316. A state accredits both public and private schools,
disclosed no fact in addition to those listed licenses their teachers, and supplies textbooks
above, the most probable judicial resolution on secular subjects to all such schools. Country
would be that Schoolhouse, a private school that offers
elementary and secondary education in the state,
(A) there is no enforceable restriction because denies admission to all non-Caucasians. In a
judicial recognition constitutes state action suit to enjoin as unconstitutional the continued
which is in conflict with the Fourteenth racially exclusionary admissions policy of the
Amendment to the United States Country Schoolhouse, which of the following is
Constitution. the strongest argument AGAINST the school?
(B) there is no enforceable restriction because
of Owner’s conflict of interest in that he (A) Because education is a public function, the
did not make the restriction applicable to Country Schoolhouse may not discriminate
the 100 acres he retains. on racial grounds.
(C) the restriction in use set forth in the deeds (B) The state is so involved in school
will be enforced at the suit of any present regulation and support that the equal

74
protection clause of the Fourteenth (B) The performances of Paul and Daniel
Amendment is applicable to the school. under the contract were concurrently
(C) The state is constitutionally obligated to conditional.
eliminate segregation in all public and (C) Payment by Daniel of the $100 was a
private educational institutions within the condition subsequent to Paul’s duty of
state. performance.
(D) Any school with teachers who are licensed (D) Performance by Paul under the contract
by the state is forbidden to discriminate on was a condition precedent to Daniel’s duty
racial grounds. of payment of the $100.

319. Which of the following statements regarding the


Questions 317–319 are based on the following fact legal effect of Daniel’s illness is LEAST accurate?
situation.
(A) Daniel’s illness and the related
Paul and Daniel entered into a contract in writing development excused Paul from his
on November 1, the essential part of which read as obligations to deliver the cards on or
follows: “Paul to supply Daniel with 200 personalized before December 15.
Christmas cards bearing a photograph of Daniel and (B) Prompt notice by Daniel to Paul of
his family on or before December 15, 1970, and Daniel’s recovery from illness was an
Daniel to pay $100 30 days thereafter. Photograph to implied condition of Paul’s duty under the
be taken by Paul at Daniel’s house. Cards guaranteed circumstances.
to be fully satisfactory and on time.” Because Daniel (C) Paul was under a duty of immediate
suddenly became ill, Paul was unable to take the performance of his promise to deliver the
necessary photograph of Daniel and his family until cards on or before December 15 by reason
the first week of December. The final week’s delay of the express language of the contract and
was caused by Paul’s not being notified promptly by despite the illness of Daniel and the related
Daniel of his recovery. Before taking the photograph developments.
of Daniel and his family, Paul advised Daniel that (D) Daniel’s conduct after his illness constituted
he was likely to be delayed a day or two beyond
December 15 in making delivery because of the time a waiver of the necessity of Paul’s
required to process the photograph and cards. Daniel performing on or before December 15.
told Paul to take the photograph anyway. The cards
were finally delivered by Paul to Daniel on December
17, Paul having diligently worked on them in the
interim. Although the cards pleased the rest of the
family, Daniel refused to accept them because, as Questions 320–322 are based on the following fact
he said, squinting at one of the cards at arm’s length situation.
without bothering to put on his reading glasses, “The
photograph makes me look too old. Besides, the cards Carr ran into and injured Pedersen, a pedestrian. With
weren’t delivered on time.” Carr in his car were Wanda and Walter Passenger.
Passerby saw the accident and called the police
317. In an action by Paul against Daniel, which of the department, which sent Sheriff to investigate.
following would be Daniel’s best defense?
All of these people were available as potential
(A) The cards, objectively viewed, were not witnesses in the case of Pedersen v. Carr. Pedersen
satisfactory. alleges that Carr, while drunk, struck Pedersen, who
(B) The cards, subjectively viewed, were not was in a duly marked crosswalk.
satisfactory.
(C) The cards were not delivered on time. 320. Pedersen’s counsel wishes to prove that after
(D) Daniel’s illness excused him from further the accident Carr went to Pedersen and offered
obligation under the contract. $1,000 to settle Pedersen’s claim. The trial judge
should rule this evidence
318. Which of the following statements is most
accurate? (A) admissible as an admission of a party.
(B) admissible as an admission to show Carr’s
(A) Payment by Daniel of the $100 was a liability, provided that the court gives a
condition precedent to Paul’s duty of cautionary instruction that the statement
performance.

75
should not be considered as bearing on the Questions 324–325 are based on the four case
issue of damages. summaries below. For each question, select the case
(C) inadmissible, since it is not relevant to that would be most applicable as a precedent.
either the question of liability or the
question of damages. (A) Commonwealth v. Mason. Two sisters see
(D) inadmissible, because even though it is a wealthy neighbor’s pedigreed dog on the
relevant and an admission, the policy street. They take the dog home, intending
of the law is to encourage settlement to conceal it until the owner offers a
negotiations. reward. Held, guilty of larceny.
(B) Saferite v. State. Two young men saw a
321. Pedersen’s counsel wants to have Sheriff testify motorcar on the street with the keys in
to the following statement made to him by the ignition. They drove the car to a
Walter Passenger, out of the presence of Carr: neighboring town with the intention, they
“We were returning from a party at which we said, of visiting the wife of one of them.
had all been drinking.” The trial judge should The car was wrecked on their way back.
rule this testimony Conviction for larceny reversed.
(C) People v. Noblett. Defendant, a tenant of a
(A) admissible as an admission of a party. city apartment, advertised it for sublease.
(B) admissible as a declaration against interest. Will agreed to sublease for three months,
(C) inadmissible, because it is hearsay, not and on March 12 paid Defendant $550,
within any exception. the total agreed rental. Will was to receive
(D) inadmissible, because it would lead the possession on March 20, but possession
court into nonessential side issues. was never given him. Held, not guilty of
common law larceny.
322. On the evening of the day of the accident, (D) King v. Pear. From a stablekeeper,
Walter Passenger wrote a letter to his sister in Defendant hired a horse to go to Sutton
which he described the accident. When Walter is and back, saying that he would be back
later testifying on direct examination and says he at 8 p.m. He did not return. Investigation
cannot remember some details of the accident, shows that Defendant had given a false
Pedersen’s counsel seeks to show him the letter address, and that he sold the horse the
to assist him. The trial judge should rule this same day. Conviction of larceny affirmed.

(A) permissible under the doctrine of present 324. Jones, angry at a neighbor with whom he had
recollection refreshed. quarreled, for revenge surreptitiously removed
(B) permissible under the doctrine of past a piece of stone statuary from the neighbor’s
recollection recorded. garden and concealed it in his garage. He
(C) objectionable, because the letter was not a intended to return it a day or two later, after
spontaneous utterance. giving the neighbor a chance to feel bad over
(D) objectionable, because the letter is a self- its being stolen. Suspecting who was guilty, the
serving declaration insofar as the witness, neighbor had Jones arrested and charged with
Walter, is concerned. larceny.

325. Harris, a heroin addict, broke into a house and


took several cameras and watches, which he
323. The most generally accepted basis on which a promptly pawned to obtain cash with which to
court will hold that X has a legal duty to aid obtain a “fix.” Harris was later charged with
another is the recognition by X that there is larceny of the cameras and watches.
immediate danger of serious harm to

(A) another human being from a stranger’s
wrongful conduct. 326. Chase, as seller, and Scott, as buyer, enter into
(B) his neighbor from a stranger’s wrongful a written contract for the sale and purchase of
conduct. land that is complete in all respects except that
(C) his cousin from a stranger’s wrongful no reference is made to the quality of title to be
conduct. conveyed. Which of the following will result?
(D) another human being from X’s own non-
negligent conduct. (A) The contract will be unenforceable.

76
(B) Chase will be required to convey a Questions 329–330 are based on the following fact
marketable title. situation.
(C) Chase will be required to convey only
what he owned on the date of the contract. All lawyers practicing in the state of Erewhon must be
(D) Chase will be required to convey only members of the State Bar Association, by order of the
what he owned on the date of the contract state supreme court. Several state officials serve on the
plus whatever additional title rights he may Bar Association’s Board of Bar Governors. The Board
acquire prior to the closing date. of Bar Governors authorizes the payment of dues for
two staff members to the Cosmopolitan Club, a private
327. A state statute requires that all buses which dining club licensed to sell alcoholic beverages.
operate as common carriers on the highways of The Cosmopolitan Club is frequented by affluent
the state shall be equipped with seat belts for businessmen and professionals and by legislators. It is
passengers. Transport Lines, an interstate carrier, generally known that the purpose of the membership
challenges the validity of the statute and the of the Bar Association staff is to enable them to go
right of the state to make the requirement. What where members of the “elite” meet and to lobby for
is the best basis for a constitutional challenge by legislation in which the Bar Association is interested.
Transport Lines? The State Bar Association has numerous committees
and subcommittees concerned with family law,
(A) Violation of the due process clause of the real estate law, unauthorized practice, etc., and its
Fourteenth Amendment. recommendations often influence state policy. Some
(B) Violation of the equal protection clause of committee meetings are held at the Cosmopolitan
the Fourteenth Amendment. Club. The club is known to have rules which restrict
(C) Unreasonable burden on interstate membership by race, religion, and sex.
commerce.
(D) Difficulty of enforcement. Plaintiffs, husband and wife, who are members of the
Erewhon Bar Association, petitioned the Board of
328. Amy Docent, a state college instructor, was Bar Governors to adopt a resolution prohibiting the
discharged because of her refusal to comply payment of club dues to and the holding of meetings
with a state statute requiring public employees of the Bar Association or its committees at places
to swear or affirm that they will (1) “uphold which discriminate on the basis of race, religion, or
and defend” the state and federal constitutions sex. After substantial public discussion, the Board of
and (2) “oppose the overthrow” of the state or Bar Governors, by a close vote, failed to pass such a
federal government “by force, violence, or any resolution. These events received extensive coverage
improper method.” The statute had previously in the local newspapers. Plaintiffs have brought an
been held constitutional by the state supreme action in federal court seeking an injunction against
court. Docent filed a complaint in federal district such payments and the holding of meetings in such
court alleging the unconstitutionality of the places as the Cosmopolitan Club.
statute and seeking an injunction and damages.
329. The strongest argument for Plaintiffs is
Which of the following is the state’s strongest
argument for sustaining the validity of the (A) private rights to discriminate and associate
statute? freely must defer to a public interest
against discrimination on the basis of race,
(A) Government employment is a privilege, religion, or sex.
not a right. (B) the failure of the State Bar Association to
(B) The oath as a whole is only a commitment pass a resolution forbidding discrimination
to abide by constitutional processes. on the basis of race, religion, or sex
(C) The First and Fourteenth Amendments constitutes a denial of equal protection.
permit a state to fix the conditions of state (C) the State Bar Association is an agency of
employment. the state and its payment of dues to such
(D) The state has a compelling need to keep private clubs promotes discrimination on
disloyal persons out of governmental the basis of race, religion, and sex.
positions of trust. (D) the State Bar Association’s payment
of dues to such private clubs promotes
discrimination on the basis of race,
religion, and sex.

77
330. Which of the following actions should a federal (B) ordered, because Alpha alone owns the
district court take with respect to jurisdiction? entire fee simple in Blackacre.
(C) denied, because Bill has a valid interest in
(A) Hear the case on the merits, because a Blackacre.
federal claim is presented. (D) denied, because the American Red Cross
(B) Hear the case on the merits, because the has a valid interest in Blackacre.
expenditure of state funds in support
of segregation is forbidden by the Fifth 332. In 1946, the interest of the American Red Cross
Amendment. in Blackacre could be best described as a
(C) Abstain from jurisdiction, because the
constitutional issue should be litigated first (A) valid contingent remainder.
in a state court. (B) void executory interest.
(D) Dismiss the case for lack of jurisdiction, (C) valid executory interest.
because the issue of Bar Association (D) void contingent remainder.
activities is solely within the domain of
state law.

Questions 333–335 are based on the following fact


situation.
Questions 331–332 are based on the following fact
situation. Brill saved the life of Ace’s wife, Mary, who
thereafter changed her will to leave Brill $1,000.
In 1945, Owen, owner of both Blackacre and However, upon Mary’s death she had no property
Whiteacre, executed and delivered two separate except an undivided interest in real estate held in
deeds by which he conveyed the two tracts of land tenancy by the entirety of Ace. The property had been
as follows: Blackacre was conveyed “To Alpha purchased by Ace from an inheritance.
and his heirs as long as it is used exclusively for
residential purposes, but if it is ever used for other After Mary died, Ace signed and delivered to Brill
than residential purposes, to the American Red Cross.” the following instrument: “In consideration of Brill’s
Whiteacre was conveyed “To Beta and her heirs as saving my wife’s life and his agreement to bring no
long as it is used exclusively for residential purposes, claims against my estate based on her will, I hereby
but if it is used for other than residential purposes promise to pay Brill $1,000.”
prior to 1965, then to the Salvation Army.” In 1950,
Owen died leaving a valid will by which he devised Upon Ace’s death, Brill filed a claim for $1,000.
all his real estate to his brother, Bill. The will had no Ace’s executor contested the claim on the ground
residuary clause. Owen was survived by Bill and by that the instrument was not supported by sufficient
Owen’s daughter, Delia, who was Owen’s sole heir. consideration.

For the purpose of this set of questions, it may be 333. In most states, would Brill’s saving of Mary’s
assumed that the common law rule against perpetuities life be regarded as sufficient consideration for
applies in the state where the land is located and that Ace’s promise?
the state also has a statute providing that “all future
estates and interests are alienable, descendible, and (A) Yes, because Ace was thereby morally
devisable in the same manner as possessory estates obligated to Brill.
and interests.” (B) Yes, because Ace was thereby materially
benefited.
331. In 1955, Alpha and Delia entered into a contract (C) No, because Ace had not asked Brill to
with John whereby Alpha and Delia contracted save her.
to sell Blackacre to John in fee simple. After (D) No, because the value of Brill’s act was
examining the title, John refused to perform on too uncertain.
the ground that Alpha and Delia could not give
good title. Alpha and Delia joined in an action 334. With respect to the recital that Brill had agreed
against John for specific performance. Specific not to file a claim against Ace’s estate, what
performance will be additional fact would most strengthen Brill’s
claim?
(A) ordered, because Alpha and Delia together
own a fee simple absolute in Blackacre.

78
(A) Brill’s agreement was made in a writing he (B) Brown did not know that his act was
signed. morally wrong.
(B) Brill reasonably believed he had a valid (C) Brown did not know the quality of the act
claim when the instrument was signed. he was performing.
(C) Mary had contributed to accumulation of (D) Brown’s acts were the product of a mental
the real property. disease.
(D) Brill paid Ace $1 when he received the
instrument.
Questions 337–340 are based on the following fact
335. On which of the following theories would it be situation.
most likely that Brill could recover?
Walker, a pedestrian, started north across the street in
(A) Ace and Brill have made a compromise. a clearly marked north-south crosswalk with the green
(B) Ace must give restitution for benefits it traffic light in her favor. Walker was in a hurry, and
would be unjust to retain. before reaching the north curb on the street, she cut
(C) Ace is bound by promissory estoppel. to her left diagonally across the street to the east-west
(D) Ace executed a binding unilateral contract. crosswalk and started across it. Just after she reached
the east-west crosswalk, the traffic light turned green
in her favor. She had proceeded about five steps
farther across the street to the west in the crosswalk
336. Brown suffered from the delusion that he was a when she was struck by a car approaching from her
special agent of God. He frequently experienced right that she thought would stop but did not. The car
hallucinations in the form of hearing divine was driven by Driver, 81 years of age, who failed to
commands. Brown believed God told him stop his car after seeing that the traffic light was red
several times that the local Roman Catholic against him. Walker had a bone disease, resulting
bishop was corrupting the diocese into heresy, in very brittle bones, that is prevalent in only 0.02
and that the bishop should be “done away with.” percent of the population. As a result of the impact
Brown, a devout Catholic, conceived of himself Walker suffered a broken leg and the destruction
as a religious martyr. He knew that shooting of her family heirloom, a Picasso original painting
bishops for heresy is against the criminal law. that she was taking to her bank for safekeeping. The
He nevertheless carefully planned how he might painting had been purchased by Walker’s grandmother
kill the bishop. One evening Brown shot the for $750 but was valued at $500,000 at the time of the
bishop, who was then taken to the hospital where accident.
he died two weeks later.
Walker has filed suit against Driver. Driver’s attorney
Brown told the police he assumed the institutions has alleged that Walker violated a state statute
of society would support the ecclesiastical requiring that pedestrians stay in crosswalks, and that
hierarchy and he expected to be persecuted if Walker had not violated the statute she would have
for his God-inspired actions. Psychiatrist had to walk 25 feet more to reach the impact point
Stevens examined Brown and found that Brown and therefore would not have been at a place where
suffered from schizophrenic psychosis, that she could have been hit by Driver. Walker’s attorney
in the absence of this psychosis, he would not ascertains that there is a statute as alleged by Driver,
have shot the bishop, and that because of the that his measurements are correct, that there is a state
psychosis, Brown found it extremely difficult to statute requiring observance of traffic lights, and that
determine whether he should obey the specific Driver’s license expired two years prior to the collision.
command that he do away with the bishop or
the general commandment “Thou shalt not kill.” 337. The violation of the crosswalk statute by Walker
Brown was charged with murder. should not defeat her cause of action against
Driver because
If Brown interposes an insanity defense and
the jurisdiction in which he is tried has adopted (A) Driver violated the traffic light statute at a
only the M’Naghten test of insanity, then the later point in time than Walker’s violation.
strongest argument for the defense under that (B) pedestrians are entitled to assume that
test is that automobile drivers will obey the law.
(C) Walker was hit while in the crosswalk.
(A) Brown did not know the nature of the act
he was performing.

79
(D) the risks that the statute was designed to prompt medical attention, Paulsen suffered a
protect against probably did not include an severe brain injury from lack of oxygen.
earlier arrival at another point.
If Paulsen asserts a claim against Dow for his
338. The failure of Driver to have a valid driver’s injuries, will Paulsen prevail?
license has which of the following effects?
(A) Yes, if the jurisdiction relieves physicians
(A) It makes Driver liable to Walker because of malpractice liability for emergency first
Driver is a trespasser on the highway. aid.
(B) It would not furnish a basis for liability. (B) Yes, if a reasonably prudent person with
(C) It proves that Driver is an unfit driver in Dow’s experience, training, and knowledge
this instance. would have assisted Paulsen.
(D) It makes Driver absolutely liable for (C) No, because Dow was not responsible for
Walker’s injury. Paulsen’s condition.
(D) No, because Dow knew that Paulsen was
339. If Walker establishes liability on the part of substantially certain to sustain serious
Driver for her physical injuries, should Walker’s injury.
recovery include damages for a broken leg?

(A) No, since only 0.02 percent of the Questions 342–343 are based on the following fact
population have bones as brittle as situation.
Walker’s.
(B) No, unless a person of ordinary health The state of Champlain enacts the Young Adult
would probably have suffered a broken leg Marriage Counseling Act, which provides that,
from the impact. before any persons less than 30 years of age may
(C) Yes, because Driver could foresee be issued a marriage license, they must receive at
that there would be unforeseeable least five hours of marriage counseling from a state-
consequences of the impact. licensed social worker. This counseling is designed
(D) Yes, even though the extent of the injury to assure that applicants for marriage licenses know
was not a foreseeable consequence of the their legal rights and duties in relation to marriage
impact. and parenthood, understand the “true nature” of the
marriage relationship, and understand the procedures
340. Walker’s violation of the crosswalk statute for obtaining divorces.
should not be considered by the jury because
342. Pine, aged 25, contemplated marrying Ross,
(A) there is no dispute in the evidence about aged 25. Both are residents of the state of
factual cause. Champlain. Pine has not yet proposed to Ross
(B) as a matter of law, the violation of the because he is offended by the counseling
statute results in liability for all resulting requirement.
harm.
(C) as a matter of law, Driver’s conduct was Pine sues in court seeking a declaratory
an independent intervening cause. judgment that the Young Adult Marriage
(D) as a matter of law, the injury to Walker Counseling Act is unconstitutional. Which of the
was not the result of a risk the statute was following is the clearest ground for dismissal of
designed to protect against. this action by the court?

(A) Pine and Ross are residents of the same


state.
341. Paulsen was eating in a restaurant when he (B) No substantial federal question is
began to choke on a piece of food that had presented.
lodged in his throat. Dow, a physician who was (C) The suit presents a nonjustifiable political
sitting at a nearby table, did not wish to become question.
involved and did not render any assistance, (D) The suit is unripe.
although prompt medical attention would have
been effective in removing the obstruction from 343. In a case in which the constitutionality of the
Paulsen’s throat. Because of the failure to obtain Young Adult Marriage Counseling Act is in

80
issue, the burden of persuasion will probably be 344. In an action brought by Bell to enjoin Akers
on the from interfering with Bell’s continued use of
the common driveway between the two lots, the
(A) person challenging the law, because there decision should be for
is a strong presumption that elected state
legislators acted properly. (A) Akers, because the termination of the
(B) person challenging the law, because the necessity for the easement terminated the
Tenth Amendment authorizes states to easement.
determine the conditions on which they (B) Akers, because the continuation of the
issue marriage licenses. easement after the change of circumstances
(C) state, because there is a substantial impact would adversely affect the marketability of
on the right to marry, and that right is both lots without adding any commensurate
fundamental. value to either.
(D) state, because there is a substantial impact (C) Bell, because an incorporeal hereditament
on the discrete and insular class of young lies in grant and cannot be terminated
adults. without a writing.
(D) Bell, because the removal of the need for
the easement created by express grant does
not affect the right to the easement.
Questions 344–345 are based on the following fact
situation. 345. In an action brought by Bell to enjoin Ogden
from erecting the apartment building in such a
Ogden was the fee simple owner of three adjoining way as to obstruct the view from Bell’s living
vacant lots fronting on a common street in a primarily room window, the decision should be for
residential section of a city which had no zoning
laws. The lots were identified as Lots 1, 2, and 3. (A) Bell, because Ogden’s proposed building
Ogden conveyed Lot 1 to Akers and Lot 2 to Bell. would be an obstruction of Bell’s natural
Ogden retained Lot 3, which consisted of three acres right to an easement for light and air.
of woodland. Bell, whose lot was between the other (B) Bell, because Bell was misled by Ogden’s
two, built a house on his lot. Bell’s house included a failure to complain when Bell was building
large window on the side facing Lot 3. The window his house.
provided a beautiful view from Bell’s living room, (C) Ogden if, but only if, it can be shown that
thereby adding value to Bell’s house. Ogden’s intention to erect such a building
was made known to Bell at or prior to the
Akers erected a house on his lot. Ogden made no time of Ogden’s conveyance to Bell.
complaint to either Akers or Bell concerning the (D) Ogden, because Bell has no easement for
houses they built. After both Akers and Bell had light, air, or view.
completed their houses, the two of them agreed to
and did build a common driveway running from the
street to the rear of their respective lots. The driveway
was built on the line between the two houses so that 346. Dever was indicted for the murder of Vickers
one-half of the way was located on each lot. Akers by poison. At trial, the prosecutor calls the
and Bell exchanged right-of-way deeds by which each county coroner, Dr. Wolfe, who is a board-
of them conveyed to the other, his heirs and assigns, certified pathologist, to testify that, in accord
an easement to continue the right of way. Both deeds with good practice in her specialty, she has
were properly recorded. After Akers and Bell had studied microphotographic slides, made under
lived in their respective houses for 30 years, a new her supervision by medical assistants, of tissue
public street was built bordering on the rear of Lots taken from Vickers’ corpse and that it is Wolfe’s
1, 2, and 3. Akers informed Bell that, since the new opinion, based on that study, that Vickers died
street removed the need for their common driveway, of poisoning. The slides have not been offered in
he considered the right-of-way terminated; therefore, evidence.
he intended to discontinue its use and expected Bell to
do the same. At about the same time, Ogden began the Dr. Wolfe’s opinion should be
erection of a six-story apartment house on Lot 3. If the
apartment house is completed, it will block the view (A) excluded, because the cause of death is a
from Bell’s window and will substantially reduce the critical issue to be decided by the trier of
value of Bell’s lot. fact.

81
(B) excluded, because her opinion is based on (A) The state sprinkler requirement denies the
facts not in evidence. company property or liberty without due
(C) admitted, because Wolfe followed process.
accepted medical practice in arriving at her (B) The state sprinkler requirement denies the
opinion. company equal protection of the laws.
(D) admitted, because her opinion is based (C) As applied, the state sprinkler requirement
on matters observed pursuant to a duty violates the supremacy clause.
imposed by law. (D) As applied, the state sprinkler requirement
violates the obligation of contracts clause.
347. At the trial of Davis for a murder that occurred
in Newtown, the prosecution called Waite, who 349. On January 15, Carpenter agreed to repair
testified that she saw Davis kill the victim. Davis Householder’s house according to certain
believed that Waite was 600 miles away in Old specifications and to have the work completed
Town, engaged in the illegal sale of narcotics, by April 1. On March 1, Householder’s property
on the day in question. On cross-examination was inundated by floodwaters which did not
by Davis, Waite was asked whether she had in abate until March 15. Householder could not get
fact sold narcotics in Old Town on that date. the house into a condition which would permit
Waite refused to answer on the ground of self- Carpenter to begin the repairs until March 31.
incrimination. On that date Carpenter notified Householder that
he would not repair the house.
The judge, over the prosecutor’s objection,
ordered that if Waite did not testify, her direct Which one of the following facts, if it was the
testimony should be stricken. The order to only one true and known to both parties on
testify or have the testimony stricken can best be January 15, would best serve Carpenter as the
supported on the basis that basis for a defense in an action brought against
him by Householder for breach of contract?
(A) Waite had not been charged with any
crime and, thus, could claim no privilege (A) Carpenter’s busy schedule permitted him
against self-incrimination. to work on Householder’s house only
(B) Waite’s proper invocation of the privilege during the month of March.
prevented adequate cross-examination. (B) Any delay in making the repairs would not
(C) the public interest in allowing an accused seriously affect Householder’s use of the
to defend himself or herself outweighs property.
the interest of a nonparty witness in the (C) The cost of making repairs was increasing
privilege. at the rate of three percent a month.
(D) the trial record, independent of testimony, (D) The area around Householder’s property
does not establish that Waite’s answer was frequently flooded during the month
could incriminate her. of March.

348. A statute of the state of Tuscarora made it a 350. In a telephone call on March 1, Adams, an
misdemeanor to construct any building of more unemployed, retired person, said to Dawes, “I
than five stories without an automatic fire will sell my automobile for $3,000 cash. I will
sprinkler system. hold this offer open through March 14.” On
March 12, Adams called Dawes and told her
A local construction company built a 10-story that he had sold the automobile to Clark. Adams
federal office building in Tuscarora. It in fact had not sold the automobile to anyone.
constructed the building according to the precise On March 14, Dawes learned that Adams still
specifications of a federal contract authorized owned the automobile, and on that date called
by federal statutes. Because the building was Adams and said, “I’m coming over to your place
built without the automatic fire sprinkler system with $3,000.” Adams replied, “Don’t bother. I
required by state law, Tuscarora decided to won’t deliver the automobile to you under any
prosecute the private contractor. circumstances.” Dawes protested, but made no
further attempt to pay for or take delivery of the
Which of the following is the company’s automobile.
strongest defense to that prosecution?

82
In an action by Dawes against Adams for breach 352. If Child’s claims against Mobilco, Heatco, and
of contract, Dawes probably will Coolco are based on strict liability in tort, Child
will probably recover against
(A) succeed, because Adams had assured her
that the offer would remain open through (A) Mobilco only, because the ventilating
March 14. system was defectively designed by
(B) succeed, because Adams had not in fact Mobilco.
sold the automobile to Clark. (B) Heatco only, because it was the excessive
(C) not succeed, because Dawes had not heat from the furnace that caused Child’s
tendered the $3,000 to Adams on or before injuries.
March 14. (C) Mobilco and Heatco only, because the
(D) not succeed, because on March 12, Adams combination of Mobilco’s design and
had told Dawes that he had sold the Heatco’s furnace caused Child’s injuries.
automobile to Clark. (D) Mobilco, Heatco, and Coolco, because the
combination of Mobilco’s design, Heatco’s
furnace, and Coolco’s air conditioning unit
Questions 351–352 are based on the following fact caused Child’s injuries.
situation.

Parents purchased a new mobile home from Seller.
The mobile home was manufactured by Mobilco and 353. Cars driven by Pugh and Davidson collided,
had a ventilating system designed by Mobilco with and Davidson was charged with driving while
both a heating unit and an air conditioner. Mobilco intoxicated in connection with the accident. She
installed a furnace manufactured by Heatco and pleaded guilty and was merely fined, although
an air conditioning unit manufactured by Coolco. under the statute the court could have sentenced
Each was controlled by an independent thermostat her to two years in prison.
installed by Mobilco. Because of the manner in
which Mobilco designed the ventilating system, the Thereafter, Pugh, alleging that Davidson’s
first time the ventilating system was operated by intoxication had caused the collision, sued
Parents, cold air was vented into Parents’ bedroom Davidson for damages. At trial, Pugh offers the
to keep the temperature at 68°F (20°C). The cold air properly authenticated record of Davidson’s
then activated the heater thermostat, and hot air was conviction. The record should be
pumped into the bedroom of Child, the six-month-
old child of Parents. The temperature in Child’s (A) admitted as proof of Davidson’s character.
room reached more than 170°F (77°C) before Child’s (B) admitted as proof of Davidson’s
mother became aware of the condition and shut the intoxication.
system off manually. As a result, Child suffered (C) excluded, because the conviction was not
permanent physical injury. the result of a trial.
(D) excluded, because it is hearsay, not within
Claims have been asserted by Child, through a duly any exception.
appointed guardian, against Mobilco, Seller, Heatco,
and Coolco. 354. Pitt sued Dow for damages for injuries that Pitt
incurred when a badly rotted limb fell from a
351. If Child’s claim against Seller is based on curbside tree in front of Dow’s home and hit
negligence, the minimum proof necessary to Pitt. Dow claimed that the tree was on city
establish Seller’s liability is that the ventilating property and thus was the responsibility of the
system city. At trial, Pitt offered testimony that a week
after the accident, Dow had cut the tree down
(A) was defective. with a chain saw. The offered evidence is
(B) was defective and had not been inspected
by Seller. (A) inadmissible, because there is a policy to
(C) was defective and had been inspected by encourage safety precautions.
Seller, and the defect was not discovered. (B) inadmissible, because it is irrelevant to
(D) was defective, and the defect would have the condition of the tree at the time of the
been discovered if Seller had exercised accident.
reasonable care in inspecting the system. (C) admissible to show that the tree was on
Dow’s property.

83
(D) admissible to show that the tree was in a 357. Carver is a chemical engineer. She has no
rotted condition. interest in or connection with Chemco. Carver
noticed that Chemco’s most recent publicly
355. A state statute makes it a felony for any teacher issued financial statement listed, as part of
at a state institution of higher education to Chemco’s assets, a large inventory of a certain
accept anything of value from a student at the special chemical compound. This asset was
same institution. Monroe, a student at the state listed at a cost of $100,000, but Carver knew
university, offered Professor Smith, his English that the ingredients of the compound were in
teacher, $50 in exchange for a good grade in short supply and that the current market value of
his English course. Smith agreed and took the the inventory was in excess of $1,000,000. There
money. Professor Smith and Monroe are tried was no current public quotation of the price of
jointly for violation of the state statute. Professor Chemco stock. The book value of Chemco stock,
Smith is charged with violating the statute, and according to the statement, was $5 a share; its
Monroe with aiding and abetting him. actual value was $30 a share.

Monroe’s best argument for a dismissal of the Knowing these facts, Carver offered to purchase
charge against him is that from Page at $6 a share the 1,000 shares of
Chemco stock owned by Page. Page and Carver
(A) a principal and an accessory cannot be had not previously met. Page sold the stock to
tried together, since the principal must be Carver for $6 a share.
convicted first.
(B) he cannot be an accessory, since he is the If Page asserts a claim based on misrepresentation
victim of the crime. against Carver, will Page prevail?
(C) the legislature did not intend to punish the
person giving the thing of value. (A) Yes, because Carver knew that the value
(D) he did not assist Professor Smith in of the stock was greater than the price she
violating the statute. offered.
(B) Yes, if Carver did not inform Page of the
356. In which of the following situations is Defendant true value of the inventory.
most likely to be guilty of the crime charged? (C) No, unless Carver told Page that the stock
was not worth more than $6 a share.
(A) Without the permission of Owner, (D) No, if Chemco’s financial statement was
Defendant takes Owner’s car with the available to Page.
intention of driving it three miles to a
grocery store and back. Defendant is 358. Dean, charged with murder, was present with
charged with larceny. her attorney at a preliminary examination when
(B) Defendant gets permission to borrow White, who was the defendant in a separate
Owner’s car for the evening by falsely prosecution for concealing the body of the
promising to return it, although he does not murder victim, testified for the prosecution
intend to do so. Two days later, he changes against Dean. When called to testify at Dean’s
his mind and returns the car. Defendant is trial, White refused to testify, though ordered to
charged with larceny by trick. do so.
(C) Defendant gets permission to borrow
Owner’s car for the evening by The prosecution offers evidence of White’s
misrepresenting his identity and falsely testimony at the preliminary examination. The
claiming he has a valid driver’s license. He evidence is
returns the car the next day. Defendant is
charged with obtaining property by false (A) admissible as former testimony.
pretenses. (B) admissible as past recollection recorded.
(D) With permission, Defendant, promising to (C) inadmissible, because it would
return it by 9 p.m., borrows Owner’s car. violate White’s privilege against self-
Later in the evening, Defendant decides incrimination.
to keep the car until the next morning (D) inadmissible, because it is hearsay, not
and does so. Defendant is charged with within any exception.
embezzlement.
359. Potts, a building contractor, sued Dennis for
failure to pay on a small cost-plus construction

84
contract. At trial, Potts, who personally 361. A grand jury was investigating a bank robbery.
supervised all of the work, seeks to testify to The only information known to the prosecutor
what he remembers about the amount of pipe was a rumor that Taylor might have been
used, the number of workers used on the job, involved. The grand jury subpoenaed Taylor. He
and the number of hours spent grading. refused to answer questions about the robbery
and was granted use immunity. He then testified
Dennis objects on the ground that Potts had that he and Simmons had robbed the bank. The
routinely recorded these facts in notebooks grand jury indicted both Taylor and Simmons
which are in Potts’ possession. for the bank robbery. The prosecutor permitted
Simmons to enter a plea to a lesser offense in
Potts’ testimony is exchange for Simmons’ agreement to testify
against Taylor. The prosecutor had no evidence
(A) admissible as a report of regularly as to the identity of the robbers except the
conducted business activity. testimony of Simmons and Taylor.
(B) admissible as based on first-hand
knowledge. At Taylor’s trial, his objection to Simmons’
(C) inadmissible, because it violates the best being permitted to testify should be
evidence rule.
(D) inadmissible, because a summary of (A) sustained, because the prosecutor may not
writings cannot be made unless the bargain away the rights of one codefendant
originals are available for examination. in a deal with another.
(B) sustained, because Simmons’ testimony
360. While testifying as a witness in a civil trial, was acquired as a result of Taylor’s grand
Walters was asked on cross-examination if he jury testimony.
had been convicted in the circuit court of Jasper (C) overruled, because the police suspected
County of stealing $200 from his employer Taylor even before he testified in the grand
on August 16, 1977. Walters said, “No, I have jury hearing.
never been convicted of any crime.” In fact, (D) overruled, because a witness cannot be
Walters had pleaded guilty to such a charge and precluded from testifying if his testimony
had been placed on probation. is given voluntarily.

Walters was then charged with perjury on the 362. A federal statute requires United States civil
ground that his statement denying the conviction service employees to retire at age 75. However,
was false. A statute in the jurisdiction defines that statute also states that civil service employees
perjury as knowingly making a false statement of the armed forces must retire at age 65.
while under oath.
Prentis, a 65-year-old service employee of the
At trial, the state proved Walters’ statement and Department of the Army, seeks a declaratory
the prior conviction. Walters testified that the judgment that would forbid his mandatory
attorney who represented him in the theft case retirement until age 75.
had told him that, because he had been placed
on probation, he had not been convicted of a The strongest argument that Prentis can make to
crime. Walters had served his probationary invalidate the requirement that he retire at age 65
period satisfactorily and been discharged from is that the law
probation. The alleged advice of the attorney
was incorrect. (A) denies him a privilege or immunity of
national citizenship.
If the jury believes Walters, it should find him (B) deprives him of a property right without
just compensation.
(A) guilty, because his mistake was one of law. (C) is not within the scope of any of the
(B) guilty, because reliance on the advice of an enumerated powers of Congress in Article I,
attorney is not a defense. §8.
(C) not guilty if the jury also finds that his (D) invidiously discriminates against him on
reliance on the attorney’s advice was the basis of age in violation of the Fifth
reasonable. Amendment.
(D) not guilty, because he lacked the necessary
mental state.

85
363. Light Company is the sole distributor of electrical with her free hand. Smythe then dragged Nelson
power in City. The Company owns and maintains from her car, forced her into his squad car, and
all of the electric poles and equipment in City. took her to the police station.
Light Company has complied with the National
Electrical Safety Code, which establishes The incident took place on the street in front of
minimum requirements for the installation and the apartment where Nelson and her aged father,
maintenance of power poles. The Code has been Joplin, lived. Smythe did not know that Joplin
approved by the federal and state governments. had observed what took place from a window in
the apartment.
Light Company has had to replace insulators on
its poles repeatedly because unknown persons If Nelson’s father, Joplin, asserts a claim against
repeatedly shoot at and destroy them. This Smythe for the intentional infliction of emotional
causes the power lines to fall to the ground. distress, will Joplin prevail?
On one of these occasions, Paul, Faber’s five-
year-old son, wandered out of Faber’s yard, (A) Yes, if Smythe’s acts caused Joplin severe
intentionally touched a downed wire, and was emotional distress.
seriously burned. (B) Yes, if it is found that Smythe’s behavior
was extreme and outrageous with respect
If a claim on Paul’s behalf is asserted against to Nelson.
Light Company, the probable result is that Paul (C) No, because Smythe did not know that
will Joplin was watching.
(D) No, because Joplin was not within the zone
(A) recover if Light Company could have of physical danger.
taken reasonable steps to prevent the lines
from falling when the insulators were 365. Davison was driving through an apartment
destroyed. building area plagued with an unusually high
(B) recover, because a supplier of electricity is incidence of burglaries and assaults. Acting
strictly liable in tort. pursuant to a police department plan to combat
(C) not recover unless Light Company failed crime by the random stopping of automobiles in
to exercise reasonable care to stop the the area between midnight and 6 a.m., a police
destruction of the insulators. officer stopped Davison and asked him for
(D) not recover, because the destruction of the identification. As Davison handed the officer
insulators was intentional. his license, the officer directed a flashlight into
the automobile and saw what appeared to be the
364. The city of Metropolis has an ordinance that barrel of a shotgun protruding from under the front
makes it an offense, punishable by fine, for seat on the passenger side of the car. The officer
the owner of a dog to permit the dog to run ordered Davison from the car, searched him, and
unleashed on a public way. discovered marijuana cigarettes and a shotgun.

Smythe, a police officer, observed a small dog At Davison’s trial for unlawful possession of
running loose in the street. As Smythe picked narcotics, his motion to suppress the use of the
the dog up, Nelson, who was seated in her car marijuana as evidence should be
lawfully parked at the curb, called out, “Oh,
thank you, Officer, for returning Fido.” Smythe (A) sustained, because the marijuana was
asked Nelson whether the dog was hers, and discovered as a result of the unlawful
when she acknowledged ownership, he asked stopping of Davison’s automobile.
to see her driver’s license. Nelson gave her (B) sustained, because the use of the flashlight
name and address, but she refused to produce a constituted a search of the interior of
driver’s license. Smythe then told her to produce Davison’s automobile without probable
her driver’s license if she did not want to go cause.
to jail. Nelson responded by saying, “Isn’t this (C) denied, because the officer’s conduct was
ridiculous?” Smythe took her by the arm and consistent with the established police plan.
said, “Let’s go. You are under arrest.” (D) denied, because the discovery of the gun in
plain view created the reasonable suspicion
Nelson cried out that Smythe was hurting her but necessary to justify the arrest and search of
he refused to release her arm, and she struck him Davison.

86
366. In which of the following situations is Defendant shall construct and thereafter maintain and
most likely to be guilty of common law murder? operate on said premises a public health
center.
(A) During an argument in a bar, Norris
punches Defendant. Defendant, mistakenly The grantee, Burton, constructed a public health
believing that Norris is about to stab him, center on the tract within the time specified
shoots and kills Norris. and operated it for five years. At the end of
(B) While committing a robbery of a liquor this period, Burton converted the structure
store, Defendant accidentally drops his into a senior citizens’ recreational facility. It is
revolver, which goes off. The bullet strikes conceded by all parties in interest that a senior
and kills Johnson, a customer in the store. citizens’ recreational facility is not a public
(C) While hunting deer, Defendant notices health center.
something moving in the bushes. Believing
it to be a deer, Defendant fires into the In an appropriate action, Anders seeks a
bushes. The bullet strikes and kills Griggs, declaration that the change in the use of the
another hunter. facility has caused the land and structure to
(D) In celebration of the Fourth of July, revert to her. In this action, Anders should
Defendant discharges a pistol within the
city limits in violation of a city ordinance. (A) win, because the language of the deed
The bullet ricochets off the street and created a determinable fee, which leaves a
strikes and kills Abbott. possibility of reverter in the grantor.
(B) win, because the language of the deed
367. Morgan conveyed Greenacre, her one-family created a fee subject to condition
residence, to “Perez for life, remainder to Rowan, subsequent, which leaves a right of entry
her heirs and assigns, subject, however, to First or power of termination in the grantor.
Bank’s mortgage thereon.” There was an unpaid (C) lose, because the language of the deed
balance on the mortgage of $10,000, which created only a contractual obligation and
is payable in $1,000 annual installments plus did not provide for retention of property
interest at six percent on the unpaid balance, interest by the grantor.
with the next payment due on July 1. Perez (D) lose, because an equitable charge is
is now occupying Greenacre. The reasonable enforceable only in equity.
rental value of the property exceeds the sum
necessary to meet all current charges. There is 369. A group of children, ranging in age from 8 to 15,
no applicable statute. regularly played football on the common area of
an apartment complex owned by O’Neill. Most
Under the rules governing contributions between of the children lived in the apartment complex,
life tenants and remaindermen, how should the but some lived elsewhere. O’Neill knew that the
burden for payment be allocated? children played on the common area and had not
objected.
(A) Rowan must pay the principal payment,
but Perez must pay the interest to First Peter, a 13-year-old who did not live in the
Bank. apartment complex, fell over a sprinkler head
(B) Rowan must pay both the principal and while running for a pass and broke his leg.
interest payments to First Bank. Although Peter had played football on the
(C) Perez must pay both the principal and common area before, he had never noticed the
interest payments to First Bank. sprinkler heads, which protruded one inch above
(D) Perez must pay the principal payment, but the ground and were part of a permanently
Rowan must pay the interest to First Bank. installed underground sprinkler system.

368. Anders conveyed her only parcel of land If a claim is asserted on Peter’s behalf, Peter will
to Burton by a duly executed and delivered
warranty deed, which provided: (A) prevail if the sprinkler head was a hazard
that Peter probably would not discover.
To have and to hold the described tract (B) prevail, because O’Neill had not objected
of land in fee simple, subject to the to children playing on the common area.
understanding that within one year from
the date of the instrument said grantee

87
(C) not prevail, because Peter did not live in 372. In an action by Byer against Singer for breach of
the apartment complex. contract, Byer probably will
(D) not prevail unless the sprinkler heads
were abnormally dangerous to users of the (A) succeed, because the carrier will be
common area. deemed to be Singer’s agent.
(B) succeed, because the risk of loss was on
370. Congress passes a law regulating the wholesale Singer.
retail prices of “every purchase or sale of oil, (C) not succeed, because of impossibility of
natural gas, and electric power made in the performance.
United States.” The strongest argument in (D) not succeed, because the risk of loss was
support of the constitutionality of this statute is on Byer.
that

(A) the Constitution expressly empowers
Congress to enact laws for “the general 373. Dutton, disappointed by his eight-year-old son’s
welfare.” failure to do well in school, began systematically
(B) Congress has the authority to regulate such depriving the child of food during summer
products’ interstate transportation and vacation. Although his son became seriously ill
importation from abroad. from malnutrition, Dutton failed to call a doctor.
(C) Congress may regulate the prices of every He believed that as a parent he had the sole
purchase and sale of goods and services right to determine whether the child was fed or
made in this country, because commerce received medical treatment. Eventually the child
includes buying and selling. died. An autopsy disclosed that the child had
(D) in inseverable aggregates, the domestic suffered agonizingly as a result of the starvation,
purchases or sales of such products affect that a physician’s aid would have alleviated the
interstate or foreign commerce. suffering, and that although the child would have
died in a few months from malnutrition, the actual
cause of death was an untreatable form of cancer.
Questions 371–372 are based on the following fact
situation. The father was prosecuted for murder, defined in
the jurisdiction as “unlawful killing of a human
In a written contract, Singer agreed to deliver to Byer being with malice aforethought.” The father
500 described chairs at $20 each F.O.B. Singer’s place should be
of business. The contract provided that “neither party
will assign this contract without the written consent of (A) acquitted, because of the defendant’s good
the other.” Singer placed the chairs on board a carrier faith belief concerning parental rights in
on January 30. On February 1 Singer said in a signed supervising children.
writing, “I hereby assign to Wheeler all my rights (B) acquitted, because summoning the
under the Singer-Byer contract.” Singer did not request physician or feeding the child would not
and did not get Byer’s consent to this transaction. On have prevented the child’s death from
February 2 the chairs were destroyed while in transit cancer.
in a derailment of the carrier’s railroad car. (C) convicted, because the father’s treatment
of his son showed reckless indifference to
371. In an action by Wheeler against Byer, Wheeler the value of life.
probably will recover (D) convicted, because the child would have
died from malnutrition had he not been
(A) $10,000, the contract price. afflicted with cancer.
(B) the difference between the contract price
and the market value of the chairs. 374. Vance had cheated Dodd in a card game.
(C) nothing, because the chairs had not been Angered, Dodd set out for Vance’s house with
delivered. the intention of shooting him. Just as he was
(D) nothing, because the Singer-Byer contract about to set foot on Vance’s property, Dodd
forbade an assignment. was arrested by a police officer who noticed
that Dodd was carrying a revolver. A statute in
the jurisdiction makes it a crime to “enter the
property of another with the intent to commit
any crime of violence thereon.”

88
If charged with attempting to violate the statute, I will begin work on April 5.” Ohner received
Dodd should be found this letter on March 22 but did not reply to it.
Plummer, without Ohner’s knowledge, began the
(A) not guilty, because the statute defines an work on April 5.
attempted crime and there cannot be an
attempt to attempt. Which of the following best characterizes the legal
(B) not guilty, because to convict him would relationship between Ohner and Plummer as of
be to punish him simply for having a guilty April 5?
mind.
(C) guilty, because he was close enough (A) A contract was formed on March 20 when
to entering the property and he had the Plummer posted his letter.
necessary state of mind. (B) A contract was formed on March 22 when
(D) guilty, because this is a statute designed to Ohner received Plummer’s letter.
protect the public from violence and Dodd (C) A contract was formed on April 5 when
was dangerous. Plummer began work.
(D) There was no contract between the parties
375. Congress enacted a statute providing that as of April 5.
persons may challenge a state energy law on
the ground that it is in conflict with the federal
Constitution in either federal or state court. Questions 377–378 are based on the following fact
According to this federal statute, any decision situation.
by a lower state court upholding a state energy
law against a challenge based on the federal On January 15, in a signed writing, Artisan agreed
Constitution may be appealed directly to the to remodel Ohner’s building according to certain
United States Supreme Court. specifications, and Ohner agreed to pay the specified
price of $5,000 to Artisan’s niece, Roberta Neese, as a
The provisions of this statute that authorize direct birthday present. Neese did not learn of the agreement
United States Supreme Court review of specified until her birthday on May 5.
decisions rendered by lower state courts are
Before they signed the writing, Artisan and Ohner had
(A) constitutional, because congressional control orally agreed that their “written agreement will be null
over questions of energy usage is plenary. and void unless Ohner is able to obtain a $5,000 loan
(B) constitutional, because Congress may from the First National Bank before January 31.”
establish the manner in which the appellate
jurisdiction of the United States Supreme 377. For this question only, assume that Ohner was
Court is exercised. unable to obtain the loan, and on January 31,
(C) unconstitutional, because they infringe on phoned Artisan and told him, “Don’t begin the
the sovereign right of states to have their work. The deal is off.” In an action for breach
supreme courts review decisions of their of contract brought against Ohner by the proper
lower state courts. party, will Ohner be successful in asserting as a
(D) unconstitutional, because under Article III defense his inability to obtain a loan?
of the U.S. Constitution the United States
Supreme Court does not have authority (A) Yes, because obtaining a loan was a
to review directly decisions of lower state condition precedent to the existence of an
courts. enforceable contract.
(B) Yes, because the agreement about
376. After several days of negotiations, Ohner wrote obtaining a loan is a modification of a
to Plummer: “Will pay you $3,000 if you will construction contract and is not required to
install new plumbing in my office building be in writing.
according to the specifications I have sent you. (C) No, because the agreement about obtaining
I must have your reply by March 30.” Plummer a loan contradicts the express and implied
replied by a letter that Ohner received on March terms of the writing.
15: “Will not do it for less than $3,500.” On (D) No, because Ohner is estopped to deny the
March 20, Plummer wrote to Ohner: “Have validity of the written agreement.
changed my mind. I will do the work for
$3,000. Unless I hear from you to the contrary,

89
378. For this question only, assume that Ohner and void.” Talbot objected to Andrews’s moving
obtained the loan, that Artisan completed the in, even if Andrews were to pay a part of the rent.
remodeling on May 1, and that on May 3, at
Artisan’s request, Ohner paid the $5,000 to When Andrews moved in, Talbot brought an
Artisan. If Neese learns of Ohner’s payment to appropriate action against Lane, Rogers, and
Artisan on May 5, at the same time she learns Andrews for a declaratory judgment that Rogers
of the written Artisan-Ohner contract, will she had no right to assign. Rogers’ defense was that
succeed in action against Ohner for $5,000? he and Talbot were tenants in common of a term
for years, and that he, Rogers, had a right to
(A) Yes, because she is an intended beneficiary assign a fractional interest in his undivided one-
of the written Artisan-Ohner contract. half interest. In this action, Talbot will
(B) Yes, because the written Artisan-Ohner
contract operated as an assignment to (A) prevail, because a cotenant has no right
Neese, and Artisan thereby lost whatever to assign all or any part of a leasehold
rights he may have had to the $5,000. without the consent of all interested
(C) No, because Neese had not furnished any parties.
consideration to support Ohner’s promise (B) prevail, because the lease provision
to pay $5,000 to her. prohibits assignment.
(D) No, because on May 3, Artisan and Ohner (C) not prevail, because he is not the
effectively modified their written contract, beneficiary of the nonassignment provision
thereby depriving Neese of whatever right in the lease.
she may have had under that contract. (D) not prevail, because his claim amounts to a
void restraint on alienation.

379. Dryden is on trial on a charge of driving while Questions 381–382 are based on the following fact
intoxicated. When Dryden was booked at the situation.
police station, a videotape was made that showed
him unsteady, abusive, and speaking in a slurred The owner of Newacre executed and delivered to a
manner. If the prosecutor lays a foundation power company a right-of-way deed for the building
properly identifying the tape, should the court and maintenance of an overhead power line across
admit it in evidence and permit it to be shown to Newacre. The deed was properly recorded. Newacre
the jury? then passed through several intermediate conveyances
until it was conveyed to Sloan about 10 years after
(A) Yes, because it is an admission. the date of the right-of-way deed. All the intermediate
(B) Yes, because its value is not substantially deeds were properly recorded, but none of them
outweighed by unfair prejudice. mentioned the right-of-way.
(C) No, because the privilege against self-
incrimination is applicable. Sloan entered into a written contract to sell Newacre
(D) No, because specific instances of conduct to Jones. By the terms of the contract, Sloan promised
cannot be proved by extrinsic evidence. to furnish an abstract of title to Jones. Sloan contracted
directly with Abstract Company to prepare and deliver
380. Talbot and Rogers, as lessees, signed a valid an abstract to Jones, and Abstract Company did so.
lease for a house. Lane, the landlord, duly The abstract omitted the right-of-way deed. Jones
executed the lease and delivered possession of delivered the abstract to his attorney and asked the
the premises to the lessees. attorney for an opinion as to title. The attorney signed
and delivered to Jones a letter stating that, from the
During the term of the lease, Rogers verbally attorney’s examination of the abstract, it was his
invited Andrews to share the house with the “opinion that Sloan had a free and unencumbered
lessees. Andrews agreed to pay part of the rent marketable title to Newacre.”
to Lane, who did not object to this arrangement,
despite a provision in the lease that provided that Sloan conveyed Newacre to Jones by a deed which
“any assignment, subletting, or transfer of any included covenants of general warranty and against
rights under this lease without the express written encumbrances. Jones paid the full purchase price.
consent of the landlord is strictly prohibited, null, After Jones had been in possession of Newacre for

90
more than a year, he learned about the right-of-way (C) the bus driver operating a free school
deed. Sloan, Jones, Abstract Company, and Jones’s bus service under the sponsorship of a
attorney were all without actual knowledge of the local church to refuse to allow African-
existence of the right-of-way at the time of the American pupils on the bus, solely because
conveyance from Sloan to Jones. of their race.
(D) the federal official in charge of distributing
381. If Jones sues Abstract Company for damages certain federal benefits directly to students
caused to Jones by the presence of the right-of- from distributing them to African-American
way, the most likely result will be a decision for pupils, solely because of their race.

(A) Jones, because Jones was a third-party


creditor beneficiary of the contract Questions 384–385 are based on the following fact
between Sloan and Abstract Company. situation.
(B) Jones, because the abstract prepared by
Abstract Company constitutes a guarantee Innes worked as a secretary in an office in a building
of Jones’s title to Newacre. occupied partly by her employer and partly by Glass,
(C) Abstract Company, because Abstract a retail store. The two areas were separated by walls
Company had no knowledge of the and were in no way connected, except that the air
existence of the right-of-way. conditioning unit served both areas and there was a
(D) Abstract Company, because there was no common return-air duct.
showing that any fraud was practiced upon
Jones. Glass began remodeling, and its employees did the
work, which included affixing a plastic surfacing
382. If Jones sues Sloan because of the presence of material to counters. To fasten the plastic to the
the right-of-way, the most likely result will be a counters, the employees purchased glue, with the
decision for brand name Stick, that was manufactured by Steel,
packaged in a sealed container by Steel, and retailed
(A) Jones, because Sloan is liable for his by Paint Company.
negligent misrepresentation.
(B) Jones, because the covenants in Sloan’s In the course of the remodeling job, one of Glass’
deed to Jones have been breached. employees turned on the air conditioning and caused
(C) Sloan, because Jones relied upon Abstract fumes from the glue to travel from Glass through
Company, not Sloan, for information the air conditioning unit and into Innes’ office. The
concerning title. employees did not know that there was common duct-
(D) Sloan, because Sloan was without work for the air conditioners. Innes was permanently
knowledge of any defects in the title to blinded by the fumes from the glue.
Newacre.
The label on the container of glue read, “DANGER.
Do not smoke near this product. Extremely flammable.
Contains butanone, toluol, and hexane. Use with
383. Congress enacts a criminal statute prohibiting adequate ventilation. Keep out of the reach of children.”
“any person from interfering in any way with
any right conferred on another person by the equal The three chemicals listed on the label are very toxic
protection clause of the Fourteenth Amendment.” and harmful to human eyes. Steel had received no
reports of eye injuries during the 10 years that the
Application of this statute to Jones, a private product had been manufactured and sold.
citizen, would be most clearly constitutional if
Jones, with threats of violence, coerces 384. If Innes asserts a claim against Paint Company,
the most likely result is that she will
(A) a public school teacher to exclude African-
American pupils from her class, solely (A) recover if she can recover against Steel.
because of their race. (B) recover, because Innes was an invitee of a
(B) African-American pupils, solely because tenant in the building.
of their race, to refrain from attending (C) not recover unless Paint Company was
a privately owned and operated school negligent.
licensed by the state. (D) not recover, because the glue came in a
sealed package.

91
385. If Innes asserts a claim against Glass, the most (C) not succeed, because the $1,000 for each
likely result is that she will “A” was promised only as a bonus.
(D) not succeed, because Esther was already
(A) recover, because a user of a product is held legally obligated to use her best efforts in
to the same standard as the manufacturer. law school.
(B) recover, because the employees of Glass
caused the fumes to enter her area of the
building. 388. In Polk’s negligence action against Dell arising
(C) not recover, because Glass used the glue out of a multiple-car collision, Witt testified
for its intended purposes. for Polk that Dell went through a red light. On
(D) not recover, because the employees of cross-examination, Dell seeks to question Witt
Glass had no reason to know that the about her statement that the light was yellow,
fumes could injure Innes. made in a deposition that Witt gave in a separate
action between Adams and Baker. The transcript
of the deposition is self-authenticating.

Questions 386–387 are based on the following fact On proper objection, the court should rule the
situation. inquiry

When Esther, Gray’s 21-year-old daughter, finished (A) admissible for impeachment only.
college, Gray handed her a signed memorandum (B) admissible as substantive evidence only.
stating that if she would go to law school for three (C) admissible for impeachment and as
academic years, he would pay her tuition, room, and substantive evidence.
board and would “give her a $1,000 bonus” for each (D) inadmissible, because it is hearsay, not
“A” she got in law school. Esther’s uncle, Miller, who within any exception.
was present on this occasion, read the memorandum
and thereupon said to Esther, “and if he doesn’t pay 389. In 1965 Hubert Green executed his will which in
your expenses, I will.” Gray paid her tuition, room, pertinent part provided, “I hereby give, devise,
and board for her first year but died just before the and bequeath Greenvale to my surviving widow
end of that year. Subsequently, Esther learned that for life, remainder to such of my children as
she had received two “A’s” in the second semester. shall live to attain the age of 30 years, but if any
The executor of Gray’s estate has refused to pay her child dies under the age of 30 years survived by
anything for the two “A’s” and has told her that the a child or children, such child or children shall
estate will no longer pay her tuition, room, and board take and receive the share which his, her, or
in law school. their parent would have received had such parent
lived to attain the age of 30 years.”
386. In an action by Esther against Miller on account
of the executor’s repudiation of Gray’s promise At the date of writing his will, Green was
to pay future tuition, room, and board, which married to Susan, and they had two children,
of the following would be Miller’s strongest Allan and Beth. Susan died in 1970 and Hubert
defense? married Waverly in 1972. At his death in 1980,
Green was survived by his wife, Waverly,
(A) The parties did not manifestly intend a and three children, Allan, Beth, and Carter.
contract. Carter, who was born in 1974, was his child by
(B) Gray’s death terminated the agreement. Waverly.
(C) The agreement was oral.
(D) The agreement was divisible. In a jurisdiction which recognizes the common
law Rule Against Perpetuities unmodified by
387. In an action against Gray’s estate for $2,000 on statute, the result of the application of the rule is
account of the two “A’s,” if the only defense that the
raised is lack of consideration, Esther probably
will (A) remainder to the children and to the
grandchildren is void because Green could
(A) succeed under the doctrine of promissory have subsequently married a person who
estoppel. was unborn at the time Green executed his
(B) succeed on a theory of bargained-for will.
exchange for her father’s promise.

92
(B) remainder to the children is valid, but the (C) Baker has a cause of action against Zeller
substitutionary gift to the grandchildren for Zeller’s failure to deliver 100 bushels
is void because Green could have of wheat.
subsequently married a person who was (D) Baker is obligated to give Zeller a
unborn at the time Green executed his will. reasonable time to attempt to obtain the
(C) gift in remainder to Allan and Beth or their other five bushels of wheat.
children is valid, but the gift to Carter or
his children is void. 392. A federal statute sets up a program of dental
(D) remainder to the children and the education. The statute provides that the Secretary
substitutionary gift to the grandchildren are of Health and Human Services “shall, on a current
valid. basis, spend all of the money appropriated for this
purpose” and “shall distribute the appropriated
390. Siddon worked as a private duty nurse and on funds” by a specified formula to state health
occasion worked in Doctors’ Hospital. The departments that agree to participate in the
hospital called Registry, the private duty referral program. In the current year Congress has
agency through which Siddon usually obtained appropriated $100 million for expenditure on
employment, and asked that in the future she this program.
not be assigned to patients in Doctors’ Hospital.
Registry asked the hospital why it had made In order to ensure a budget surplus in the current
the request. Doctors’ Hospital sent a letter to fiscal year, the President issues an executive
Registry giving as the reason for its request that order directing the various cabinet secretaries to
significant amounts of narcotics had disappeared cut expenditures in this year by 10 percent in all
during Siddon’s shift from the nursing stations at categories. He also orders certain programs to
which she had worked. be cut more drastically because he believes that
“they are not as important to the general welfare
If Siddon asserts a claim based on defamation as other programs.” The President identifies the
against Doctors’ Hospital, Siddon will dental education program as such a program and
orders it to be cut by 50 percent. Assume that no
(A) recover, because the hospital accused other federal statutes are relevant.
Siddon of improper professional conduct.
(B) recover if Siddon did not take the To satisfy constitutional requirements, how
narcotics. much money must the Secretary of Health
(C) not recover if narcotics disappeared during and Human Services distribute for the dental
Siddon’s shifts. education program this year?
(D) not recover if the hospital reasonably
believed that Siddon took the narcotics. (A) $50 million, because the President could
reasonably determine that this program is
391. Zeller contracted in writing to deliver to Baker not as important to the general welfare as
100 bushels of wheat on August 1 at $3.50 a other programs.
bushel. Because his suppliers had not delivered (B) $50 million, because as chief executive
enough wheat to him by that time, Zeller on the President has the constitutional
August 1 had only 95 bushels of wheat with authority to control the actions of all of his
which to fulfill his contract with Baker. subordinates by executive order.
(C) $90 million, because any more drastic cut
If Zeller tenders 95 bushels of wheat to Baker for the program would be a denial of equal
on August 1, and Baker refuses to accept or pay protection to beneficiaries of this program
for any of the wheat, which of the following best as compared to beneficiaries of other
states the legal relationship between Zeller and programs.
Baker? (D) $100 million, because the President may
not unilaterally suspend the effect of a
(A) Zeller has a cause of action against Baker, valid federal statute imposing a duty to
because Zeller has substantially performed spend appropriated monies.
his contract.
(B) Zeller is excused from performing his 393. Powers sued Debbs for battery. At trial,
contract because of impossibility of Powers’s witness Wilson testified that Debbs
performance. had made an unprovoked attack on Powers.

93
On cross-examination, Debbs asks Wilson 395. Which of the following is NOT a correct
about a false claim that Wilson once filed on an statement of the parties’ legal status immediately
insurance policy. The question is after Selco’s notice on June 10?

(A) proper, because the conduct involved (A) Byco has a cause of action for total breach
untruthfulness. of contract because of Selco’s repudiation,
(B) proper provided that the conduct resulted but that cause of action will be lost if
in conviction of Wilson. Selco retracts its repudiation before Byco
(C) improper, because the impeachment changes its position or manifests to Selco
involved a specific instance of misconduct. that Byco considers the repudiation final.
(D) improper, because the claim form would (B) Byco can bring suit to rescind the
be the best evidence. contract even if it elects to await Selco’s
performance for a commercially reasonable
time.
Questions 394–395 are based on the following fact (C) Byco can await performance by Selco for a
situation. commercially reasonable time, but if Byco
awaits performance beyond that period, it
On March 31, Selco and Byco entered into a written cannot recover any resulting damages that
agreement in which Selco agreed to fabricate and sell to it reasonably could have avoided.
Byco 10,000 specially designed brake linings for a new (D) Byco has a cause of action for breach of
type of power brake manufactured by Byco. The contract contract that it can successfully assert only
provided that Byco would pay half of the purchase after it has given Selco a commercially
price on May 15 in order to give Selco funds to “tool reasonable time to perform.
up” for the work; that Selco would deliver 5,000 brake
linings on May 31; that Byco would pay the balance
of the purchase price on June 15; and that Selco would
deliver the balance of the brake linings on June 30. 396. A state statute provides that persons moving into
a community to attend a college on a full-time
On May 10, Selco notified Byco that it doubted that basis may not vote in any elections for local or
it could perform because of problems encountered state officials that are held in that community.
in modifying its production machines to produce the Instead, the statute provides that for voting
brake linings. On May 15, however, Selco assured purposes all such persons shall retain their
Byco that the production difficulties had been residence in the communities from which they
overcome, and Byco paid Selco the first 50 percent came. In the state the age of majority is 18.
installment of the purchase price. Selco did not deliver
the first 5,000 brake linings on May 31, or at any time Which of the following is the strongest argument
thereafter; on June 10, Selco notified Byco that it to demonstrate the unconstitutionality of this
would not perform the contract. state statute?

394. Which of the following correctly states Byco’s (A) A state does not have an interest that
rights and obligations immediately after receipt is sufficiently compelling to justify the
of Selco’s notice on May 10? exclusion from voting of an entire class of
persons.
(A) Byco could treat the notice as an anticipatory (B) There are less restrictive means by which
repudiation, and had a cause of action on the state could assure that only actual
May 10 for breach of the entire contract. residents of a community vote in its
(B) Byco could treat the notice as an anticipatory elections.
repudiation, and could sue to enjoin an actual (C) Most persons moving to a community to
breach by Selco on May 31. attend college full time are likely to have
(C) Byco had no cause of action for breach of attained the age of majority under the laws
contract, but could suspend its performance of this state.
and demand assurances that Selco would (D) On its face this statute impermissibly
perform. discriminates against interstate commerce.
(D) Byco had no cause of action for breach
of contract, and was required to pay the
installment of the purchase price due on May
15 to preserve its rights under the contract.

94
397. At a time when Ogawa held Lot 1 in the (D) denied, because estoppel does not apply
Fairoaks subdivision in fee simple, Vine when the defendant is charged with
executed a warranty deed that recited that Vine violating two different statutes.
conveyed Lot 1, Fairoaks, to Purvis. The deed
was promptly and duly recorded. 399. Ellis, an electrical engineer, designed an
electronic game known as Zappo. Ellis entered
After the recording of the deed from Vine to into a licensing agreement with Toyco under
Purvis, Ogawa conveyed Lot 1 to Vine by a which Toyco agreed to manufacture Zappo
warranty deed that was promptly and duly according to Ellis’s specifications and to market
recorded. Later, Vine conveyed the property it and pay a royalty to Ellis.
to Rand by warranty deed and the deed was
promptly and duly recorded. Rand paid the fair Carla, whose parents had purchased a Zappo
market value of Lot 1 and had no knowledge of game for her, was injured while playing the
any claim of Purvis. game. Carla recovered a judgment against
Toyco on the basis of a finding that the Zappo
In an appropriate action, Rand and Purvis contest game was defective because of Ellis’s improper
title to Lot 1. In this action, judgment should be design.
for
In a claim for indemnity against Ellis, will
(A) Purvis, because Purvis’ deed is senior to Toyco prevail?
Rand’s.
(B) Rand, because Rand paid value without (A) Yes, because as between Ellis and Toyco,
notice of Purvis’ claim. Ellis was responsible for the design of
(C) Purvis or Rand, depending on whether a Zappo.
subsequent grantee is bound, at common (B) Yes, because Toyco and Ellis were joint
law, by the doctrine of estoppel by deed. tortfeasors.
(D) Purvis or Rand, depending on whether (C) No, because Toyco, as the manufacturer,
Purvis’ deed is deemed recorded in Rand’s was strictly liable to Carla.
chain of title. (D) No, if Toyco, by a reasonable inspection,
could have discovered the defect in the
398. Dillon held up a gasoline station. During design of Zappo.
the robbery he shot and killed a customer
who attempted to apprehend him. Dillon 400. While crossing Spruce Street, Pesko was hit by
was prosecuted for premeditated murder and a car that she did not see. Pesko sued Dorry for
convicted. Thereafter, he was indicted for armed her injuries.
robbery of the station.
At trial, Pesko calls Williams, a police officer,
Before the trial, his attorney moved to dismiss to testify that, 10 minutes after the accident, a
the indictment on the ground that further driver stopped him and said, “Officer, a few
proceedings were unconstitutional because of minutes ago I saw a hit-and-run accident on
Dillon’s prior conviction. Spruce Street involving a blue convertible,
which I followed to the drive-in restaurant at
The motion to dismiss should be Oak and Third,” and that a few seconds later
Williams saw Dorry sitting alone in a blue
(A) granted, because once Dillon was convertible in the drive-in restaurant’s parking
convicted on any of the charges arising lot.
out of the robbery, the prosecution was
constitutionally estopped from proceeding Williams’ testimony about the driver’s statement
against Dillon on any charge stemming should be
from the same transaction.
(B) granted, because the double jeopardy (A) admitted as a statement of recent
clause prohibits a subsequent trial on what perception.
is essentially a lesser included offense. (B) admitted as a present sense impression.
(C) denied, because there is no constitutional (C) excluded, because it is hearsay, not within
requirement that all known charges against any exception.
Dillon be brought in the same prosecution. (D) excluded, because it is more prejudicial
than probative.

95
Questions 401–402 are based on the following fact (C) No, because there was no separate
situation. consideration for Gritz’s promise.
(D) No, because such proof is barred by the
The Kernel Corporation, through its president, Statute of Frauds.
Demeter Gritz, requested from Vault Finance, Inc.,
a short-term loan of $100,000. On April 1, Gritz and
Vault’s loan officer agreed orally that Vault would
make the loan on the following terms: (1) The loan Questions 403–405 are based on the following fact
would be repaid in full on or before the following situation.
July 1 and would carry interest at an annual rate of 15
percent (a lawful rate under the applicable usury law); Adams, Bennett, and Curtis are charged in a common
and (2) Gritz would personally guarantee repayment. law jurisdiction with conspiracy to commit larceny.
The loan was approved and made on April 5. The only The state introduced evidence that they agreed to go
document evidencing the loan was a memorandum, to Nelson’s house to take stock certificates from a
written and supplied by Vault and signed by Gritz for safe in Nelson’s bedroom, that they went to the house,
Kernel, that read in its entirety: and that they were arrested as they entered Nelson’s
bedroom.
“April 5
Adams testified that he believed the stock certificates
In consideration of a loan advanced on this date, belonged to Curtis, that he thought Nelson was
Kernel Corporation hereby promises to pay improperly keeping them from Curtis, and that he
Vault Finance, Inc., $100,000 on September 1. went along to aid in retrieving Curtis’s property.

Kernel Corporation Bennett testified that he suspected Adams and Curtis


of being thieves and joined up with them in order
By /s/ Demeter Gritz to catch them. He also testified that he made an
Demeter Gritz, President” anonymous telephone call to the police alerting them
to the crime and that the call caused the police to be
Kernel Corporation did not repay the loan on or before waiting for them when they walked into Nelson’s
July 1, although it had sufficient funds to do so. On bedroom.
July 10, Vault sued Kernel as principal debtor and
Gritz individually as guarantor for $100,000, plus 15 Curtis did not testify.
percent interest from April 5.
403. If the jury believes Adams, it should find him
401. At the trial, can Vault prove Kernel’s oral
commitment to repay the loan on or before July 1? (A) guilty, because there was an agreement and
the entry into the bedroom is sufficient for
(A) Yes, because the oral agreement was the overt act.
supported by an independent consideration. (B) guilty, because good motives are not a
(B) Yes, because the evidence of the parties’ defense to criminal liability.
negotiations is relevant to their contractual (C) not guilty, because he did not have a
intent concerning maturity of the debt. corrupt motive.
(C) No, because such evidence is barred by the (D) not guilty, because he did not intend to
preexisting duty rule. steal.
(D) No, because such evidence contradicts the
writing and is barred by the parol evidence 404. If the jury believes Bennett, it should find him
rule.
(A) guilty, because there was an agreement and
402. At the trial, can Vault prove Gritz’s oral promise the entry into the bedroom is sufficient for
to guarantee the loan? the overt act.
(B) guilty, because he is not a police officer
(A) Yes, because Gritz signed the and thus cannot claim any privilege of
memorandum. apprehending criminals.
(B) Yes, because, as president of the debtor- (C) not guilty, because he did not intend to
company, Gritz is a third-party beneficiary steal.
of the loan. (D) not guilty, because he prevented the theft
from occurring.

96
405. If the jury believes both Adams and Bennett, it (D) lose, because a high-voltage power line is
should find Curtis a nuisance per se.

(A) guilty, because there was an agreement and 407. Pratt sued Danvers for injuries suffered by Pratt
the entry into the bedroom is sufficient for when their automobiles collided. At trial Pratt
the overt act. offers into evidence a properly authenticated
(B) guilty, because he intended to steal. letter from Danvers that says, “your claim seems
(C) not guilty, because a conviction would too high, but, because I might have been a little
penalize him for exercising his right not to negligent, I’m prepared to offer you half of what
be a witness. you ask.”
(D) not guilty, because Adams and Bennett did
not intend to steal. The letter is

(A) admissible as an admission by a party-


opponent.
406. O’Neal entered into a written contract to sell (B) admissible as a statement against pecuniary
her house and six acres known as Meadowacre interest.
to Perez for $75,000. Delivery of the deed and (C) inadmissible, because Danver’s statement
payment of the purchase price were to be made is lay opinion on a legal issue.
six months after the contract. The contract (D) inadmissible, because Danver’s statement
provided that Meadowacre was to be conveyed was made in an effort to settle the claim.
“subject to easements, covenants, and restrictions
of record.” The contract was not recorded. 408. Dobbs, while intoxicated, drove his car through
a playground crowded with children just to watch
After the contract was signed but before the the children run to get out of his way. His car
deed was delivered, Electric Company decided struck one of the children, killing her instantly.
to run a high-voltage power line in the area
and required an easement through a portion Which of the following is the best theory for
of Meadowacre. O’Neal, by deed, granted an finding Dobbs guilty of murder?
easement to Electric Company in consideration
of $5,000; the deed was duly recorded. (A) Transferred intent.
(B) Felony murder, with assault with a deadly
The power line would be a series of towers with weapon as the underlying felony.
several high-voltage lines that would be clearly (C) Intentional killing, since he knew that the
visible from the house on Meadowacre but children were there and he deliberately
would in no way interfere with the house. drove his car at them.
(D) Commission of an act highly dangerous
When Perez caused the title to Meadowacre to to life, without an intent to kill but with
be searched, the deed of easement to Electric disregard of the consequences.
Company was found. O’Neal appeared at the time
and place scheduled for the closing and proffered 409. Darden was prosecuted for armed robbery. At
an appropriate deed to Perez and demanded the trial, Darden testified in his own behalf, denying
purchase price. Perez refused to pay or accept that he had committed the robbery. On cross-
the deed. In an appropriate action for specific examination, the prosecutor intends to ask Darden
performance against Perez, O’Neal demanded whether he was convicted of burglary six years
$75,000. earlier.

In this action, O’Neal should The question concerning the burglary conviction is

(A) obtain an order for specific performance at (A) proper if the court finds that the probative
a price of $75,000. value for impeachment outweighs the
(B) obtain an order for specific performance at prejudice to Darden.
a price of $70,000. (B) proper, because the prosecutor is entitled
(C) lose, because Perez did not contract to to make this inquiry as a matter of right.
take subject to the easement to Electric (C) improper, because burglary does not
Company. involve dishonesty or false statement.

97
(D) improper, because the conviction must be (B) Yes, because an expert may base her
proved by court record, not by question on opinion on facts made known to her at the
cross-examination. trial.
(C) No, because she has no personal
410. Osif owned Broadacres in fee simple. For knowledge of Peel’s condition.
a consideration of $5,000, Osif gave Bard (D) No, because permanence of injury is an
a written option to purchase Broadacres for issue to be decided by the jury.
$300,000. The option was assignable. For a
consideration of $10,000, Bard subsequently 413. A federal criminal law makes it a crime for
gave an option to Cutter to purchase Broadacres any citizen of the United States not specifically
for $325,000. Cutter exercised his option. authorized by the President to negotiate with a
foreign government for the purpose of influencing
Bard thereupon exercised his option. Bard paid the foreign government in relation to a dispute
the agreed price of $300,000 and took title to with the United States. The strongest constitutional
Broadacres by deed from Osif. Thereafter, Cutter ground for the validity of this law is that
refused to consummate his purchase.
(A) under several of its enumerated powers,
Bard brought an appropriate action against Cutter Congress may legislate to preserve the
for specific performance, or, if that should be monopoly of the national government
denied, then for damages. Cutter counterclaimed over the conduct of United States foreign
for return of the $10,000. In this action the court affairs.
will (B) the President’s inherent power to negotiate
for the United States with foreign
(A) grant money damages only to Bard. countries authorizes the President, even
(B) grant specific performance to Bard. in the absence of statutory authorization,
(C) grant Bard only the right to retain the to punish citizens who engage in such
$10,000. negotiations without permission.
(D) require Bard to refund the $10,000 to (C) the law deals with foreign relations and
Cutter. therefore is not governed by the First
Amendment.
411. Congress enacts a statute punishing “each (D) federal criminal laws dealing with
and every conspiracy entered into by any two international affairs need not be as specific
or more persons for the purpose of denying as those dealing with domestic affairs.
persons housing, employment, or education,
solely because of their race.” Under which of 414. Arthur and Celia, brother and sister, both of
the following constitutional provisions is the legal age, inherited Goodacre, their childhood
authority of Congress to pass such a statute most home, from their father. They thereby became
clearly and easily justifiable? tenants in common.

(A) The obligation of contracts clause. Goodacre had never been used as anything
(B) The general welfare clause of Article I, §8. except a residence. Arthur had been residing on
(C) The Thirteenth Amendment. Goodacre with his father at the time his father
(D) The Fourteenth Amendment. died. Celia had been residing in a distant city.
After their father’s funeral, Arthur continued to
412. In Peel’s personal injury action, Wilson, a live on Goodacre, but Celia returned to her own
physician who had no previous knowledge of the residence.
matter, sat in court and heard all the evidence
about Peel’s symptoms and conditions. There was no discussion between Arthur and
Celia concerning their common ownership, nor
Wilson is called to give her opinion whether had there ever been any administration of their
Peel’s injuries are permanent. May Wilson so father’s estate. Arthur paid all taxes, insurance,
testify? and other carrying charges on Goodacre. He paid
no rent or other compensation to Celia, nor did
(A) Yes, provided she first identifies the data Celia request any such payment.
on which her opinion is based.

98
Thirty years later, a series of disputes arose (D) No, if the explosion was caused by
between Arthur and Celia for the first time negligent construction on Acme’s part.
concerning their respective rights to Goodacre.
The jurisdiction where the land is located 416. If Farber asserts a claim against Acme Company
recognizes the usual common law types for the loss of his lettuce crop, will Farber
of cotenancies, and there is no applicable prevail?
legislation on the subject.
(A) No, if Acme did not design the storage
If Arthur claims the entire title to Goodacre in facility.
fee simple and brings an action against Celia to (B) No, because Acme was an independent
quiet title in himself, and if the state where the contractor.
land is located has an ordinary 20-year adverse (C) Yes, because the operation of the storage
possession statute, the decision should be for facility was an abnormally dangerous
activity.
(A) Arthur, because during the past 30 years (D) Yes, if the explosion resulted from a defect
Arthur has exercised the type of occupancy of which Acme was aware.
ordinarily considered sufficient to satisfy
the adverse possession requirements.
(B) Arthur, because the acts of the parties
indicate Celia’s intention to renounce her 417. Johnson and Tenniel owned Brownacre as joint
right to inheritance. tenants with the right of survivorship. Johnson
(C) Celia, because there is no evidence that executed a mortgage on Brownacre to Lowden
Arthur has performed sufficient acts to in order to secure a loan. Subsequently, but
constitute her ouster. before the indebtedness was paid to Lowden,
(D) Celia, because one cotenant cannot acquire Johnson died intestate, with Stokes as her
title by adverse possession against another. only heir at law. The jurisdiction at which
Brownacre is located recognizes the title theory
of mortgages.
Questions 415–416 are based on the following fact
situation. In an appropriate action, the court should
determine that title to Brownacre is vested
Gasco owns a storage facility where flammable gases
are stored in liquified form under high pressure in (A) in Tenniel, with the entire interest subject
large spherical tanks. The facility was constructed to the mortgage.
for Gasco by Acme Company, a firm that specializes (B) in Tenniel, free and clear of the mortgage.
in the construction of such facilities. After the (C) half in Tenniel, free of the mortgage, and
facility had been in use for five years, an explosion half in Stokes, subject to the mortgage.
in the facility started a large fire that blanketed the (D) half in Tenniel and half in Stokes, with
surrounding countryside with a high concentration of both subject to the mortgage.
oily smoke and soot. Farber owns a large truck farm
near the facility. His entire lettuce crop was destroyed 418. Pursuant to a state statute, Clovis applied
by oil deposits left by the smoke. for tuition assistance to attend the Institute
of Liberal Arts. He was qualified for such
415. If Farber asserts a claim against Gasco for the assistance in every way except that he was a
loss of his lettuce crop and is unable to show resident alien who did not intend to become a
any negligence on the part of Gasco, will Farber United States citizen. The state’s restriction of
prevail? such grants to United States citizens or resident
aliens seeking such citizenship is probably
(A) Yes, because the operation of the storage
facility was an abnormally dangerous (A) valid, because aliens are not per se “a
activity. discrete and insular minority” specially
(B) Yes, because the intrusion of the smoke protected by the Fourteenth Amendment.
onto Farber’s farm amounted to a trespass. (B) valid, because the line drawn by the state
(C) No, if the explosion was caused by internal for extending aid was reasonably related to
corrosion that reasonable inspection a legitimate state interest.
procedures would not have disclosed.

99
(C) invalid, because the justifications for this from my share of the profits, if any, under the
restriction are insufficient to overcome the Vintage-Bouquet contract.” Amicusbank gave
burden imposed on a state when it uses prompt notice of this transaction to Vintage.
such an alienage classification.
(D) invalid, because the privileges and If Vintage thereafter refuses to account for any
immunities clause of Article IV does not profits to Amicusbank and Amicusbank sues
permit such an arbitrary classification. Vintage for Bouquet’s share of profits then
realized, Vintage’s strongest argument in defense
is that
Questions 419–421 are based on the following fact
situation. (A) the Bouquet-Vintage contract did not
expressly authorize an assignment of
A written contract was entered into between Bouquet, rights.
a financier-investor, and Vintage Corporation, a (B) Bouquet and Vintage are partners, not
winery and grape-grower. The contract provided that simply debtor and creditor.
Bouquet would invest $1,000,000 in Vintage for its (C) Amicusbank is not an assignee of
capital expansion and, in return, that Vintage, from Bouquet’s rights under the Bouquet-
grapes grown in its famous vineyards, would produce Vintage contract.
and market at least 500,000 bottles of wine each year (D) Amicusbank is not an intended third-
for five years under the label “Premium Vintage- party beneficiary of the Bouquet-Vintage
Bouquet.” contract.

The contract included provisions that the parties would 421. For this question only, assume the following
share equally the profits and losses from the venture facts. Soon after making its contract with
and that, if feasible, the wine would be distributed by Bouquet, Vintage, without Bouquet’s knowledge
Vintage only through Claret, a wholesale distributor of or assent, sold its vineyards but not its winery to
fine wines. Neither Bouquet nor Vintage had previously Agribiz, a large agricultural corporation. Under
dealt with Claret. Claret learned of the contract two the terms of this sale, Agribiz agreed to sell to
days later from reading a trade newspaper. In reliance Vintage all grapes grown on the land for five
thereon, he immediately hired an additional sales years. Agribiz’s employees have no experience
executive and contracted for enlargement of his wine in wine grape production, and Agribiz has
storage and display facility. no reputation in the wine industry as a grape
producer or otherwise. The Bouquet-Vintage
419. If Vintage refuses to distribute the wine through contract was silent on the matter of Vintage’s
Claret and Claret then sues Vintage for breach of selling any or all of its business assets.
contract, is it likely that Claret will prevail?
If Bouquet seeks an appropriate judicial remedy
(A) Yes, because Vintage’s performance was against Vintage for entering into the Vintage-
to run to Claret rather than to Bouquet. Agribiz transaction, is Bouquet likely to prevail?
(B) Yes, because Bouquet and Vintage could
reasonably foresee that Claret would (A) Yes, because the Vintage-Agribiz
change his position in reliance on the transaction created a significant risk of
contract. diminishing the profits in which Bouquet
(C) No, because Bouquet and Vintage did not would share under his contract with
expressly agree that Claret would have Vintage.
enforceable rights under their contract. (B) Yes, because the Bouquet-Vintage contract
(D) No, because Bouquet and Vintage, having did not contain a provision authorizing a
no apparent motive to benefit Claret, delegation of Vintage’s duties.
appeared in making the contract to have (C) No, because Vintage remains in a position
been protecting or serving only their own to perform under the Bouquet-Vintage
interests. contract.
(D) No, because Vintage, as a corporation,
420. For this question only, assume the following must necessarily perform its contracts by
facts. Amicusbank lent Bouquet $200,000 and delegating duties to individuals.
Bouquet executed a written instrument providing
that Amicusbank “is entitled to collect the debt

100
422. Which of the following is most likely to be were normally supervised by three attendants,
found to be a strict liability offense? but at this particular time, as Mary Weld knew,
there was only one attendant present to care for
(A) A city ordinance providing for a fine of not about 20 children of assorted ages.
more than $200 for shoplifting.
(B) A federal statute making it a felony to About 30 minutes later, while the attendant was
possess heroin. looking the other way, Bobby suddenly started
(C) A state statute making it a felony to fail to to cry. The attendant found him lying on his
register a firearm. back, picked him up, and called his mother. It
(D) A state statute making the sale of was later discovered that Bobby had suffered a
adulterated milk a misdemeanor. skull fracture.

423. Simmons and Boyd entered into a written If a claim is asserted against Dugan on Bobby’s
contract for the sale and purchase of Wideacre. behalf, will Bobby prevail?
The contract provided that “Simmons agrees
to convey a good and marketable title to Boyd (A) Yes, because Dugan owed the child the
60 days from the date of this contract.” The highest degree of care.
purchase price was stated as $60,000. (B) Yes, because a tw0-year-old is incapable
of contributory negligence.
At the time set for closing Simmons tendered (C) No, unless Dugan or his employees failed
a deed in the form agreed to in the contract. to exercise reasonable care to assure
Boyd’s examination of the record prior to the Bobby’s safety.
date of closing had disclosed, however, that (D) No, if Mary Weld assumed the risk by
the owner of record was not Simmons, but leaving Bobby in the nursery.
Olson. Further investigation by Boyd revealed
that, notwithstanding the state of the record, 425. Congress passes an act requiring that all owners
Simmons had been in what Boyd conceded as of bicycles in the United States register them
adverse possession for 15 years. The period of with a federal bicycle registry. The purpose
time to acquire title by adverse possession in the of the law is to provide reliable evidence of
jurisdiction is 10 years. Boyd refuses to pay the ownership to reduce bicycle theft. No fee is
purchase price or to take possession “because of charged for the registration. Although most
the inability of Simmons to transfer a marketable stolen bicycles are kept or resold by the thieves
title.” in the same cities in which the bicycles were
stolen, an increasing number of bicycles are
In an appropriate action by Simmons against being taken to cities in other states for resale.
Boyd for specific performance, Simmons will
Is this act of Congress constitutional?
(A) prevail, because he has obtained a “good
and marketable title” by adverse possession. (A) Yes, because Congress has the power to
(B) prevail, because Simmons’ action for regulate property for the general welfare.
specific performance is an action in rem (B) Yes, because Congress could determine
even though Olson is not a party. that in inseverable aggregates bicycle
(C) not prevail, because Boyd cannot be thefts affect interstate commerce.
required to buy a lawsuit even if the (C) No, because most stolen bicycles remain
probability is great that Boyd would within the state in which they were stolen.
prevail against Olson. (D) No, because the registration of vehicles is
(D) not prevail, because Simmons’ failure to a matter reserved to the states by the Tenth
disclose his lack of record title constitutes Amendment.
fraud.
426. Donaldson broke into Professor Ruiz’s office
424. When Mary Weld visited Dugan’s Alleys in order to look at examination questions.
to participate in the weekly bowling league The questions were locked in a drawer, and
competition held there, she brought her two- Donaldson could not find them. Donaldson
year-old son, Bobby, along and left him in a believed that looking at examination questions
nursery provided by Dugan for the convenience was a crime, but in this belief he was mistaken.
of his customers. The children in the nursery

101
Charged with burglary, Donaldson should be A statute imposes liability on the owner of an
automobile for injuries to a third party that are
(A) acquitted, because he did not complete the caused by the negligence of any person driving
crime and he has not been charged with the automobile with the owner’s consent. The
attempt. statute applies to situations of this kind, even
(B) acquitted, because what he intended to do if the owner did not specifically authorize the
when he broke in was not a crime. mechanic to test drive the car.
(C) convicted, because he had the necessary
mental state and committed the act of Placek sued Astin and Garrison jointly for
breaking and entering. damages arising from the accident. In that
(D) convicted, because factual impossibility is action, Astin crossclaims to recover from
not a defense. Garrison the amount of any payment Astin may
be required to make to Placek. The trier of fact
427. A statute of the state of Lanape flatly bans the has determined that the accident was caused
sale or distribution of contraceptive devices to solely by negligent driving on Garrison’s part,
minors. Drugs, Inc., a national retailer of drugs and that Placek’s damages were $100,000.
and related items, is charged with violating
the Lanape statute. Which of the following is In this action, the proper outcome will be that
the strongest constitutional argument Drugs,
Inc., could make in defending itself against (A) Placek should have judgment for $50,000
prosecution for violation of this statute? each against Astin and Garrison; Astin
should recover nothing from Garrison.
(A) The statute constitutes an undue burden on (B) Placek should have judgment for $100,000
interstate commerce. against Garrison only.
(B) The statute denies minors one of their (C) Placek should have judgment for $100,000
fundamental rights without due process. against Astin and Garrison jointly, and
(C) The statute denies Drugs, Inc., a privilege Astin should have judgment against
or immunity of state citizenship. Garrison for 50 percent of any amount
(D) The statute violates the First Amendment collected from Astin by Placek.
right to freedom of religion because it (D) Placek should have judgment for $100,000
regulates morals. against Astin and Garrison jointly, and
Astin should have judgment against
428. In a tort action, Fisher testified against Dawes. Garrison for any amount collected from
Dawes then called Jones, who testified that Astin by Placek.
Fisher had a bad reputation for veracity. Dawes
then also called Weld to testify that Fisher once 430. Martinez, a widower, owns in fee simple a
perpetrated a hoax on the police. ranch, Ranchacre. Martinez has one child,
Enrique, who is married. Enrique has one child,
Weld’s testimony is Ana Maria, who is also married but has no
children. In an effort to dispose of Ranchacre to
(A) admissible, provided that the hoax his descendants and to honor a request by Ana
involved untruthfulness. Maria that she be skipped in any disposition,
(B) admissible, provided that the hoax resulted Martinez conveys Ranchacre to his son, Enrique,
in conviction of Fisher. for life with the remainder to Ana Maria’s
(C) inadmissible, because it is merely children in fee simple.
cumulative impeachment.
(D) inadmissible, because it is extrinsic What interest, if any, is created in favor of
evidence of a specific instance of Ana Maria’s unborn children at the time of the
misconduct. conveyance?

429. Astin left her car at Garrison’s Garage to have (A) A contingent remainder.
repair work done. After completing the repairs, (B) A vested remainder subject to divestment.
Garrison took the car out for a test drive and was (C) A springing use.
involved in an accident that caused damages to (D) None.
Placek.

102
Questions 431–432 are based on the following fact reached over, picked up the five-dollar bill, and
situation. put it in his pocket. As he stood up to leave,
another customer who had seen him take the
On June 1, Kravat, a manufacturer of men’s neckties, money ran over to him and hit him in the face
received the following order from Clothier: “Ship 500 with her umbrella. Enraged, Dent choked the
two-inch ties, assorted stripes, your catalogue No. V34. customer to death.
Delivery by July l.”
Dent is charged with murder. He requests an
On June 1, Kravat shipped 500 three-inch ties that instruction allowing the jury to find him guilty
arrived at Clothier’s place of business on June 3. of voluntary manslaughter rather than murder.
Clothier immediately telegraphed Kravat: “Reject Dent’s request should be
your shipment. Order was for two-inch ties.” Clothier,
however, did not ship the ties back to Kravat. Kravat (A) granted, because the jury could find that
replied by telegram: “Will deliver proper ties before Dent acted recklessly and not with the
July 1.” Clothier received this telegram on June 4, but intent to cause death or serious bodily
did not reply to it. harm.
(B) granted, because the jury could find that
On June 30, Kravat tendered 500 two-inch ties in being hit in the face with an umbrella
assorted stripes, designated in his catalogue as item constitutes adequate provocation.
No. V34; but Clothier refused to accept them. (C) denied, because the evidence shows that
Dent intended to kill or to cause serious
431. Did Clothier properly reject the ties delivered on bodily harm.
June 3? (D) denied, because the evidence shows that
Dent provoked the assault on himself by
(A) Yes, because the ties were nonconforming his criminal misconduct.
goods.
(B) Yes, because Kravat did not notify 434. Dever drove his car into an intersection and
Clothier that the ties were shipped as an collided with a fire engine that had entered the
accommodation to Clothier. intersection from Dever’s right. The accident was
(C) No, because Kravat could accept Clothier’s caused by negligence on Dever’s part. As a result
offer by prompt shipment of either of the accident, the fire engine was delayed
conforming or nonconforming goods. in reaching Peters’ house, which was entirely
(D) No, because Clothier waived his right consumed by fire. Peters’ house was located about
to reject the ties by not returning them ten blocks from the scene of the accident.
promptly to Kravat.
If Peters asserts a claim against Dever, Peters
432. Did Clothier properly reject the ties tendered on will recover
June 30?
(A) the part of his loss that would have been
(A) Yes, because Kravat’s shipping the three- prevented if the collision had not occurred.
inch ties on June 1 was a present breach of (B) the value of his house before the fire.
contract. (C) nothing if Dever had nothing to do with
(B) Yes, because Kravat’s shipping the three- causing the fire.
inch ties on June 1 was an anticipatory (D) nothing, because Dever’s conduct did not
repudiation. create an apparent danger to Peters.
(C) No, because Kravat cured the June
1 defective delivery by his tender of 435. Miller applied to the state liquor board for transfer
conforming goods on June 30. of the license of Miller’s Bar and Grill to a new
(D) No, because a contract for the sale of goods site. The board held a hearing on the application.
can be modified without consideration.
At that hearing, Hammond appeared without
being subpoenaed and stated that Miller had
underworld connections. Although Hammond
433. Dent, while eating in a restaurant, noticed that did not know this information to be true, he had
a departing customer at the next table had left heard rumors about Miller’s character and had
a five-dollar bill as a tip for the waitress. Dent

103
noticed several underworld figures going in and ten-minute periods. Brown gave Young $100
out of Miller’s Bar and Grill. In fact, Miller had and went through the treatment.
no underworld connections.
Young is charged with obtaining money by false
In a claim against Hammond based on pretenses. Each of the following, if true, will
defamation, Miller will absolve Young of guilt for obtaining money by
false pretenses EXCEPT:
(A) not recover if Hammond reasonably
believed his statement to be true. (A) Young honestly believed that the device
(B) not recover if the board granted Miller’s would cure arthritis, but his belief was
application. unreasonable.
(C) recover, because Hammond’s statement (B) Brown honestly believed that the device
was false. would cure arthritis, but her belief was
(D) recover, because Hammond appeared unreasonable.
before the board voluntarily. (C) Young was playing a practical joke on
Brown and intended to return the money.
436. Santos agreed to sell and Perrine agreed to buy a (D) Brown was an undercover police officer
described lot on which a single-family residence and did not believe that the device would
had been built. Under the contract, Santos cure arthritis.
agreed to convey marketable title subject only to
conditions, covenants, and restrictions of record 438. Congress enacted a law prohibiting the killing,
and all applicable zoning laws and ordinances. capture, or removal of any form of wildlife upon
The lot was subject to a 10-foot sideline setback or from any federally owned land.
originally set forth in the developer’s duly
recorded subdivision plot. The applicable zoning Which of the following is the most easily
ordinance zones the property for single-family justifiable source of national authority for this
units and requires an 8.5-foot sideline setback. federal law?

Prior to closing, a survey of the property was (A) The commerce clause of Article I, § 8.
made. It revealed that a portion of Santos’s (B) The privileges and immunities clause of
house was 8.4 feet from the sideline. Article IV.
(C) The enforcement clause of the Fourteenth
Perrine refused to consummate the transaction on Amendment.
the ground that Santos’s title is not marketable. (D) The property clause of Article IV, § 3.
In an appropriate action, Santos seeks specific
performance. Who will prevail in such an action? 439. David is being tried in federal court for criminal
conspiracy with John to violate federal narcotics
(A) Santos, because any suit against Perrine law. At trial, the prosecutor calls David’s new
concerning the setback would be frivolous. wife, Wanda, and asks her to testify about
(B) Santos, because the setback violation falls a meeting between David and John that she
within the doctrine de minimis non curat lex. observed before she married David.
(C) Perrine, because any variation, however
small, amounts to a breach of contract. Which of the following is the most accurate
(D) Perrine, because the fact that Perrine may statement of the applicable rule concerning
be exposed to litigation is sufficient to whether Wanda may testify?
make the title unmarketable.
(A) The choice is Wanda’s.
437. Young, believing that Brown suffered from (B) The choice is David’s.
arthritis, told her that for $100 he could cure her (C) Wanda is permitted to testify only if both
with a device he had invented. The device was a Wanda and David agree.
large box with a series of electric light bulbs along (D) Wanda is compelled to testify even if both
the sides. Brown, after examining the device, Wanda and David object.
agreed to take the treatment, which consisted of
placing her hands inside the box for several 440. Opus, the owner of Stoneacre, entered into a
written agreement with Miner. Under this written
agreement, which was acknowledged and duly

104
recorded, Miner, for a five-year period, was Questions 442–443 are based on the following fact
given the privilege to enter on Stoneacre to situation.
remove sand, gravel, and stone in whatever
quantities Miner desired. Miner was to make Mater, a wealthy widow, wishing to make a substantial
monthly payments to Opus on the basis of the and potentially enduring gift to her beloved adult
amount of sand, gravel, and stone removed stepson Prodigal, established with Vault Savings and
during the previous month. Under the terms of Loan Association a passbook savings account by an
the agreement, Miner’s privilege was exclusive initial deposit of $10,000.
against all others except Opus, who reserved the
right to use Stoneacre to any purpose whatsoever, 442. For this question only, assume the following
including the removal of sand, gravel, and stone. facts. The passbook was issued solely in
Prodigal’s name; but Mater retained possession
One year after the agreement was entered into, of it and Prodigal was not then informed of the
the state brought a condemnation action to savings account. Subsequently, Mater became
take Stoneacre for a highway interchange. In disgusted with Prodigal’s behavior and decided
the condemnation action, is Miner entitled to to give the same savings account solely to her
compensation? beloved adult daughter Distaff. As permitted
by the rules of Vault Savings and Loan, Mater
(A) Yes, because he has a license, which is a effected this change by agreement with Vault.
property right protected by the due process This time she left possession of the passbook
clause. with Vault. Shortly thereafter, Prodigal learned
(B) Yes, because he has a profit à prendre, of the original savings account in his name and
which is a property right protected by the the subsequent switch to Distaff’s name.
due process clause.
(C) No, because he has a license, and licenses If Prodigal now sues Vault Savings and Loan
are not property rights protected by the due for $10,000 plus accrued interest, will the action
process clause. succeed?
(D) No, because he has a profit à prendre,
which is not a property right protected by (A) Yes, because Prodigal was a third-party
intended beneficiary of the original Mater-
the due process clause.
Vault deposit agreement.
(B) Yes, because Prodigal was a constructive
441. Dan was an alcoholic who frequently experienced assignee of Mater’s claim, as depositor, to
auditory hallucinations that commanded him to the savings account.
engage in bizarre and sometimes violent behavior. (C) No, because Prodigal never obtained
He generally obeyed these commands. The possession of the passbook.
hallucinations appeared more frequently when he (D) No, because Prodigal’s right, if any, to the
was intoxicated, but he sometimes experienced funds on deposit was effectively abrogated
them when he had not been drinking. by the second Mater-Vault deposit
agreement.
After Dan had been drinking continuously
for a three-day period, an elderly woman 443. For this question only, assume the following
began to reproach him about his drunken facts. The passbook was issued by Vault to
condition, slapping him on the face and Mater solely in her own name. That same
shoulders as she did so. Dan believed that he day, disinterested witnesses being present,
was being unmercifully attacked and heard the she handed the passbook to Prodigal and said,
hallucinatory voice telling him to strangle his “As a token of my love and affection for you,
assailant. He did so, and the woman died. I give you this $10,000 savings account.”
Shortly thereafter, she changed her mind and
If Dan is charged with second-degree murder, wrote Prodigal, “I hereby revoke my gift to
Dan’s best chance of acquittal would be to rely you of the $10,000 savings account with Vault
on a defense of Savings and Loan Association. Please return my
passbook immediately. Signed: Mater.” Prodigal
(A) intoxication. received the letter but ignored it, and Mater died
(B) lack of malice aforethought. unexpectedly a few days later.
(C) self-defense.
(D) insanity.

105
In litigation between Prodigal and Mater’s estate, (A) constitutional, because disagreements
which of the following is a correct statement of over federal grant-in-aid funds necessarily
the parties’ rights with respect to the money on involve federal questions within the
deposit with Vault? judicial power of the United States.
(B) constitutional, because the spending of
(A) The estate prevails, because Mater’s gift to federal monies necessarily includes the
Prodigal was revocable and was terminated authority to provide for the effective
by her death. settlement of disputes involving them.
(B) The estate prevails, because Mater’s gift to (C) unconstitutional, because it vests authority
Prodigal was revocable and was terminated in the federal court to determine a matter
by her express revocation. prohibited to it by the Eleventh Amendment.
(C) Prodigal prevails, because he took (D) unconstitutional, because it vests authority
Mater’s claim to the savings account by in a federal court to render an advisory
a gratuitous but effective and irrevocable opinion.
assignment from Mater.
(D) Prodigal prevails, because his failure to 446. Purvis purchased a used car from Daley, a used
reject the gift, even if the assignment was car dealer. Knowing that they were false, Daley
revocable, created an estoppel against made the following statements to Purvis prior to
Mater and her estate. the sale:

Statement 1. This car has never been


involved in an accident.
444. In a civil suit by Pine against Decker, Decker
called Wall, a chemist, as an expert witness Statement 2. This car gets 25 miles to the
and asked him a number of questions about his gallon on the open highway.
education and experience in chemistry. Over
Pine’s objection that Wall was not shown to be Statement 3. This is as smooth-riding a car
qualified in chemistry, the trial court permitted as you can get.
Wall to testify as to his opinion in response to a
hypothetical question. If Purvis asserts a claim against Daley based on
deceit, which of the false statements made by
On cross-examination, Pine asked Wall if he had Daley would support Purvis’ claim?
failed two chemistry courses while doing his
graduate work. The answer should be (A) Statement 1 only.
(B) Statement 2 only.
(A) admitted, because it is relevant to the (C) Statements 1 and 2 only.
weight to be given to Wall’s testimony. (D) Statements 2 and 3 only.
(B) admitted, because specific acts bearing
on truthfulness may be inquired about on 447. In a contract suit by Perez against Drake, each
cross-examination. of the following is an accepted method of
(C) excluded, because the court has determined authenticating Drake’s signature on a document
that Wall is qualified to testify as an offered by Perez EXCEPT:
expert.
(D) excluded, because Wall’s character has not (A) A non-expert who, in preparation for trial,
been put in issue. has familiarized himself with Drake’s
usual signature testifies that, in his opinion,
445. Congress enacts a law providing that all the questioned signature is genuine.
disagreements between the United States and (B) The jury, without the assistance of an
a state over federal grant-in-aid funds shall expert, compares the questioned signature
be settled by the filing of a suit in the federal with an admittedly authentic sample of
district court in the affected state. “The judgment Drake’s handwriting.
of that federal court shall be transmitted to the (C) A witness offers proof that the signature is
head of the federal agency dispensing such funds on a document that has been in existence
who, if satisfied that the judgment is fair and for at least 20 years, that was in a place
lawful, shall execute the judgment according to where it would be if it was authentic,
its terms.” This law is and that has no suspicious circumstances
surrounding it.

106
(D) A witness testifies that Drake admitted that Dunbar claims that Stone owed her $200 and that she
the signature is his. went to the drugstore to try to collect the debt. She
said that she asked Balcom to come along just in case
448. For a valuable consideration, Amato, the owner Stone made trouble but that she did not plan on using
of Riveracre, signed and gave to Barton a duly any force and did not know that Balcom was armed.
executed instrument that provided as follows:
“The grantor may or may not sell Riveracre 450. If Dunbar is prosecuted for murder on the basis
during her lifetime, but at her death, or if she of felony murder and the jury believes her claim,
earlier decides to sell, the property will be she should be found
offered to Barton at $500 per acre. Barton shall
exercise this right, if at all, within 60 days of (A) guilty, because her companion, Balcom,
receipt of said offer to sell.” Barton recorded the committed a homicide in the course of a
instrument. The instrument was not valid as a will. felony.
(B) guilty, because her taking Balcom with
Is Barton’s right under the instrument valid? her to the store created the risk of death
that occurred during the commission of a
(A) Yes, because the instrument is recorded. felony.
(B) Yes, because Barton’s right to (C) not guilty, because she did not know
purchase will vest or fail within the that Balcom was armed and thus did not
period prescribed by the Rule Against have the required mental state for felony
Perpetuities. murder.
(C) No, because Barton’s right to purchase is (D) not guilty, because she believed she was
a restraint on the owner’s power to make a entitled to the money and thus did not
testamentary disposition. intend to steal.
(D) No, because Barton’s right to purchase is
an unreasonable restraint on alienation. 451. If Dunbar is prosecuted for murder on the basis
of being an accessory to Balcom in committing
449. James and Mary Green were walking to their car a murder and the jury believes her claim, she
one evening after having seen a movie. As they should be found
were passing a dark alleyway, Daves leaped out
brandishing a gun. He pushed Mary against the (A) guilty, because in firing the shot Balcom
wall of a nearby building, held the gun to her was trying to help her.
head, and demanded money from James. James (B) guilty, because she and Balcom were
handed over his cash. Daves grabbed the cash acting in concert in a dangerous under-
and ran away. taking.
(C) not guilty, because she had no idea that
Which of the following, listed in descending Balcom was armed and she did not plan to
order of seriousness, is the most serious crime use force.
for which Daves may be convicted? (D) not guilty, because she was exercising self-
help and did not intend to steal.
(A) Robbery from James Green.
(B) Larceny from James Green.
(C) Assault on James and Mary Green.
(D) Assault on Mary Green. 452. The President of the United States recognizes the
country of Ruritania and undertakes diplomatic
relations with its government through the
Questions 450–451 are based on the following fact Secretary of State. Ruritania is governed by a
situation. repressive totalitarian government.

Dunbar and Balcom went into a drugstore, where In an appropriate federal court, Dunn brings
Dunbar reached into the cash register and took out a suit against the President and Secretary of
$200. Stone, the owner of the store, came out of a State to set aside this action on the ground
back room, saw what had happened, and told Dunbar that it is inconsistent with the principles of our
to put the money back. Balcom then took a revolver constitutional form of government. Dunn has
from under his coat and shot and killed Stone. a lucrative contract with the United States
Department of Commerce to provide commercial
information about Ruritania.

107
The contract expressly terminates, however, During presentation of Daly’s case, Daly
“when the President recognizes the country of called Wilson to testify that she is the roofing
Ruritania and undertakes diplomatic relations company’s personnel manager and that she
with its government.” had determined, by examining the company’s
employment records, that Walters had worked
Which of the following is the most proper there only three years. The trial judge should
disposition of the Dunn suit by the federal court? rule that Wilson’s testimony is

(A) Suit dismissed, because Dunn does not (A) inadmissible, because it is not the best
have standing to bring this action. evidence.
(B) Suit dismissed, because there is no (B) inadmissible, because it is impeachment on
adversity between Dunn and the a collateral question.
defendants. (C) admissible as evidence of a regularly
(C) Suit dismissed, because it presents a conducted activity.
nonjustifiable political question. (D) admissible as tending to impeach Walters’
(D) Suit decided on the merits. credibility.

453. Acorp and Beeco are companies that each 455. Andres conveyed Applewood Farm “to Bogatz,
manufacture pesticide X. Their plants are located her heirs and assigns, so long as the premises
along the same river. During a specific 24-hour are used for residential and farm purposes, then
period, each plant discharged pesticide into the to Cohen and his heirs.” The common law Rule
river. Both plants were operated negligently Against Perpetuities, unmodified by statute,
and such negligence caused the discharge of the is part of the law of the jurisdiction in which
pesticide into the river. Applewood Farm is located. As a consequence of
the conveyance, Cohen’s interest in Applewood
Landesmann operated a cattle ranch downstream Farm is
from the plants of Acorp and Beeco. Landesmann’s
cattle drank from the river and were poisoned by (A) nothing.
the pesticide. The amount of the discharge from (B) a valid executory interest.
either plant alone would not have been sufficient (C) a possibility of reverter.
to cause any harm to Landesmann’s cattle. (D) a right of entry for condition broken.

If Landesmann asserts a claim against Acorp


and Beeco, what, if anything, will Landesmann Questions 456–457 are based on the following fact
recover? situation.

(A) Nothing, because neither company Cycle Company manufactured a bicycle that it sold to
discharged enough pesticide to cause harm Bike Shop, a retail bicycle dealer, which in turn sold
to Landesmann’s cattle. it to Roth. Shortly thereafter, while Roth was riding
(B) Nothing, unless Landesmann can establish the bicycle along a city street, he saw a traffic light
how much pesticide each plant discharged. facing him turn from green to yellow. He sped up,
(C) One-half of Landesmann’s damages from hoping to cross the intersection before the light turned
each company. red. However, Roth quickly realized that he could not
(D) The entire amount of Landesmann’s do so and applied the brake, which failed. To avoid
damages, jointly and severally, from the the traffic that was then crossing in front of him, Roth
two companies. turned sharply to his right and onto the sidewalk,
where he struck Perez, a pedestrian. Both Perez and
454. Paulsen sued Daly for nonpayment of a personal Roth sustained injuries.
loan to Daly, as evidenced by Daly’s promissory
note to Paulsen. Paulsen called Walters to testify 456. If Roth asserts a claim against Bike Shop based
that he knows Daly’s handwriting and that on strict liability in tort, will Roth prevail?
the signature on the note is Daly’s. On direct
examination, to identify himself, Walters gave (A) Yes, if the brake failed because of a defect
his name and address and testified that he had present when the bicycle left the factory of
been employed by a roofing company for seven Cycle Company.
years. (B) Yes, because the brake failed while Roth
was riding the bicycle.

108
(C) No, if Roth contributed to his own injury $20,000 price upon delivery. In order to obtain
by speeding up. operating funds, Fido as borrower entered into a
(D) No, if Bike Shop carefully inspected the written loan agreement on October 5 with the High
bicycle before selling it. Finance Company. In relevant part, this agreement
recited, “Fido Factory hereby transfers and assigns to
457. If Perez asserts a claim based on negligence High Finance its (Fido Factory’s) October 1 mechanical
against Cycle Company and if it is found that dog contract with Toy Store, as security for a 50-day
the brake failure resulted from a manufacturing loan of $15,000, the advance and receipt of which are
defect in the bicycle, will Perez prevail? hereby acknowledged by Fido Factory.” No copy of
this agreement, or statement relating to it, was filed in
(A) Yes, because Cycle Company placed an office of public record.
a defective bicycle into the stream of
commerce. On October 15, Fido notified Toy Store, “We regret
(B) Yes, if the defect could have been to advise that our master shaft burned out last night
discovered through the exercise of because our night supervisor let the lubricant level get
reasonable care by Cycle Company. too low. We have just fired the supervisor, but the shaft
(C) No, because Perez was not a purchaser of cannot be repaired or replaced until about January 1.
the bicycle. We can guarantee delivery of your order, however,
(D) No, if Roth was negligent in turning onto not later than January 20.” Toy Store rejected this
the sidewalk. proposal as unacceptable and immediately contracted
with the only other available manufacturer to obtain

the 1,000 dogs at $30 per unit by November 15.
458. Damson was short of money. He decided to
go into Winston’s house to take Winston’s 459. For this question only, assume that on November
silverware and then to sell it. That night, while 1, Toy Store sues Fido for damages and alleges
Winston was away, Damson entered by picking the above facts, except those relating to the
the lock on the front door. He picked up a chest Fido-High Finance loan agreement. Upon Fido’s
of silverware from the dining room and went out motion to dismiss the complaint, the court
the front door of the house to his car. As he was should
putting the chest of silverware into the trunk, he
had second thoughts and decided that he did not (A) sustain the motion, because Fido on
wish to become a thief. He reentered the house October 15 stated its willingness, and gave
and replaced the chest of silverware where he assurance of its ability, to perform the
had found it. As he came out of the house the contract in January.
second time, he was arrested by the police, who (B) sustain the motion, because Toy Store’s
had been called by a neighbor. lawsuit is premature in any case until after
November 15.
Damson is (C) deny the motion, because Toy Store’s
complaint alleges an actionable tort by
(A) guilty of burglary and larceny. Fido.
(B) guilty of burglary and attempted larceny. (D) deny the motion, because Toy Store’s
(C) guilty of burglary but not guilty of any complaint alleges an actionable breach of
larceny offense. contract by Fido.
(D) not guilty of burglary or any larceny
offense. 460. For this question only, assume that by November
16, Fido, without legal excuse, has delivered no
dogs, and that Toy Store has brought an action
Questions 459–460 are based on the following fact against Fido. In an action brought on November
situation. 16 by Toy Store against High Finance Company
on account of Fido’s default, Toy Store can
On October 1, Toy Store, Inc., entered into a written recover
contract with Fido Factory, Inc., for the purchase at
$20 per unit of 1,000 mechanical dogs, to be specially (A) nothing, because the October 5 assignment
manufactured by Fido according to Toy Store’s by Fido to High Finance of Fido’s contract
specifications. Fido promised to deliver all of the dogs with Toy Store was only an assignment for
“not later than November 15, for the Yule shopping security.
season,” and Toy Store promised to pay the full

109
(B) nothing, because no record of the October applied to his admitted conduct. In light of
5 transaction between Fido and High the nature of the rights involved, which of
Finance was publicly filed. the following is the most probable burden of
(C) $10,000 in damages, because Toy Store persuasion on this constitutional issue?
was a third-party intended beneficiary of
the October 5 transaction between Fido (A) The state has the burden of persuading the
and High Finance. court that the application of this statute
(D) $10,000 in damages, because the October to Zeller is necessary to vindicate an
5 transaction between Fido and High important state interest.
Finance effected, with respect to Toy Store (B) The state has the burden of persuading the
as creditor, a novation of debtors. court that the application of this statute to
Zeller is rationally related to a legitimate
state interest.
(C) Zeller has the burden of persuading the
461. A 10-lot subdivision was approved by the proper court that the application of this statute
governmental authority. The authority’s action to him is not necessary to vindicate an
was pursuant to a map filed by Diaz, which important state interest.
included an undesignated parcel in addition to (D) Zeller has the burden of persuading the
the 10 numbered lots. The undesignated parcel is court that the application of this statute to
differently shaped and somewhat larger than any him is not rationally related to a legitimate
one of the numbered lots. Subdivision building state interest.
restrictions were imposed on “all the lots shown
on said map.” 463. According to a statute of the state of Kiowa, a
candidate for state office may have his name
Diaz contracts to sell the unnumbered lot, placed on the official election ballot only if
described by metes and bounds, to Butts. Is title he files with the appropriate state official a
to the parcel marketable? petition containing a specified number of voter
signatures. Roderick failed to get his name
(A) Yes, because the undesignated parcel is placed on the state ballot as an independent
not a lot to which the subdivision building candidate for governor because he failed to file
restrictions apply. a petition with the number of voter signatures
(B) Yes, because the undesignated parcel is not required by state statute. In a suit against the
part of the subdivision. appropriate state officials in federal district
(C) No, because the undesignated parcel court, Roderick sought an injunction against the
has never been approved by the proper petition signature requirement on the ground that
governmental authority. it was unconstitutional.
(D) No, because the map leaves it uncertain as
to whether the unnumbered lot is subject to Which of the following, if established, constitutes
the building restrictions. the strongest argument for Roderick?

462. A state statute requires the permanent removal (A) Compliance with the petition signature
from parental custody of any child who has requirement is burdensome.
suffered “child abuse.” That term is defined to (B) The objectives of the statute could be
include “corporal punishment of any sort.” satisfactorily achieved by less burdensome
means.
Zeller very gently spanks his six-year-old son on (C) Because of the petition signature
the buttocks whenever he believes that spanking requirement, very few independent
is necessary to enforce discipline on him. Such a candidates have ever succeeded in getting
spanking occurs not more than once a month and on the ballot.
has never physically harmed the child. (D) The motivation for the statute was a desire
to keep candidates off the ballot if they did
The state files suit under the statute to terminate not have strong support among voters.
Zeller’s parental rights solely because of these
spankings. Zeller defends only on the ground 464. In March, when Ohm was 17, Stereo delivered
that the statute in question is unconstitutional as to Ohm a television set. At that time Ohm agreed
in writing to pay $400 for the set on July 1 when
he would reach his 18th birthday. Eighteen is

110
the applicable statutory age of majority, and on shifting weight of the beef caused the trailer to
that date Ohm was to receive the proceeds of a overturn. Morris was unable to avoid a collision with
trust. On July 1, when the reasonable value of the overturned trailer and was injured.
the television set was $250, Ohm sent Stereo a
signed letter stating, “I’ll only pay you $300; The trailer had been manufactured by Trailco. A
that is all the set is worth.” number of truckers had complained to Trailco that the
design of the trailer, which allowed the load to swing
In an action against Ohm for money damages on freely, was dangerous. Dixon knew of the dangerous
July 2, what is the maximum amount that Stereo propensity of the trailer. A restraining device that
will be entitled to recover? could be installed in the trailer would prevent the load
from shifting and was available at nominal cost. Dixon
(A) Nothing knew of the restraining device but had not installed it.
(B) $250, the reasonable value of the set
(C) $300, the amount Ohm promised to pay in 467. If Morris asserts a claim based on strict liability
his letter of July 1 tort against Trailco, she will
(D) $400, the original sale price
(A) recover unless Morris was negligently
465. Dray was prosecuted for bank robbery. At trial, driving when the truck overturned.
the bank teller, Wall, was unable to identify (B) recover, because Dixon’s knowledge of the
Dray, now bearded, as the bank robber. The dangerous propensity of the trailer does not
prosecutor then showed Wall a group of relieve Trailco of liability.
photographs, and Wall testified that she had (C) not recover, because there was no privity
previously told the prosecutor that the middle of contract between Morris and Trailco.
picture (concededly a picture of Dray before he (D) not recover if Dixon was negligent in
grew a beard) was a picture of the bank robber. failing to install the restraining device in
the trailer.
Wall’s testimony is
468. If Morris asserts a claim for her injuries against
(A) inadmissible, because it is hearsay, not Dixon, she will
within any exception.
(B) inadmissible, because it is a violation of (A) prevail if the use of a restraining device
Dray’s right of confrontation. would have prevented the trailer from
(C) admissible as prior identification by the overturning.
witness. (B) prevail, because Dixon is strictly liable to
(D) admissible as past recollection recorded. Morris for injuries resulting from defects
in the trailer.
466. Which of the following is LEAST likely to be (C) not prevail unless Dixon was driving in a
the underlying felony in a prosecution for felony negligent manner at the time Morris was
murder? injured.
(D) not prevail, because Dixon was not the
(A) Arson. manufacturer or seller of the trailer.
(B) Manslaughter.
(C) Attempted rape.
(D) Burglary.
469. Fernwood Realty Company developed a
residential development, known as the Fernwood
Questions 467–468 are based on the following fact Development, which included single-family
situation. dwellings, townhouses, and high-rise apartments
for a total of 25,000 dwelling units. Included
Morris was driving north on an interstate highway in the deed to each unit was a covenant under
at about 50 miles per hour when a tractor-trailer rig, which the grantee and the grantee’s “heirs and
owned and driven by Dixon, passed her. The tractor assigns” agreed to purchase electrical power
was pulling a refrigerated meat trailer fully loaded from only a plant Fernwood promised to build
with beef carcasses hanging freely from the trailer and maintain within the development. Fernwood
ceiling. When Dixon cut back in front of Morris, the constructed the plant and the necessary power

111
lines. The plant did not supply power outside the (C) unconstitutional as an undue delegation of
development. An appropriate and fair formula legislative power to the executive.
was used to determine price. (D) unconstitutional, because it violates the due
process clause of the Fifth Amendment.
After constructing and selling 12,500 of
the units, Fernwood sold its interest in the 471. Ortega owned Blackacre in fee simple and by his
development to Gaint Realty Investors. Gaint will specifically devised Blackacre as follows:
operated the power plant and constructed “To my daughter, Eugenia, her heirs and assigns,
and sold the remaining 12,500 units. Each but if Eugenia dies survived by a husband and
conveyance from Gaint contained the a child or children, then to Eugenia’s husband
same covenant relating to electrical power during his lifetime with remainder to Eugenia’s
that Fernwood had included in the 12,500 children, their heirs and assigns. Specifically
conveyances it had made. provided, however, that if Eugenia dies
survived by a husband and no child, Blackacre
Page bought a dwelling unit from Olm, who is specifically devised to my nephew, Luis, his
had purchased it from Fernwood. Subsequently, heirs and assigns.”
Page, whose lot was along the boundary of the
Fernwood Development, ceased buying electrical While Ortega’s will was in probate, Luis
power from Gaint and began purchasing power quitclaimed all interest in Blackacre to Eugenia’s
from General Power Company, which provided husband, José. Three years later, Eugenia died,
such service in the area surrounding the survived by José but no children. Eugenia left a
Fernwood Development. Both General Power will devising her interest in Blackacre to José.
and Gaint have governmental authorization to The only applicable statute provides that any
provide electrical services to the area. Gaint interest in land is freely alienable.
instituted an appropriate action against Page to
enjoin her from obtaining electrical power from Luis instituted an appropriate action against José
General Power. If judgment is for Page, it most to establish title to Blackacre. Judgment should
likely will be because be for

(A) the covenant does not touch and concern (A) Luis, because his quitclaim deed did not
the land. transfer his after-acquired title.
(B) the mixture of types of residential units (B) Luis, because José took nothing under
is viewed as preventing one common Ortega’s will.
development scheme. (C) José, because Luis had effectively
(C) the covenant is a restraint on alienation. conveyed his interest to José.
(D) there is no privity of estate between Page (D) José, because the doctrine of after-acquired
and Gaint. title applies to a devise by will.

470. Congress passes an Energy Conservation 472. Duncan was charged with aggravated assault. At
Act. The act requires all users of energy in trial Duncan did not testify; however, he sought
this country to reduce their consumption by a to offer opinion evidence of his good character
specified percentage, to be set by a presidential for truth and veracity.
executive order. The act sets forth specific
standards the President must use in setting This testimony should be
the percentage and detailed procedures to be
followed. (A) admitted, because a criminal defendant
is entitled to offer evidence of his good
The provision that allows the President to set the character.
exact percentage is probably (B) admitted, because a party’s credibility is
necessarily in issue.
(A) constitutional, because it creates a limited (C) excluded, because character is not
administrative power to implement the admissible to prove conduct in conformity
statute. therewith.
(B) constitutional, because inherent executive (D) excluded, because it is evidence of a trait
powers permit such action even without not pertinent to the case.
statutory authorization.

112
473. Alford was a suspect in a homicide committed (A) this law is a necessary and proper means of
during a robbery of a liquor store. Barber was a protecting United States property.
friend of Alford. Police telephoned Barber and (B) the animals are moving in the stream of
asked if he would help locate Alford. Barber interstate commerce.
agreed and met the police officers at headquarters (C) the police powers of the federal
later that night. government encompass protection of wild
animals.
After a discussion during which police asked (D) shooting wild animals is a privilege, not a
questions about Alford and the homicide, Barber right.
said that he wanted to get something “off his
chest” and advised the officers that he was in on
the robbery but that Alford had shot the owner Questions 475–477 are based on the following fact
of the store without his permission or prior situation.
knowledge. The officers then for the first time
gave Barber his Miranda warnings. Poe ordered some merchandise from Store. When the
merchandise was delivered, Poe decided that it was
Barber was indicted for felony murder. He not what he had ordered, and he returned it for credit.
moved to prevent the introduction of his Store refused to credit Poe’s account, continued to
statement into evidence. His motion should be bill him, and, after 90 days, turned the account over to
Kane, a bill collector, for collection.
(A) granted, because Barber was effectively
in custody and entitled to receive Kane called at Poe’s house at 7 p.m. on a summer
Miranda warnings at the beginning of the evening while many of Poe’s neighbors were seated
discussion. on their porches. When Poe opened the door, Kane,
(B) granted, because Barber’s rights to counsel who was standing just outside the door, raised an
and to due process were violated by the electrically amplified bullhorn to his mouth. In a voice
interrogation at police headquarters. that could be heard a block away, Kane called Poe a
(C) denied, because his statement was freely “deadbeat” and asked him when he intended to pay his
and voluntarily given and he was not bill to Store.
entitled to Miranda warnings.
(D) denied, because by visiting headquarters Poe, greatly angered, slammed the door shut. The
voluntarily, Barber waived his right to door struck the bullhorn and jammed it forcibly
receive Miranda warnings at the beginning against Kane’s face. As a consequence, Kane lost his
of the discussion. front teeth.

474. The federal government has complete jurisdiction 475. If Poe asserts a claim based on defamation
over certain parkland located within the state against Kane, will Poe prevail?
of Plains. To conserve the wildlife that inhabits
that land, the federal government enacts a statute (A) Yes, if Kane’s remarks were heard by any
forbidding all hunting of animals in the federal of Poe’s neighbors.
park. That statute also forbids the hunting of (B) Yes, because Kane’s conduct was extreme
animals that have left the federal park and have and outrageous.
entered the state of Plains. (C) No, unless Kane knew that Poe owed no
money to Store.
Hanson has a hunting license from the state of (D) No, unless Poe suffered some special
Plains authorizing him to hunt deer anywhere damage.
in the state. On land within the state of Plains
located adjacent to the federal park, Hanson 476. If Poe asserts a claim based on intentional
shoots a deer he knows has recently left the infliction of emotional distress against Kane,
federal land. will Poe prevail?

Hanson is prosecuted for violating the federal (A) Yes, because Kane’s conduct was extreme
hunting law. The strongest ground supporting and outrageous.
the constitutionality of the federal law forbidding (B) Yes, because Kane was intruding on Poe’s
the hunting of wild animals that wander off property.
federal property is that (C) No, unless Poe suffered physical harm.

113
(D) No, if Poe still owed Store for the (A) Eureka, as inventor of the LBVC, was not
merchandise. a merchant.
(B) the invoice restriction was a material
477. If Kane asserts a claim of battery against Poe, alteration of preexisting terms.
will Kane prevail? (C) Eureka’s written reply that quoted $39.99
per LBVC but did not contain a restriction
(A) Yes, because Poe had not first asked Kane on retail sales, was not an offer that
to leave the property. Gourmet accepted by ordering 24 LBVCs.
(B) Yes, if Poe knew that the door was (D) Gourmet was consciously aware when
substantially certain to strike the bullhorn. taking delivery of the goods that the
(C) No, if Kane’s conduct triggered Poe’s television ad had said, “Not available in
response. stores.”
(D) No, because Kane was an intruder on
Poe’s property.

480. Downs was indicted in state court for bribing


a public official. During the course of the
Questions 478–479 are based on the following fact investigation, police had demanded and received
situation. from Downs’s bank the records of Downs’s
checking account for the preceding two years.
Eureka, Inc., inventor of the LBVC, a laser-beam The records contained incriminating evidence.
vegetable chopper, ran a television ad that described
the chopper and said, “The LBVC is yours for only On the basis of a claim of violation of his
$49.99 if you send your check or money order to Box constitutional rights, Downs moves to prevent
007, Greenville. Not available in stores.” Gourmet, the introduction of the records in evidence. His
who owned a retail specialty shop, wrote Eureka, motion should be
“What’s your best firm price for two dozen LBVCs?”
Eureka sent a written reply that said in its entirety, (A) granted, because a search warrant should
“We quote you for prompt acceptance $39.99 per unit have been secured for seizure of the
for 24 LBVCs.” Gourmet subsequently mailed a check records.
to Eureka in the appropriate amount, with a memo (B) granted, because the records covered such
enclosed saying, “I accept your offer for 24 LBVCs.” an extensive period of time that their
seizure unreasonably invaded Downs’s
478. A contract would arise from these right of privacy.
communications only if (C) denied, because the potential destructibility
of the records, coupled with the public
(A) both parties were merchants. interest in proper enforcement of the
(B) Eureka had at least 24 LBVCs in stock criminal laws, created an exigent situation
when Gourmet’s check and memo were justifying the seizure.
received. (D) denied, because the records were business
(C) Gourmet’s check and memo were mailed records of the bank in which Downs had
within three months after his receipt of no legitimate expectation of privacy.
Eureka’s letter.
(D) Gourmet’s check and memo were mailed
within a reasonable time after his receipt of Questions 481–482 are based on the following fact
Eureka’s letter. situation.

479. For this question only, assume the following Three states, East Winnetka, Midland, and West
facts: Eureka shipped 24 LBVCs to Gourmet Hampton, are located next to one another in that
after receiving his check and memo, and with order. The states of East Winnetka and West Hampton
the shipment sent Gourmet an invoice that permit the hunting and trapping of snipe, but the state
conspicuously stated, among other things, the of Midland strictly forbids these activities in order to
following lawful provision: “These items shall protect snipe, a rare species of animal, from extinction.
not be offered for resale at retail.” Gourmet The state of Midland has a state statute that provides,
received and read but disregarded the invoice “Possession of snipe traps is prohibited. Any game
restriction and displayed the 24 LBVCs for warden finding a snipe trap within the state shall seize
resale. Eureka has a cause of action against and destroy it.” Snipe traps cost about $15 each.
Gourmet for breach of contract only if

114
Prentis is a resident of West Hampton and an ardent (B) The federal rule preempts the Midland
snipe trapper. She drove her car to East Winnetka state statute, because the federal rule does
to purchase a new improved snipe trap from a not contain affirmative authorization for
manufacturer there. In the course of her trip back continued state regulation.
across Midland with the trap in her car, Prentis (C) The federal rule does not preempt the
stopped in a Midland state park to camp for a few Midland state statute, because the Midland
nights. While she was in that park, a Midland game state statute regulates wild animals, a field
warden saw the trap, which was visible on the front of exclusive state power.
seat of her car. The warden seized the trap and (D) The federal rule does not preempt the
destroyed it in accordance with the Midland statute Midland state statute, because the purposes
after Prentis admitted that the seized item was a of the federal rule and the Midland state
prohibited snipe trap. No federal statutes or federal statute are different.
administrative regulations apply.

481. For this question only, assume that Prentis
demonstrates that common carriers are permitted Questions 483–484 are based on the following fact
to transport snipe traps as cargo across Midland situation.
for delivery to another state and that in practice
the Midland statute is enforced only against Orris had title to Brownacre in fee simple. Without
private individuals transporting those traps Orris’ knowledge, Hull entered Brownacre in 1950
in private vehicles. If Prentis challenges the and constructed an earthen dam across a watercourse.
application of the Midland statute to her on the The earthen dam trapped water that Hull used to water
basis only of a denial of equal protection, the a herd of cattle he owned. After 12 years of possession
application of the statute will probably be found of Brownacre, Hull gave possession of Brownacre to
Burns. At the same time, Hull also purported to transfer
(A) constitutional, because the traps constitute his cattle and all his interests in the dam and water to
contraband in which Prentis could have no Burns by a document that was sufficient as a bill of
protected property interest. sale to transfer personal property but was insufficient
(B) constitutional, because there is a rational as a deed to transfer real property.
basis for differentiating between the
possession of snipe traps as interstate cargo One year later, Burns entered into a lease with Orris to
by common carriers and the possession of lease Brownacre for a period of five years. After the
snipe traps by private individuals. end of the five-year term of the lease, Burns remained
(C) unconstitutional, because the state cannot on Brownacre for an additional three years and then
demonstrate a compelling public purpose left Brownacre. At that time Orris conveyed Brownacre
for making this differentiation between by a quitclaim deed to Powell. The period of time to
common carriers and such private acquire title by adverse possession in the jurisdiction
individuals. is 10 years.
(D) unconstitutional, because interstate travel
is a fundamental right that may not be 483. After Orris’s conveyance to Powell, title to
burdened by state law. Brownacre was in
482. For this question only, assume that a valid (A) Hull.
federal administrative rule, adopted under a (B) Orris.
federal consumer product safety act, regulates (C) Burns.
the design of snipe traps. The rule was issued (D) Powell.
to prevent traps from causing injury to human
beings, e.g., by pinching fingers while persons 484. After Orris’s conveyance to Powell, title to the
were setting the traps. No other federal law earthen dam was in
applies. Which of the following best states the
effect of the federal rule on the Midland state (A) the person who then held title to
statute? Brownacre in fee simple.
(B) Burns, as purchaser of the dam under the
(A) The federal rule preempts the Midland bill of sale.
state statute, because the federal rule (C) the person who then owned the water
regulates the same subject matter: snipe rights as an incident thereto.
traps.

115
(D) Hull, as the builder of the dam. take the deed right back.” Marsh recovered and
left the hospital, but shortly thereafter, before
any other relevant event, Simon died intestate.

485. An issue in Parker’s action against Daves for Marsh brought an appropriate action against
causing Parker’s back injury was whether Simon’s heirs to determine title.
Parker’s condition had resulted principally from
a similar occurrence five years before, with If Marsh wins, it will be because
which Daves had no connection.
(A) the court will impose a constructive trust
Parker called Watts, his treating physician, who to carry out the intent of the deceased son.
offered to testify that when she saw Parker after (B) the presumption of delivery arising from
the latest occurrence, Parker told her that before the recording is not valid unless the
the accident he had been working full time, grantee has knowledge at the time of the
without pain or limitation of motion, in a job recording.
that involved lifting heavy boxes. (C) Simon’s declaration was a constructive
reconveyance of the land.
Watts’s testimony should be (D) there was no effective acceptance of
delivery of the deed.
(A) admitted, because it is a statement of
Parker’s then existing physical condition. 488. Plummer, a well-known politician, was scheduled
(B) admitted, because it is a statement made to address a large crowd at a political dinner. Just
for purposes of medical diagnosis or as he was about to sit down at the head table,
treatment. Devon pushed Plummer’s chair to one side. As
(C) excluded, because it is hearsay, not within a result, Plummer fell to the floor. Plummer
any exception. was embarrassed at being made to look foolish
(D) excluded, because Parker is available as a before a large audience but suffered no physical
witness. harm.

486. West, a witness in a contract case, testified on If Plummer asserts a claim against Devon for
direct examination that four people attended a damages because of his embarrassment, will
meeting. When asked to identify them, she gave Plummer prevail?
the names of three but despite trying was unable
to remember the name of the fourth person. (A) Yes, if Devon knew that Plummer was
about to sit on the chair.
The attorney who called her as a witness seeks (B) Yes, if Devon negligently failed to notice
to show her his handwritten notes of the part that Plummer was about to sit on the chair.
of his pretrial interview with her in which she (C) No, because Plummer suffered no physical
provided all four names. harm along with his embarrassment.
(D) No, if in moving the chair Devon intended
The trial court is likely to consider the showing only a good-natured practical joke on
of the notes taken as Plummer.

(A) a proper attempt to introduce recorded


recollection. Questions 489–490 are based on the following fact
(B) a proper attempt to refresh West’s situation.
recollection.
(C) an improper attempt to lead the witness. In a writing signed by both parties, Paul Plannah, a
(D) an improper attempt to support West’s renowned architect, agreed for a fee of $25,000 to
credibility. design and supervise construction of a new house for
Phoebe Threedee, a famous sculptor, the fee to be
487. While hospitalized, Marsh asked her attorney paid upon completion of the house. Plannah and
to draw a deed conveying her home to her son, Threedee got along poorly, and, when the design
Simon. While Marsh remained in the hospital, plans were about two-thirds complete, they had a
the deed was drawn, properly executed, and heated argument over the proper location of a marble
promptly and properly recorded. On being staircase. Hoping to avoid such encounters, Plannah,
informed of the existence of the deed, Simon without Threedee’s knowledge, assigned to Donna
told his mother, “I want no part of the property; Drafty, a newly licensed architect practicing solo,

116
“all of my rights and duties under my design and Which of the following parties most clearly has
construction supervision contract with Threedee.” standing to contest the constitutionality of this
Drafty expressly promised Plannah to carry out the statute of Green in federal court?
work to the best of Drafty’s ability.
(A) A business in another state that supplies
489. For this question only, assume that Threedee, from that other state 95 percent of the
on learning of the assignment, refused to allow goods and services bought by a corporation
Drafty to proceed as architect and brought an that has annual sales in Green of $20
action against Plannah to compel him to resume million.
and complete performance of the contract. (B) A corporation selling $300,000 worth of
goods in Green but presently purchasing
Is Threedee entitled to such relief? only $10,000 in goods and services in
Green.
(A) Yes, because Plannah’s services under the (C) The governor of an adjacent state on behalf
contract are unique. of the state and its residents.
(B) Yes, because Plannah has personally (D) The owner of high-grade, secured bonds
completed two-thirds of the design work. issued by a corporation with sales in Green
(C) No, because the Plannah-Threedee contract of $10 million that currently purchases
is one for personal services by Plannah. only $1 million in goods and services in
(D) No, because Plannah effectively delegated Green.
his remaining duties under the Plannah- 492. Constance owned Greenacre in fee simple. She
Threedee contract to Drafty. executed two instruments in the proper form of
deeds. The first instrument purported to convey
490. For this question only, assume that Threedee an undivided one-half interest in Greenacre to
allowed Drafty to proceed with the design work Henry and his wife, Audrey, as joint tenants with
but that Drafty without legal excuse abandoned right of survivorship. The second instrument
the project shortly after construction began. purported to convey an undivided one-half interest
in Greenacre to Susan, the only child of Henry.
Which of the following legal conclusions are Susan was 13 years old at the time. The common
correct? law joint tenancy is unmodified by statute.
I. Plannah is liable to Threedee for legal No actual consideration was paid for the deeds.
damages, if any, caused by Drafty’s Constance handed the two deeds to Henry.
default. Henry promptly and properly recorded the deed
II. Drafty is liable to Threedee for legal to himself and Audrey and put the deed to his
damages, if any, caused by Drafty’s daughter, Susan, in a safe-deposit box without
default. recording it.
III. Threedee is indebted to Drafty, on a
divisible contract theory, for a prorated The same year, Henry, Audrey, and Susan were
portion of the agreed $25,000 architect’s on a vacation when the plane in which they
fee promised to Plannah. were flying went down, and all three were killed
simultaneously. Henry, Audrey, and Susan died
(A) I and II only intestate. The applicable statute in the jurisdiction
(B) I and III only provides that “when title to property on its
(C) II and III only devolution depends on priority of death and
(D) I, II, and III there is insufficient evidence that the persons
have died otherwise than simultaneously, the
property of each person shall be disposed of as
if he had survived.” An appropriate action was
491. There is high and persistent unemployment in instituted by the heirs of Henry, Audrey, and
the industrialized state of Green. Its legislature Susan. Constance, who is not an heir of any of
therefore enacted a statute requiring every the deceased, was a party to the action.
business with annual sales in Green of over
$1 million to purchase each year goods and/or The court should determine that title to
services in Green equal in value to at least half Greenacre is
of its sales in Green.
(A) entirely in Constance.

117
(B) one-half in the heirs of Henry and one-half (A) recover if Prout suffered severe emotional
in the heirs of Audrey. distress as a consequence of Denton’s
(C) one-half in Constance, one-quarter in the conduct.
heirs of Henry, and one-quarter in the heirs (B) recover, because Denton intended to
of Audrey. frighten Prout.
(D) one-half in the heirs of Susan, one-quarter (C) not recover, because Denton made no
in the heirs of Henry, and one-quarter in threat of immediate physical harm to Prout
the heirs of Audrey. or his family.
(D) not recover if Prout suffered no physical
493. In Peck’s antitrust suit against manufacturers harm as a consequence of Denton’s
of insulation, Peck’s interrogatories asked for conduct.
information concerning total sales of insulation
by each of the defendant manufacturers in a
particular year. The defendants replied to the
interrogatories by referring Peck to the Insulation 496. The German-made Doppelpferd, featuring sleek
Manufacturers’ Annual Journal for the styling and remarkable fuel efficiency, is the
information. most popular automobile in the United States.
Its U.S. sales are booming, and the average retail
If, at trial, Peck offers the annual journal as markup in such sales is 30 percent. Hardsell
evidence of the sales volume, this evidence is Motors, Inc., a franchised Doppelpferd dealer in
the United States, contracted with Shift to sell
(A) admissible as an adoptive admission of the him a new Doppelpferd for $9,000 cash, the sale
defendants. to be consummated after delivery to Hardsell
(B) admissible as a business record. of the car, which Hardsell ordered from the
(C) inadmissible, because it is hearsay, not manufacturer specifically for Shift. The signed
within any exception. retail contractual document was a contract
(D) inadmissible as lacking sufficient drafted by Hardsell’s lawyer, and Shift did not
authentication. question or object to any of its terms, including
the price inserted by Hardsell. When the car
arrived from Germany, Shift repudiated the
Questions 494–495 are based on the following fact contract. Hardsell at once sold the car for $9,000
situation. cash to Karbuff, for whom Hardsell had also
ordered from the manufacturer a Doppelpferd
When Denton heard that his neighbor, Prout, intended identical to Shift’s.
to sell his home to a minority purchaser, Denton told
Prout that Prout and his wife and children would meet In an action against Shift for breach of contract,
with “accidents” if he did so. Prout then called the Hardsell will probably recover
prospective purchaser and told him that he was taking
the house off the market. (A) $9,000 minus what it cost Hardsell to
purchase the car from the manufacturer.
494. If Prout asserts a claim against Denton for (B) $9,000 minus the wholesale price of
assault, Prout will an identical Doppelpferd in the local
wholesale market among dealers.
(A) recover if Denton intended to place Prout (C) nominal damages only, because Hardsell
in fear of physical harm. resold the car to Karbuff without lowering
(B) recover, because Denton’s conduct was the retail price.
extreme and outrageous. (D) nothing, because the parties’ agreement
(C) not recover if Denton took no action that was an adhesion contract and therefore
threatened immediate physical harm to unconscionable.
Prout.
(D) not recover, because Prout’s action 497. A statute in a jurisdiction makes it a crime to
removed any threat of harmful force. sell ammunition to a minor (defined as a person
under the age of 18). The courts have interpreted
495. If Prout asserts a claim against Denton for this statute as creating a strict liability offense
intentional infliction of emotional distress, Prout that does not require knowledge of the age of
will the purchaser and as creating vicarious liability.
Duncan, who was 16 years old, but looked

118
four or five years older, entered a store owned to spread the word of St. Rockport and his
by Matthews and asked a clerk for a box of coming appearance on earth.
.22-caliber shells. Matthews had instructed her
employees not to sell ammunition to minors. The Following complaints by several contributors
clerk asked Duncan his age. Duncan said he was who claimed he defrauded them, Jones was
20. The clerk then placed a box of shells on the again charged with fraud under this state statute.
counter and asked, “Anything else?” Duncan The charge was that Jones “should have known
said that was all he wanted but then discovered that his representations about St. Rockport were
he did not have enough money to pay for the false and, therefore, that he made them solely to
shells, so the clerk put the box back onto the collect cash donations for his personal gain.” A
shelf. witness for the prosecution in Jones’ trial stated
that Jones had admitted that, at times, he had
If Matthews, the owner of the store, is charged doubts about the existence of St. Rockport. Jones
with attempting to violate the statute, her best was the only religious minister prosecuted for
argument would be that fraud under this state statute.
(A) it was impossible for the sale to have The strongest constitutional defense that Jones
occurred. could assert would be that this prosecution
(B) she had strictly instructed her employees
not to sell ammunition to minors. (A) deprived him of the equal protection of the
(C) Duncan lied about his age.
(D) the clerk did not have the mental state laws because other religious ministers have
needed for attempt. not been charged under this statute.
(B) denied him procedural due process
498. Otto conveyed Goldacre to “Andy, his heirs and because it placed upon Jones the burden of
assigns, but if Andy dies and is not survived by rebutting evidence, submitted by the state,
children by his present wife, Jane, then to Bob of his bad faith in raising this money.
and his heirs and assigns.” Shortly after taking (C) denied him rights conferred by the
possession, Andy discovered rich metal deposits obligation of contracts clause by
on the land, opened a mining operation, and preventing him from taking money from
removed and sold a considerable quantity of persons who wished to contract with him
valuable ore without giving Bob any notice of to spread the word of St. Rockport.
his action. Andy has no children. Andy, Jane, (D) denied him the free exercise of religion
and Bob are all still living. Bob brought an in violation of the First and Fourteenth
action in equity for an accounting of the value Amendments because it required the state
of the ore removed and for an injunction against to determine the truth or falsity of the
further removal. content of his religious beliefs.

If the decision is for Andy, it will be because


Questions 500–501 are based on the following fact
(A) Bob has no interest in Goldacre. situation.
(B) the right to take minerals is an incident of
a defeasible fee simple. Hammond decided to kill his wife by poisoning her.
(C) the right to take minerals is an incident of He asked his friend, Jordan, a pharmacist, to obtain
the right to possession. some curare, a deadly poison, and to give it to him
(D) there was no showing that Andy acted in without recording the transaction. Because Jordan
bad faith. suspected Hammond’s motive, she supplied Hammond
with a small quantity of Marvane, an antibiotic,
499. A state statute makes fraud for personal instead of curare. Marvane is harmless if administered
financial gain a crime. Jones was convicted in small quantities, except for the less than one
of violating this statute on three separate percent of the population who are allergic to the drug.
occasions. Following his most recent conviction, Hammond injected his wife with the Marvane while
he professed to have undergone a religious she slept. She was allergic to the drug and died from
conversion and proclaimed himself to be the the injection. Jordan was distraught and confessed
divine minister of “St. Rockport,” an alleged the entire affair to the police, explaining that she had
messiah who would shortly be making his failed to report Hammond’s conduct to the authorities
appearance on earth. Jones solicited cash because she feared that it would end their friendship if
donations from the public to support his efforts she did.

119
500. Jordan is an accomplice to On February 1, Awl notified Howser that he (Awl)
would lose money on the job at that price, and would
(A) murder. not proceed with the work unless Howser would agree
(B) manslaughter. to increase the price to $90,000. Howser thereupon,
(C) criminally negligent homicide. without notifying Awl, agreed in writing with Gutter
(D) no degree of criminal homicide. for Gutter, commencing April 1, to build the house for
$75,000, which was the fair market cost of the work to
501. In a common law jurisdiction, Hammond is be done.
guilty of
On April 1, both Awl and Gutter showed up at the
(A) murder only. building site to begin work, Awl telling Howser that
(B) murder and conspiracy. he had decided to “take the loss” and would build
(C) attempted murder only. the house for $60,000 as originally agreed. Howser
(D) attempted murder and conspiracy. dismissed Awl and allowed Gutter to begin work on
the house.

503. In a contract action by Awl against Howser,
502. While on a hiking trip during the late fall, Page which of the following would the court decide
arrived, toward the end of the day, at a clearing under the prevailing American view?
where several similar cabins were located,
none of which was occupied. One of the cabins (A) Howser will win, because Awl in legal
belonged to Levin, Page’s friend, who had given effect committed a total breach of contract.
Page permission to use it. Page entered one of (B) Howser will win, because Gutter’s contract
the cabins, which she thought was Levin’s, and price was $15,000 lower than the $90,000
prepared to spend the night. In fact the cabin was demanded by Awl on February 1.
owned, not by Levin, but by Dwyer. (C) Awl will win, because Howser did not tell
him before April 1 about the contract with
When the night turned cold, Page started a fire in Gutter.
the stove. Unknown to Page, there was a defect (D) Awl will win, because he attempted to
in the stove that allowed carbon monoxide fumes perform the contract as originally agreed.
to escape into the cabin. During the night the
fumes caused serious injury to Page. 504. For this question only, assume that Awl is
liable to Howser for breach of contract and also
If Page asserts a claim against Dwyer for her assume the following additional facts: Gutter
injury, will Page recover? finished the house on schedule and then showed
Howser that he (Gutter) had spent $85,000 on
(A) Yes, if Dwyer knew that the stove was the job. Howser thereupon paid Gutter the full
defective. balance of their contract price plus an additional
(B) Yes, if Dwyer could have discovered $10,000 so that Gutter would not lose money.
the defect in the stove by a reasonable
inspection. In a contract action by Howser against Awl,
(C) No, because Dwyer had no reason to Howser will recover
anticipate Page’s presence in the cabin.
(D) No, unless Page needed to use the cabin (A) the difference between the fair market
for her own protection. value of the completed house and Awl’s
original contract price.
(B) $30,000, the difference between Awl’s
Questions 503–504 are based on the following fact original contract price and the amount Awl
situation. demanded on February 1.
(C) $25,000, the difference between Awl’s
On January 1, Awl and Howser agreed in writing that original contract price and the total amount
Awl would build a house on Howser’s lot according Howser paid Gutter for building the house.
to Howser’s plans and specifications for $60,000, the (D) $15,000, the difference between Awl’s
work to commence on April 1. Howser agreed to make original contract price and Gutter’s
an initial payment of $10,000 on April 1, and to pay contract price.
the balance upon completion of the work.

120
505. During the night, Murphy broke into a house (C) inadmissible, because criminal conduct can
with the intention of stealing a typewriter. On be shown only by admission or a record of
not finding a typewriter, she became angry, conviction.
poured lighter fluid onto a couch, and set it on (D) inadmissible, because bias must be shown
fire. The flames destroyed the couch and also on cross-examination and not by extrinsic
burned a portion of the ceiling in the room. evidence.

In a common law jurisdiction, Murphy is guilty of 508. Taylor and Scott, an unmarried couple, purchased
a condominium as tenants in common and lived
(A) burglary only. in the condominium for three years. Subsequently,
(B) arson only. they made a verbal agreement that, on the death
(C) burglary and attempted arson. of either of them, the survivor would own the
(D) burglary and arson. entire condominium, and, as a result, they decided
they did not need wills.
506. Argus Corporation is privately owned and
incorporated in the state of Kiowa. It contracted Two years later, Taylor and Scott were involved
with the United States to construct a dam across in the same automobile accident. Taylor was
the Big Sandy River in the state of Arapaho. The killed immediately. Scott died one week
state of Arapaho imposed a gross receipts tax on later. Both died intestate. Taylor’s sole heir
all business conducted within the state. Arapaho is his brother, Mark. Scott’s sole heir is her
sued Argus Corporation to collect that tax on mother, Martha. Mark claimed one-half of the
the receipts Argus received under this federal condominium, and Martha claimed all of it. The
contract. No federal statutes or administrative jurisdiction has no applicable statute except
rules are applicable, and the contract between for the Statute of Frauds; nor does it recognize
the United States and Argus Corporation does common law marriages.
not mention state taxation.
In an appropriate action by Martha claiming the
The court should hold the state tax, as applied entire ownership of the condominium, the court
here, to be will find that

(A) constitutional, because a state has (A) Martha owns the entire interest because
exclusive jurisdiction over all commercial Taylor and Scott did not make wills in
transactions executed wholly within its reliance upon their oral agreement.
borders. (B) Martha owns the entire interest because
(B) constitutional, because private contractors she is entitled to reformation of the deed to
performing work under a federal contract reflect the verbal agreement.
are not immune in these circumstances (C) Mark and Martha each own an undivided
from nondiscriminatory state taxation. one-half interest because Taylor and
(C) unconstitutional, because it violates the Scott each died as the result of the same
supremacy clause. accident.
(D) unconstitutional, because it imposes an (D) Mark and Martha each own an undivided
undue burden on interstate commerce. one-half interest because the Statute of
Frauds applies.
507. Parmott sued Dexter in an automobile collision
case. At trial, Parmott wishes to show by 509. Telco, a local telephone company, negligently
extrinsic evidence that Wade, Dexter’s primary allowed one of its telephone poles, located
witness, is Dexter’s partner in a gambling between a street and a sidewalk, to become
operation. termite-ridden. Rhodes, who was intoxicated and
driving at an excessive rate of speed, lost control
This evidence is of her car and hit the weakened telephone pole.
One week later, the pole fell and struck Walker,
(A) admissible as evidence of Wade’s a pedestrian who was walking on the sidewalk.
character. The pole fell because of the combination of the
(B) admissible as evidence of Wade’s possible force of the impact and the pole’s termite-ridden
bias in favor of Dexter. condition.

121
If Walker asserts a claim against Telco and Petrone bought a home approximately 1,800 feet
Rhodes, will Walker prevail? from the Silo plant.

(A) Yes, against Telco but not Rhodes. One year ago, Silo stopped using bagged cement
(B) Yes, against Rhodes but not Telco. and began to receive cement in bulk shipments.
(C) Yes, against Telco and Rhodes, each for Since then at least five truckloads of cement
one-half of his damages. have passed Petrone’s house daily. Cement
(D) Yes, against both Telco and Rhodes for the blows off the trucks and into Petrone’s house.
full amount of his damages. When the cement arrives at the Silo plant, it
is blown by forced air from the trucks into the
510. Dirk broke into Vera’s house one night. As he storage bin. As a consequence cement dust fills
started to stuff silverware into a sack, he was the air surrounding the plant to a distance of
surprised by Vera, who had arrived home earlier 2,000 feet. Petrone’s house is the only residence
than usual. Dirk struck Vera on the head with a within 2,000 feet of the plant.
candlestick and tied her up. He finished filling
his sack and left. If Petrone asserts a claim against Silo based on
nuisance, will Petrone prevail?
The police discovered Vera several hours
later and rushed her to the hospital. Dirk was (A) Yes, unless using bagged cement would
apprehended by the police early the following substantially increase Silo’s costs.
morning with the loot still in his possession. He (B) Yes, if the cement dust interfered
was taken to police headquarters, given Miranda unreasonably with the use and enjoyment
warnings, and asked if he wished to make a of Petrone’s property.
statement about the prior evening’s events. (C) No, because Silo is not required to change
The police did not mention that Vera had been its industrial methods to accommodate the
seriously injured and was in the hospital. Dirk needs of one individual.
said he understood his rights and was willing (D) No, if Silo’s methods are in conformity
to talk. He then admitted that he committed the with those in general use in the industry.
burglary of Vera’s house. The following day,
Vera died from injuries caused by the blow to 512. On a wholly random basis, a state agency has
her head. given a few probationary employees who were
not rehired at the end of their probationary
If, at Dirk’s trial for murder, Dirk moves to periods a statement of reasons and an
prevent introduction of the confession into opportunity for a hearing; but the agency has
evidence, his motion should most probably be very rarely done so. No statute or rule of the
agency required such a statement of reasons or
(A) denied, because failure of the police to a hearing.
advise Dirk of Vera’s condition was
harmless error since felony murder does The employment of Masters, a probationary
not require intent to kill or injure. employee, was terminated without a statement
(B) denied, because Dirk’s waiver of his rights of reasons or an opportunity for a hearing. The
did not depend upon the nature of the agency did not even consider whether it should
charges that were later filed against him. give him either.
(C) granted, because Dirk could not make a
knowing and intelligent waiver unless A suit by Masters requesting a statement of
he had information concerning Vera’s reasons and a hearing will probably be
condition.
(D) granted, because the use of a confession (A) successful on the grounds that failure to
to burglary in a prosecution for murder give Masters reasons and an opportunity
violates due process where the police for a hearing constituted a bill of attainder.
withheld information about the potential (B) successful on the grounds that an
seriousness of the offense. agency’s inconsistent practices, even if
unintentional, deny adversely affected
511. In 1956, Silo Cement Company constructed a persons the equal protection of the laws.
plant for manufacturing ready-mix concrete in (C) unsuccessful, because Masters does not
Lakeville. At that time Silo was using bagged have a right to be rehired that is protected
cement, which caused little or no dust. In 1970, by procedural due process.

122
(D) unsuccessful, because the conditions of (B) The description of the property to be sold
state employment are matters reserved to in the parties’ written agreement is too
the states by the Tenth Amendment. indefinite to permit the remedy sought.
(C) Quark’s failure to have the five-acre
513. Able, an attorney, sued Clinton, a client, for parcel surveyed was failure of a condition
his fee, based on an agreed hourly rate. Clinton precedent to his own duty of performance.
subpoenaed the attorney’s time records for the (D) The option part of the agreement is
days on which he purported to have worked unenforceable under the parol evidence
for Clinton, in order to show that Able had rule.
billed an impossible number of hours to Clinton
and others on those days. Clinton’s subpoena 515. Assume for this question only that Quark is
provided that any information concerning the not liable to Furrow for breach of a land-sale
matters handled for other clients be deleted or contract.
masked. Able moved to quash the subpoena on
the ground of attorney-client privilege. In an action by Furrow against Quark for the
reasonable value of the improvements that Furrow
The subpoena should be added to the farm, which of the following
theories would best support Furrow’s claim?
(A) upheld, because the information about
hours billed is not within the privilege. (A) Quasi-contract, for benefits unofficiously
(B) upheld, because an attorney has no right and nongratuitously conferred upon Quark
to invoke his clients’ privilege without by Furrow.
instructions from the clients. (B) Tort, for conversion by Quark in retaking
(C) quashed, because an attorney is entitled to possession of the improvements.
a right of privacy for the work product in (C) Breach of trust by Quark as trustee of a
his files. resulting trust of the improvements.
(D) quashed, because no permission was (D) Breach by Quark of an implied-in-fact
obtained from the other clients to divulge promise (manifested by his retaking
information from their files. possession of the farm and improvements)
to compensate Furrow for the improvements.

Questions 514–515 are based on the following fact


situation.
516. Abco developed a new drug, ZB, for treatment
Furrow leased in writing a 100-acre farm from Quark of Wegener’s disease. Abco extensively tested
for five years at $2,000 per year, with an option to ZB for several years on animals and human
purchase “five acres of the land for $10,000 cash” at volunteers and observed no undesirable
the end of the lease term. Before the lease was side effects. The federal Food and Drug
executed, Quark orally promised to have a five-acre Administration (FDA) then approved ZB for
parcel surveyed before the end of the lease term. sale as a prescription drug.
Furrow took possession of the farm and paid the rent
for five years. During the fifth year, having decided Five other drug companies, each acting
that he would exercise the purchase option, Furrow independently, developed drugs identical to ZB.
planted several fruit trees and built a large grain Each of these drugs was also approved by the
silo on the property. At the end of the term, Furrow FDA for sale as a prescription drug. True Blue
tendered Quark $10,000 and demanded a conveyance, Drug, a wholesaler, bought identically shaped
but Quark repudiated the option agreement and retook pills from all six of the manufacturers and sold
possession of the farm. He had never had the five-acre the pills to drugstores as Wegener’s X.
parcel surveyed.
This drug had a long-delayed side effect. Sons
514. In an action by Furrow against Quark for of male users of Wegener’s X are sterile. One
specific performance of the option agreement, such son, Crane, brought an action against Abco
which of the following is Quark’s best defense? for his damages. Abco, through True Blue Drug,
supplied about 10 percent of the Wegener’s X
(A) The option part of the agreement is sold in the state where Crane lived. It is not
unenforceable because it lacked a separate possible to establish which of the six companies
consideration. supplied the particular pills that Crane’s father
took.

123
If Crane asserts a claim against Abco based on (B) admissible as a family record.
strict liability in tort, which of the following will (C) inadmissible, because it is hearsay, not
be a decisive question in determining whether within any exception.
Crane will prevail? (D) inadmissible, because there was no showing
of firsthand knowledge by the one who
(A) Does the res ipsa loquitur doctrine apply? wrote it.
(B) Can liability be imposed on Abco without
proof that Abco knew that the drug had an 519. Smythe was charged with the murder of his
undesirable side effect? wife. In his defense, he testified that at the time
(C) Is Abco relieved of liability by the FDA he killed her, he believed that his wife was
approval of the drug? planning to destroy the world by detonating a
(D) Can liability be imposed on Abco without massive explosive device that she had developed
showing that its pills were used by Crane’s and built in the basement of their home. He
father? further testified that he had tried many times to
dissuade his wife from her plan and had tried to
517. A state statute provides that only citizens of destroy devices that she stored in the basement.
the United States may be employed by that She had, he testified, foiled his efforts by on two
state. In an action brought in a federal court, a occasions signing papers for his hospitalization,
resident alien who was prevented from obtaining which lasted for a brief period each time. He
state employment as a garbage collector said that he had concluded that the only way to
solely because of his alien status challenged prevent her scheme was to kill her and that he
the statute’s constitutionality as applied to his had become so obsessed with the importance
circumstances. of doing so that he could think of nothing else.
One day when he saw her open the door to the
Which of the following statements concerning basement he lunged at her and pushed her down
the burden of persuasion applicable to this suit is the steps to her death.
correct?
The best defense raised by Smythe’s testimony is
(A) The alien must demonstrate that there is no
rational relationship between the citizenship (A) lack of the requisite mental element.
requirement and any legitimate state interest. (B) lack of the requisite act element.
(B) The alien must demonstrate that the (C) insanity.
citizenship requirement is not necessary to (D) belief that the situation justified his
advance an important state interest. actions.
(C) The state must demonstrate that there
is a rational relationship between the 520. After being notified by Dr. Josephs that Nurse
citizenship requirement and a legitimate Norris’s employment with his office was
state interest. terminated, Norris applied for a position with
(D) The state must demonstrate that the Hospital. In her application, Norris listed her
citizenship requirement is necessary to former employment with Josephs. Josephs,
advance an important state interest. in response to a telephone inquiry from
Hospital, stated that “Norris lacked professional
518. In litigation over the estate of Baggs, who died competence.” Although Josephs believed that
intestate, Payton, who is 18 years old, claimed to be a fair assessment of Norris, his adverse
to be Baggs’s niece and entitled, therefore, to a rating was based on one episode of malpractice
share of his large estate. In support of her claim, for which he blamed Norris but which in fact
Payton offered in evidence a Bible, properly was chargeable to another doctor. Because of
identified as having belonged to Baggs’s family, the adverse comment on her qualifications
in the front of which was a list of family births, provided by Josephs, Norris was not employed
marriages, and deaths. The list recorded Payton’s by Hospital.
birth to Baggs’s oldest sister.
If Norris asserts a claim based on defamation
To prove that Payton is Baggs’s niece, the Bible against Josephs, will Norris prevail?
listing is
(A) Yes, because Josephs was mistaken in the
(A) admissible as an ancient document. facts on which he based his opinion of
Norris’s competence.

124
(B) Yes, because the statement of Josephs Which of the following constitutional provisions
reflected adversely on Norris’s may most easily and directly be used to justify
professional competence. the federal statute authorizing this leasing
(C) No, if Norris authorized Hospital to make program of the Bureau of Land Management?
inquiry of her former employer.
(D) No, if Josephs had reasonable grounds for (A) The general welfare clause of Article I,
his belief that Norris was not competent. § 8.
(B) The federal property clause of Article IV, §
521. Pullen used aluminum brackets in her business. 3.
On the telephone listed as hers in the telephone (C) The commerce clause of Article I, § 8.
book, Pullen received a call in which the (D) The supremacy clause of Article VI.
caller said, “This is John Denison of Denison
Hardware Company. We have a special on 523. Ann leased commercial property to Brenda
aluminum brackets this week at 30 percent off.” for a period of 10 years. The lease contained
Pullen ordered brackets from the caller. When the following provision: “No subleasing or
the brackets were never delivered, Pullen sued assignment will be permitted unless with the
Denison for breach of contract. written consent of the lessor.” One year later,
Brenda assigned all interest in the lease to
At trial, Denison, who denies having made the Carolyn, who assumed and agreed to perform
telephone call, objects to Pullen’s testimony the lessee’s obligations under the terms of the
concerning it. When asked, Pullen testifies that, lease. Ann learned of the assignment and wrote
aside from the telephone call, she had never to Brenda that she had no objection to the
heard Denison speak until she met him in the assignment to Carolyn and agreed to accept rent
judge’s chambers before the trial and that, in from Carolyn instead of Brenda.
her opinion, the voice on the telephone was
Denison’s. Thereafter, Carolyn paid rent to Ann for a period
of five years. Carolyn then defaulted and went
The strongest argument for admission of Pullen’s into bankruptcy. In an appropriate action, Ann
testimony concerning the telephone call is that sued Brenda for rent due.

(A) the call related to business reasonably If Ann loses, it will be because there was
transacted over the telephone.
(B) the call was received at a number assigned (A) laches.
to Pullen by the telephone company. (B) an accord and satisfaction.
(C) after hearing Denison speak in chambers, (C) a novation.
Pullen recognized Denison’s voice as that (D) an attornment.
of the person on the telephone.
(D) self-identification is sufficient
authentication of a telephone call. Questions 524–525 are based on the following fact
situation.
522. The High National Grasslands is owned by the
United States and is located in the center of a On January 2, Hugh Homey and Sue Structo entered
large western state. Acting pursuant to a federal into a written contract in which Structo agreed to build
statute authorizing such action, the United on Homey’s lot a new house for Homey, according
States Bureau of Land Management leased the to plans and specifications furnished by Homey’s
grazing rights in the High National Grasslands to architect, Barbara Bilevel, at a contract price of
ranchers located nearby. Grazingland Company $200,000. The contract provided for specified progress
owns a vast amount of rangeland adjacent to the payments and a final payment of $40,000 upon
High National Grasslands and leases its land for Homey’s acceptance of the house and issuance of a
livestock-grazing purposes to the same ranchers, certificate of final approval by the architect. Further,
but at prices higher than those charged by the under a “liquidated damages” clause in the agreement,
Bureau. Grazingland Company sued the Bureau Structo promised to pay Homey $500 for each day’s
in an appropriate federal district court to restrain delay in completing the house after the following
the Bureau from competing with that company October 1. Homey, however, told Structo on January
by leasing the High National Grasslands. 2, before the contract was signed, that he would be on
an around-the-world vacation trip most of the summer

125
and fall and would not return to occupy the house until conferred on Homey, minus progress
November 1. payments already received.
(C) Homey wins, provided he can prove by
524. For this question only, assume the following clear and convincing evidence that the fair
facts. Because she was overextended on other market value of the completed house is
construction jobs, Structo did not complete the $160,000 or less.
house until October 15. Homey returned on (D) Homey wins, provided he can prove by
November 1 as planned and occupied the house. clear and convincing evidence that total
Ten days later, after making the $40,000 final payments to Structo of $160,000 will yield
payment to Structo, Homey learned for the first a fair net profit.
time that the house had not been completed until
October 15.

If Homey sues Structo for breach of contract 526. Chemco designed and built a large tank on
on account of the 15-day delay in completion, its premises for the purpose of storing highly
which of the following will the court probably toxic gas. The tank developed a sudden leak
decide? and escaping toxic gas drifted onto the adjacent
premises, where Nyman lived. Nyman inhaled
(A) Homey will recover damages as specified the gas and died as a result.
in the contract, i.e., $500 multiplied by 15.
(B) Homey will recover his actual damages, if In a suit brought by Nyman’s personal
any, caused by the delay in completion. representative against Chemco, which of the
(C) Having waived the delay by occupying following must be established if the claim is
the house and making the final payment, to prevail?
Homey will recover nothing.
(D) Homey will recover nothing because the I. The toxic gas that escaped from Chemco’s
contractual completion date was impliedly premises was the cause of Nyman’s death.
modified to November 1 when Homey on II. The tank was built in a defective manner.
January 2 advised Structo about Homey’s III. Chemco was negligent in designing the
prospective trip and return date. tank.

525. For this question only, assume the following (A) I only.
facts. Structo completed the house on October (B) I and II only.
14 and, when Homey returned on November (C) I and III only.
1, requested the final payment of $40,000 and (D) I, II, and III.
issuance of a certificate of final approval by the
architect, Bilevel. Homey, however, refused to 527. Zall, a resident of the state of Paxico, brought
pay any part of the final installment after Bilevel suit in federal district court against Motors, Inc.,
told him, “Structo did a great job and I find no a Paxico corporation. Zall seeks recovery of
defects worth mentioning; but Structo’s contract $12,000 actual and $12,000 punitive damages
price was at least $40,000 too high, especially arising from Motors’s sale to him of a defective
in view of the big drop in housing values within automobile. Zall’s suit is based only on a
the past 10 months. I will withhold the final common law contract theory.
certificate, and you just hold on to your money.”
If Structo sues Homey for the $40,000 final From a constitutional standpoint, should the
payment after Bilevel’s refusal to issue a final federal district court hear this suit on its merits?
certificate, which of the following will the court
probably decide? (A) Yes, because Article III vests federal
courts with jurisdiction over cases
(A) Structo wins, because nonoccurrence of the involving the obligation of contracts.
condition requiring Bilevel’s certificate of (B) Yes, because it is an action affecting
final approval was excused by Bilevel’s interstate commerce.
bad-faith refusal to issue the certificate. (C) No, because this suit is not within the
(B) Structo wins, but, because all contractual jurisdiction of an Article III court.
conditions have not occurred, her recovery (D) No, because there is no case or controversy
is limited to restitution of the benefit within the meaning of Article III.

126
528. Pack sued Donlon for slander, alleging that (C) Oaks did not expressly agree to the
Donlon had publicly accused Pack of being a location of the right-of-way.
thief. In his answer, Donlon admitted making (D) the assertion of the right to install an
the accusation, but alleged that it was a true additional pipeline constitutes inverse
statement. condemnation.

At trial, Donlon offers evidence that Pack stole 531. A statute authorizes a specified federal
a ring worth $10,000 from a jewelry store. administrative agency to issue rules governing
the distribution of federal grant funds for
Evidence concerning this theft should be scientific research. The statute provides that,
in issuing those rules, the agency must follow
(A) admitted, because specific instances of procedures and substantive standards contained
conduct may be proved when character is in the statute. In a severable provision, the
directly in issue. statute also provides that otherwise valid rules
(B) admitted, because Pack’s action constituted issued by the agency under authority delegated
a felony. to it by this statute may be set aside by a
(C) excluded, because character must be majority vote of a designated standing joint
shown by reputation or opinion. committee of Congress.
(D) excluded, because its relevance is
substantially outweighed by the danger of The provision of this statute relating to
unfair prejudice. the power of the designated standing joint
committee of Congress is
529. Rimm and Hill were fooling around with a pistol
in Hill’s den. Rimm aimed the pistol in Hill’s (A) constitutional, because it is a necessary
direction and fired three shots slightly to Hill’s and proper means of ensuring that the
right. One shot ricocheted off the wall and struck rules issued by this agency are actually
Hill in the back, killing him instantly. consistent with the will of Congress.
(B) constitutional, because discretionary
The most serious crime of which Rimm can be money grants authorized by statute are
convicted is privileges, not rights, and therefore
Congress has greater freedom to
(A) murder. intervene in their administration than it
(B) voluntary manslaughter. has to intervene in the administration of
(C) involuntary manslaughter. regulatory laws.
(D) assault with a dangerous weapon. (C) unconstitutional, because it denies equal
protection of the laws to members of
530. Oaks, the owner of Blackacre, conveyed a right- Congress who are not appointed to the
of-way to United Utility “for the underground joint legislative committee authorized to
transportation of gas by pipeline, the location set aside rules of this agency.
of right-of-way to be mutually agreed upon by (D) unconstitutional, because it authorizes a
Oaks and United Utility.” United Utility then congressional change of legal rights and
installed a six-inch pipeline at a location selected obligations by means other than those
by it and not objected to by Oaks. Two years specified in the Constitution for the
later, United Utility advised Oaks of its intention enactment of laws.
to install an additional six-inch pipeline parallel
to and three feet laterally from the original 532. Perez sued Dawson for damages arising out of
pipeline. In an appropriate action, Oaks sought an automobile collision. At trial, Perez called
a declaration that United Utility has no right to Minter, an eyewitness to the collision. Perez
install the second pipeline. expected Minter to testify that she had observed
Dawson’s automobile for five seconds prior to
If Oaks prevails, it will be because the collision and estimated Dawson’s speed at
the time of the collision to have been 50 miles
(A) any right implied to expand the original per hour. Instead, Minter testified that she
use of the right-of-way creates an interest estimated Dawson’s speed to have been 25 miles
that violates the Rule Against Perpetuities. per hour.
(B) the original installation by United Utility
defined the scope of the easement.

127
Without finally excusing Minter as a witness, to my daughter, Doris.” Five years ago, Sam
Perez then called Wallingford, a police officer, executed an instrument in the proper form of
to testify that Minter had told him during his a deed, purporting to convey Blackacre and
investigation at the accident scene that Dawson Whiteacre to Joe in fee simple. Joe then enlarged
“was doing at least 50.” the motel to 12 units. Six months ago, Sam died
and Doris took possession of Whiteacre. She
Wallingford’s testimony is brought an appropriate action to enjoin Joe from
using the right-of-way.
(A) admissible as a present sense impression.
(B) admissible to impeach Minter. In this action, who should prevail?
(C) inadmissible, because Perez may not
impeach his own witness. (A) Doris, because merger extinguished the
(D) inadmissible, because it is hearsay, not easement.
within any exception. (B) Doris, because Joe has overburdened the
easement.
533. On August 1, Geriatrics, Inc., operating a (C) Joe, because he has an easement by
“lifetime care” home for the elderly, admitted necessity.
Ohlster, who was 84 years old, for a trial period (D) Joe, because he has the easement granted
of two months. On September 25, Ohlster and by Frank to Sam.
Geriatrics entered into a written lifetime care
contract with an effective commencement 535. Able and Baker are students in an advanced high
date of October 1. The full contract price was school Russian class. During an argument one
$20,000, which, as required by the terms of day in the high school cafeteria, in the presence
the contract, Ohlster prepaid to Geriatrics on of other students, Able, in Russian, accused
September 25. Ohlster died of a heart attack on Baker of taking money from Able’s locker.
October 2.
In a suit by Baker against Able based on
In a restitutionary action, can the administratrix defamation, Baker will
of Ohlster’s estate, a surviving sister, recover
on behalf of the estate either all or part of the (A) prevail, because Able’s accusation
$20,000 paid to Geriatrics on September 25? constituted slander per se.
(B) prevail, because the defamatory statement
(A) Yes, because Geriatrics would otherwise was made in the presence of third persons.
be unjustly enriched at Ohlster’s expense. (C) not prevail, unless Able made the
(B) Yes, under the doctrine of frustration of accusation with knowledge of falsity or
purpose. reckless disregard of the truth.
(C) No, because Ohlster’s life span and the (D) not prevail, unless one or more of the other
duration of Geriatrics’ commitment to him students understood Russian.
was a risk assumed by both parties.
(D) No, but only if Geriatrics can show that 536. Five years ago, Sally acquired Blackacre,
between September 25 and Ohlster’s death improved with a 15-year-old dwelling. This
it rejected, because of its commitment to year Sally listed Blackacre for sale with Bill, a
Ohlster, an application for lifetime care licensed real estate broker. Sally informed Bill
from another elderly person. of several defects in the house that were not
readily discoverable by a reasonable inspection,
534. Frank owned two adjacent parcels, Blackacre including a leaky basement, an inadequate water
and Whiteacre. Blackacre fronts on a poor supply, and a roof that leaked. Paul responded
unpaved public road, while Whiteacre fronts on to Bill’s advertisement, was taken by Bill to
Route 20, a paved major highway. Fifteen years view Blackacre, and decided to buy it. Bill saw
ago, Frank conveyed to his son, Sam, Blackacre to it that the contract specified the property to
“together with a right-of-way 25 feet wide over be “as is” but neither Bill nor Sally pointed out
the east side of Whiteacre to Route 20.” At that the defects to Paul, who did not ask about the
time, Blackacre was improved with a 10-unit condition of the dwelling. After closing and
motel. taking possession, Paul discovered the defects,
had them repaired, and demanded that Sally
Ten years ago, Frank died. His will devised reimburse him for the cost of the repairs. Sally
Whiteacre “to my son, Sam, for life, remainder

128
refused, and Paul brought an appropriate action chemicals known to cause cancer, unless the
against Sally for damages. product is clearly labeled as dangerous.

If Sally wins, it will be because The constitutionality of this federal statute may
most easily be justified on the basis of the power
(A) Sally fulfilled the duty to disclose defects of Congress to
by disclosure to Bill.
(B) the contract’s “as is” provision controls the (A) regulate commerce among the states.
rights of the parties. (B) enforce the Fourteenth Amendment.
(C) Bill became the agent of both Paul and (C) provide for the general welfare.
Sally and thus knowledge of the defects (D) promote science and the useful arts.
was imputed to Paul.
(D) the seller of a used dwelling that has been 539. Shore decided to destroy his dilapidated building
viewed by the buyer has no responsibility in order to collect the insurance money. He hired
toward the buyer. Parsons to burn down the building. Parsons broke
into the building and carefully searched it to make
537. Hydro-King, Inc., a high-volume pleasure boat sure no one was inside. He failed, however, to see
retailer, entered into a written contract with a vagrant asleep in an office closet. He started a
Zuma, signed by both parties, to sell Zuma a fire. The building was destroyed, and the vagrant
power boat for $12,000. The manufacturer’s died from burns a week later. Two days after the
price of the boat delivered to Hydro-King was fire, Shore filed an insurance claim in which he
$9,500. As the contract provided, Zuma paid stated that he had no information about the cause
Hydro-King $4,000 in advance and promised of the fire.
to pay the full balance upon delivery of the
boat. The contract contained no provision for If Shore is guilty of felony murder, it is because
liquidated damages. Prior to the agreed delivery the vagrant’s death occurred in connection with
date, Zuma notified Hydro-King that he would be the felony of
financially unable to conclude the purchase; and
Hydro-King thereupon resold the same boat that (A) arson.
Zuma had ordered to a third person for $12,000 (B) fraud.
cash. (C) conspiracy.
(D) burglary.
If Zuma sues Hydro-King for restitution of the
$4,000 advance payment, which of the following 540. Dalton is on trial for burglary. During cross-
should the court decide? examination of Dalton, the prosecutor wants
to inquire about Dalton’s earlier conviction for
(A) Zuma’s claim should be denied, because, falsifying a credit application.
as the party in default, he is deemed to
have lost any right to restitution of a Which of the following facts concerning the
benefit conferred on Hydro-King. conviction would be the best reason for the trial
(B) Zuma’s claim should be denied, because, court’s refusing to allow such examination?
but for his repudiation, Hydro-King would
have made a profit on two boat sales (A) Dalton was released from prison 12 years
instead of one. ago.
(C) Zuma’s claim should be upheld in the (B) Dalton was put on probation rather than
amount of $4,000 minus the amount of imprisoned.
Hydro-King’s lost profit under its contract (C) It was for a misdemeanor rather than a
with Zuma. felony.
(D) Zuma’s claims should be upheld in the (D) It is on appeal.
amount of $3,500 ($4,000 minus $500 as
statutory damages under the UCC). 541. Dent operates a residential rehabilitation center
for emotionally disturbed and ungovernable
538. A federal statute prohibits the sale or resale, children who have been committed to his
in any place in this country, of any product custody by their parents or by juvenile authorities.
intended for human consumption or ingestion The center’s purpose is to modify the behavior
into the human body that contains designated of the children through a teaching program
carried out in a family-like environment. Though

129
the children are not permitted to leave the center installments. Clara objected, contending that
without Dent’s permission, there are no bars or Wilma should pay all of these amounts out of
guards to prevent them from doing so. It has the profits she had made in operation of the
been held in the state where the center is located farm. When foreclosure of the mortgage seemed
that persons having custody of children have the imminent, Clara sought legal advice.
same duties and responsibilities that they would
have if they were the parents of the children. If Clara obtained sound advice relating to her
rights, she was told that
Camden, aged 12, who had been in Dent’s custody
for six months, left the center without permission. (A) her only protection lies in instituting an
Dent became aware of Camden’s absence almost action for partition to compel the sale
immediately, but made no attempt to locate him of the life estate of Wilma and to obtain
or secure his return, though reports reached him the value of her own one-third interest in
that Camden had been seen in the vicinity. Thirty- remainder.
six hours after Camden left the center, Camden (B) she could obtain appropriate relief to
committed a brutal assault upon Pell, a five- compel Wilma personally to pay the sums
year-old child, causing Pell to suffer extensive due because the income is more than
permanent injury. adequate to cover these amounts.
(C) she could be compelled personally to pay
If an action is brought against Dent on behalf of her share of the amounts due because
Pell to recover damages for Pell’s injuries, will discharge of the mortgage enhances the
Pell prevail? principal.
(D) she could not be held personally liable for
(A) No, because parents are not personally any amount but that her share in remainder
liable for their child’s intentional torts. could be lost if the mortgage installments
(B) Yes, if Camden was old enough to be are not paid.
liable for battery.
(C) Yes, because Camden was in Dent’s
custody. Questions 543–544 are based on the following fact
(D) No, unless Dent knew or had reason to situation.
know that Camden had a propensity to
attack younger children. Ohner and Planner signed a detailed writing in which
Planner, a landscape architect, agreed to landscape
542. Orin owned in fee simple Blueacre, a farm of and replant Ohner’s residential property in accordance
300 acres. He died and by will duly admitted with a design prepared by Planner and incorporated in
to probate devised Blueacre to his surviving the writing. Ohner agreed to pay $10,000 for the work
widow, Wilma, for life with remainder in fee upon its completion. Ohner’s spouse was not a party
simple to his three children, Cindy, Clara, and to the agreement, and had no ownership interest in the
Carter. All three children survived Orin. premises.

At the time of Orin’s death, there existed a 543. For this question only, assume the following
mortgage on Blueacre that Orin had given 10 facts. Shortly before the agreement was signed,
years before to secure a loan for the purchase of Ohner and Planner orally agreed that the writing
the farm. At his death, there remained unpaid would not become binding on either party unless
$40,000 in principal, payable in installments Ohner’s spouse should approve the landscaping
of $4,000 per year for the next 10 years. In design.
addition, there was due interest at the rate of 10 If Ohner’s spouse disapproves the design and
percent per annum, payable annually with the Ohner refuses to allow Planner to proceed with
installment of principal. Wilma took possession the work, is evidence of the oral agreement
and out of a gross income of $50,000 per year admissible in Planner’s action against Ohner for
realized $25,000 net after paying all expenses breach of contract?
and charges except the installment of principal
and interest due on the mortgage. (A) Yes, because the oral agreement required
approval by a third party.
Carter and Cindy wanted the three children, (B) Yes, because the evidence shows that the
including Clara, to each contribute one-third writing was intended to take effect only if
of the amounts needed to pay the mortgage the approval occurred.

130
(C) No, because the parol evidence rule bars appropriate remedial legislation expressly
evidence of a prior oral agreement even if subjecting the states to private suits for
the latter is consistent with the terms of a damages in federal court.
partial integration. (B) When Congress exercises power vested
(D) No, because the prior oral agreement in it by any provision of the Constitution,
contradicted the writing by making the Congress has unlimited authority to
parties’ duties conditional. authorize private actions for damages
against a state.
544. For this question only, assume the following (C) While the Eleventh Amendment restrains
facts. At Ohner’s insistence, the written Ohner- the federal judiciary, that amendment does
Planner agreement contained a provision that not limit the power of Congress to modify
neither party would be bound unless Ohner’s the sovereign immunity of the states.
law partner, an avid student of landscaping, (D) While the Eleventh Amendment applies
should approve Planner’s design. Before Planner to suits in federal court by citizens of one
commenced the work, Ohner’s law partner, state against another state, it does not
in the presence of both Ohner and Planner, apply to such suits by citizens against their
expressly disapproved the landscaping design. own state.
Nevertheless, Ohner ordered Planner to proceed
with the work, and Planner reluctantly did so. 546. Suspecting that Scott had slain his wife, police
When Planner’s performance was 40 percent detectives persuaded one of Scott’s employees
complete, Ohner repudiated his duty, if any, to to remove a drinking glass from Scott’s office so
pay the contract price or any part thereof. that it could be used for fingerprint comparisons
with a knife found near the body. The fingerprints
If Planner now sues Ohner for damages for matched. The prosecutor announced that he
breach of contract, which of the following would present comparisons and evidence to the
concepts best supports Planner’s claim? grand jury. Scott’s lawyer immediately filed a
motion to suppress the evidence of the fingerprint
(A) Substantial performance. comparisons to bar its consideration by the grand
(B) Promissory estoppel. jury, contending that the evidence was illegally
(C) Irrevocable waiver of condition. acquired.
(D) Unjust enrichment.
The motion should be

(A) granted, because, if there was no probable
545. A federal statute enacted pursuant to the powers cause, the grand jury should not consider
of Congress to enforce the Fourteenth Amendment the evidence.
and to regulate commerce among the states (B) granted, because the employee was
prohibits any state from requiring any of its acting as a police agent and his seizure
employees to retire from state employment of the glass without a warrant was
solely because of their age. The statute expressly unconstitutional.
authorizes employees required by a state to (C) denied, because motions based on the
retire from state employment solely because of exclusionary rule are premature in grand
their age to sue the state government in federal jury proceedings.
district court for any damages resulting from that (D) denied, because the glass was removed
state action. On the basis of this federal statute, from Scott’s possession by a private citizen
Retiree sues State X in federal district court. and not a police officer.
State X moves to dismiss the suit on the ground
that Congress lacks authority to authorize such 547. Deetz was prosecuted for homicide. He testified
suits against a state. that he shot in self-defense. In rebuttal, Officer
Watts testified that he came to the scene in
Which of the following is the strongest argument response to a telephone call from Deetz. Watts
that Retiree can offer in opposition to the state’s offers to testify that he asked, “What is the
motion to dismiss this suit? problem here, sir?” and Deetz replied, “I was
cleaning my gun and it went off accidentally.”
(A) When Congress exercises power vested in
it by the Fourteenth Amendment and/or The offered testimony is
the commerce clause, Congress may enact

131
(A) admissible as an excited utterance. Peterson paid value for the assignment without
(B) admissible to impeach Deetz and as actual knowledge of the prior assignment to
evidence that he did not act in self-defense. XYZ Bank and promptly and properly recorded
(C) inadmissible, because of Deetz’s privilege his assignment. The principal of the note was
against self-incrimination. not then due, and there had been no default in
(D) inadmissible, because it tends to exculpate payment of either interest or principal.
without corroboration.
If the issue of ownership of the Rohan note
548. Landco owns and operates a 12-story apartment and mortgage is subsequently raised in an
building containing 72 apartments, 70 of which appropriate action by XYZ Bank to foreclose,
are rented. Walker has brought an action against the court should hold that
Landco alleging that while he was walking along
a public sidewalk adjacent to Landco’s apartment (A) Peterson owns both the note and the
building a flowerpot fell from above and struck mortgage.
him on the shoulder, causing extensive injuries. (B) XYZ Bank owns both the note and the
The action is to recover damages for those mortgage.
injuries. (C) Peterson owns the note and XYZ Bank
owns the mortgage.
If Walker proves the foregoing facts and offers (D) XYZ Bank owns the note and Peterson
no other evidence explaining the accident, will owns the mortgage.
his claim survive a motion for directed verdict
offered by the defense? 550. Fruitko, Inc., ordered from Orchard, Inc., 500
bushels of No. 1 Royal Fuzz peaches, at a
(A) Yes, because Walker was injured by an specified price, “for prompt shipment.” Orchard
artificial condition of the premises while promptly shipped 500 bushels, but by mistake
using an adjacent public way. shipped No. 2 Royal Fuzz peaches instead of
(B) Yes, because such an accident does No. 1. The error in shipment was caused by the
not ordinarily happen in the absence of negligence of Orchard’s shipping clerk.
negligence.
(C) No, if Landco is in no better position than Which of the following best states Fruitko’s
Walker to explain the accident. rights and duties upon delivery of the peaches?
(D) No, because there is no basis for a
reasonable inference that Landco was (A) Orchard’s shipment of the peaches was
negligent. a counteroffer and Fruitko can refuse to
accept them.
549. Rohan executed and delivered a promissory (B) Orchard’s shipment of the peaches was
note and a mortgage securing the note to a counteroffer but, since peaches are
Acme Mortgage Company, which was named perishable, Fruitko, if it does not want to
as payee in the note and as mortgagee in the accept them, must reship the peaches to
mortgage. The note included a statement that Orchard in order to mitigate Orchard’s
the indebtedness evidenced by the note was losses.
“subject to the terms of a contract between the (C) Fruitko must accept the peaches because
maker and the payee of the note executed on a contract was formed when Orchard
the same day” and that the note was “secured shipped them.
by a mortgage of even date.” The mortgage was (D) Although a contract was formed when
promptly and properly recorded. Subsequently, Orchard shipped the peaches, Fruitko does
Acme sold the Rohan note and mortgage to not have to accept them.
XYZ Bank and delivered to XYZ Bank a written
assignment of the Rohan note and mortgage. The 551. A federal law provides that all motor vehicle
assignment was promptly and properly recorded. tires discarded in this country must be
Acme retained possession of both the note and disposed of in facilities licensed by the federal
the mortgage in order to act as collecting agent. Environmental Protection Agency. Pursuant to
Later, being short of funds, Acme sold the this federal law and all proper federal procedural
note and mortgage to Peterson at a substantial requirements, that agency has adopted very strict
discount. Acme executed a written assignment of standards for the licensing of such facilities.
the note and mortgage to Peterson and delivered As a result, the cost of disposing of tires in
to him the note, the mortgage, and the assignment. licensed facilities is substantial. The state of East

132
Dakota has a very large fleet of motor vehicles, In presenting the state’s case, the prosecutor
including police cars and trucks used to support seeks to introduce evidence that Decker had
state-owned commercial activities. East Dakota robbed two other stores in the past year.
disposes of used tires from both kinds of state
motor vehicles in a state-owned and state- This evidence is
operated facility. This state facility is unlicensed,
but its operation in actual practice meets (A) admissible to prove a pertinent trait of
most of the standards imposed by the federal Decker’s character and Decker’s action in
Environmental Protection Agency on facilities it conformity therewith.
licenses to dispose of tires. (B) admissible to prove Decker’s intent and
identity.
Consistent with United States Supreme Court (C) inadmissible, because character must be
precedent, may the state of East Dakota continue proved by reputation or opinion and may
to dispose of its used tires in this manner? not be proved by specific acts.
(D) inadmissible, because its probative value is
(A) No, because a state must comply with substantially outweighed by the danger of
valid federal laws that regulate matters unfair prejudice.
affecting interstate commerce.
(B) No, because some of the tires come from 554. Palmco owns and operates a beachfront hotel.
vehicles that are used by the state solely in Under a contract with City to restore a public
its commercial activities. beach, Dredgeco placed a large and unavoidably
(C) Yes, because some of the tires come from dangerous stone-crushing machine on City land
vehicles that are used by the state in the near Palmco’s hotel. The machine creates a
performance of core state governmental continuous and intense noise that is so disturbing
functions such as law enforcement. to the hotel guests that they have canceled their
(D) Yes, because the legitimate needs of the hotel reservations in large numbers, resulting in
federal government are satisfied by the fact a substantial loss to Palmco.
that the unlicensed state disposal scheme
meets, in actual practice, most of the Palmco’s best chance to recover damages for
federal standards for the licensing of such its financial losses from Dredgeco is under the
facilities. theory that the operation of the stone-crushing
machine constitutes
552. Kathy, a two-year-old, became ill with
meningitis. Jim and Joan, her parents, were (A) an abnormally dangerous activity.
members of a group who believed fervently that (B) a private nuisance.
if they prayed enough, God would not permit (C) negligence.
their child to die. Accordingly, they did not seek (D) a trespass.
medical aid for Kathy and refused all offers of
such aid. They prayed continuously. Kathy died 555. Owen owned Greenacre, a tract of land, in
of the illness within a week. fee simple. By warranty deed he conveyed
Greenacre to Lafe for life “and from and after
Jim and Joan are charged with murder in a the death of Lafe to Rem, her heirs and assigns.”
common law jurisdiction.
Subsequently Rem died, devising all of her
Their best defense to the charge is that estate to Dan. Rem was survived by Hannah, her
sole heir at law.
(A) they did not intend to kill or to harm
Kathy. Shortly thereafter Lafe died, survived by Owen,
(B) they were pursuing a constitutionally Dan, and Hannah.
protected religious belief.
(C) Kathy’s death was not proximately caused Title to Greenacre now is in
by their conduct.
(D) they neither premeditated nor deliberated. (A) Owen, because the contingent remainder
never vested and Owen’s reversion was
553. Decker, charged with armed robbery of a store, entitled to possession immediately upon
denied that he was the person who had robbed Lafe’s death.
the store.

133
(B) Dan, because the vested remainder in Rem Because Fabric Mill wishes to continue to
was transmitted by her will. have its fabric tested solely by Alpha Testing
(C) Hannah, because she is Rem’s heir. Company, Fabric Mill files an action in Orange
(D) either Owen or Hannah, depending upon state court challenging the constitutionality of
whether the destructibility of contingent the Orange statute as applied to its circumstances.
remainders is recognized in the applicable
jurisdiction. In this suit, the court should hold the statute to be

556. Pater and his adult daughter, Carmen, (A) constitutional, because it is reasonably
encountered Tertius, an old family friend, related to the protection of the reputation
on the street. Carmen said to Tertius, “How of the fabric industry located in the state of
about lending me $1,000 to buy a used car? Orange.
I’ll pay you back with interest one year from (B) constitutional, because it is a legitimate
today.” Pater added, “And if she doesn’t pay means of protecting the safety of the
it back as promised, I will.” Tertius thereupon public.
wrote out and handed to Carmen his personal (C) unconstitutional, because it denies to
check, payable to her, for $1,000, and Carmen Fabric Mill the equal protection of the
subsequently used the funds to buy a used car. laws.
When the debt became due, both Carmen and (D) unconstitutional, because it imposes
Pater refused to repay it, or any part of it. an unreasonable burden on interstate
commerce.
In an action by Tertius against Pater to recover
$1,000 plus interest, which of the following 558. Miller was indicted in a state court in January
statements would summarize Pater’s best 1985 for a robbery and murder that occurred in
defense? December 1982. He retained counsel, who filed
a motion to dismiss on the ground that Miller
(A) He received no consideration for his had been prejudiced by a 25-month delay in
conditional promise to Tertius. obtaining the indictment. Thereafter, Miller, with
(B) His conditional promise to Tertius was not his counsel, appeared in court for arraignment
to be performed in less than a year from and stated that he wished to plead guilty.
the time it was made.
(C) His conditional promise to Tertius was not The presiding judge asked Miller whether he
made for the primary purpose of benefiting understood the nature of the charges, possible
himself (Pater). defenses, and maximum allowable sentences.
(D) The loan by Tertius was made without Miller replied that he did, and the judge
any agreement concerning the applicable reviewed all of those matters with him. He then
interest rate. asked Miller whether he understood that he did
not have to plead guilty. When Miller responded
557. After several well-publicized deaths caused by that he knew that, the judge accepted the plea
fires in products made from highly flammable and sentenced Miller to 25 years.
fabrics, the state of Orange enacted a statute
prohibiting “the manufacture or assembly of any Six months later, Miller filed a motion to set
product in this state which contains any fabric aside his guilty plea on each of the following
that has not been tested and approved for flame grounds.
retardancy by the Zetest Testing Company.” The
Zetest Testing Company is a privately owned Which of these grounds provides a constitutional
and operated business located in Orange. basis for relief?

For many years, Fabric Mill, located in the state (A) The judge did not rule on his motion to
of Orange, has had its fabric tested for flame dismiss before accepting the guilty plea.
retardancy by the Alpha Testing Company, (B) The judge did not determine that Miller
located in the state of Green. Alpha Testing had robbed and killed the victim.
Company is a reliable organization that uses (C) The judge did not determine whether
a process for testing and approving fabrics for Miller understood that he had a right to
flame retardancy identical in all respects to that jury trial.
used by the Zetest Testing Company. (D) The judge did not determine whether the
prosecutor’s file contained any undisclosed
exculpatory material.

134
559. Paul sued Dyer for personal injuries sustained There is no applicable statute or ordinance (other
when Dyer’s car hit Paul, a pedestrian. than those dealing with various approvals for zoning,
Immediately after the accident, Dyer got out building, etc.).
of his car, raced over to Paul, and said, “Don’t
worry—I’ll pay your hospital bill.” 561. After Dora had torn down the existing building,
she proceeded to excavate deeper. Dora used
Paul’s testimony concerning Dyer’s statement is shoring that met all local, state, and federal
safety regulations, and the shoring was placed in
(A) admissible, because it is an admission of accordance with those standards.
liability by a party opponent.
(B) admissible, because it is within the excited Pam notified Dora that cracks were developing
utterance exception to the hearsay rule. in the building situated on Pam’s lot. Dora took
(C) inadmissible to prove liability, because it is the view that any subsidence suffered by Pam
an offer to pay medical expenses. was due to the weight of Pam’s building, and
(D) inadmissible, provided that Dyer kept his correctly asserted that none would have occurred
promise to pay Paul’s medical expenses. had Pam’s soil been in its natural state. Dora
continued to excavate.
560. As a result of an accident at the NPP nuclear
power plant, a quantity of radioactive vapor The building on Pam’s lot suffered extensive
escaped from the facility, and two members damage, requiring the expenditure of $750,000
of the public were exposed to excessive doses to remedy the defects.
of radiation. According to qualified medical
opinion, that exposure will double the chance Which of the following is the best comment
that these two persons will ultimately develop concerning Pam’s action to recover damages
cancer. However, any cancer that might be from Dora?
caused by this exposure will not be detectable
for at least 10 years. If the two exposed persons (A) Dora is liable, because she removed
do develop cancer, it will not be possible necessary support for Pam’s lot.
to determine whether it was caused by this (B) Dora cannot be held liable simply upon
exposure or would have developed in any event. proof that support was removed, but may
be held liable if negligence is proved.
If the exposed persons assert a claim for (C) Once land is improved with a building,
damages against NPP shortly after the escape of the owner cannot invoke the common law
the radiation, which of the following questions right of lateral support.
will NOT present a substantial issue? (D) Dora’s only obligation was to satisfy all
local, state, and federal safety regulations.
(A) Will the court recognize that the plaintiffs
have suffered a present legal injury? 562. Assume that no problems with subsidence or
(B) Can the plaintiffs prove the amount of their other misadventures occurred during construction
damages? of Dora’s new building. However, when it was
(C) Can the plaintiffs prove that any harm they completed, Pam discovered that the shadow
may suffer was caused by this exposure? created by the new higher building placed her
(D) Can the plaintiffs prevail without building in such deep shade that her ability to
presenting evidence of specific negligence lease space was diminished and that the rent
on the part of NPP? she could charge and the occupancy rate were
substantially lower. Assume that these facts are
proved in an appropriate action Pam instituted
Questions 561–562 are based on the following fact against Dora for all and any relief available.
situation.
Which of the following is the most appropriate
Pam and Dora own adjoining lots in the central comment concerning this lawsuit?
portion of a city. Each of their lots had an office
building. Dora decided to raze the existing building on (A) Pam is entitled to a mandatory injunction
her lot and to erect a building of greater height, and requiring Dora to restore conditions to those
she received all governmental approvals required to existing with the prior building insofar as
pursue her project. the shadow is concerned.

135
(B) The court should award permanent damages, Doris agreed and also promised John that she
in lieu of an injunction, equal to the would not tell anyone of their plans. Unknown
present value of all rents lost and loss on to Doris, John had no intention of marrying
rents for the reasonable life of the building. her. After Doris came to his apartment, John
(C) The court should award damages for losses told Doris he was going for cigarettes. He
suffered to the date of trial and leave open called Doris’s father and told him that he had
recovery of future damages. his daughter and would kill her if he did not
(D) Judgment should be for Dora, because Pam receive $100,000. John was arrested on Sunday
has no cause of action. afternoon when he went to pick up the $100,000.
Doris was still at the apartment and knew
nothing of John’s attempt to get the money.

563. A city ordinance makes the city building John is guilty of


inspector responsible for ensuring that all
buildings in that city are kept up to building code (A) kidnapping.
standards and requires the inspector to refer for (B) attempted kidnapping.
prosecution all known building code violations. (C) kidnapping or attempted kidnapping but
Another ordinance provides that the city building not both.
inspector may be discharged for “good cause.” (D) neither kidnapping nor attempted
The building inspector took a newspaper reporter kidnapping.
through a number of run-down buildings in a
slum neighborhood. After using various epithets 565. In a civil action for personal injury, Payne
and slurs to describe the occupants of these alleges that he was beaten up by Dabney during
buildings, the building inspector stated to the an altercation in a crowded bar. Dabney’s
reporter: “I do not even try to get these buildings defense is that he was not the person who hit
up to code or to have their owners prosecuted Payne. To corroborate his testimony about the
for code violations because if these buildings are cause of his injuries, Payne seeks to introduce,
repaired, the people who live in them will just through the hospital records custodian, a
wreck them again.” The reporter published these notation in a regular medical record made by an
statements in a story in the local newspaper. emergency room doctor at the hospital where
The building inspector admitted he made the Payne was treated for his injuries. The notation
statements. is: “Patient says he was attacked by Dabney.”

On the basis of these statements, the city council The notation is


discharged the building inspector.
(A) inadmissible, unless the doctor who made
Is the action of the city council constitutional? the record is present at trial and available
(A) Yes, because the statements demonstrate for cross-examination.
that the building inspector has an attitude (B) inadmissible as hearsay, not within any
toward a certain class of persons that exception.
interferes with the proper performance of (C) admissible as hearsay, within the exception
the obligations of his job. for records of regularly conducted activity.
(B) Yes, because the building inspector is (D) admissible as a statement made for the
a government employee and a person purpose of medical diagnosis or treatment.
holding such a position may not make
public comments inconsistent with current
governmental policy. Questions 566–567 are based on the following fact
(C) No, because the statements were lawful situation.
comments on a matter of public concern.
(D) No, because the statements were published After extensive negotiations, Tune Corporation, a
in a newspaper that is protected by the radio manufacturer, and Bill’s Comex, Inc., a retailer,
First and Fourteenth Amendments. entered into a final written agreement in which
Tune agreed to sell and Bill’s agreed to buy all of
564. John asked Doris to spend a weekend with him its requirements of radios, estimated at 20 units per
at his apartment and promised her that they month, during the period January 1, 1988, through
would get married on the following Monday. December 31, 1990, at a price of $50 per unit. A
dispute arose in late December 1990, when Bill’s

136
returned 25 undefective radios to Tune for full credit (C) By depositing the check without protest
after Tune had refused to extend the contract for a and with knowledge of its wording, Tune
second three-year period. entered into a novation discharging any
remaining duty to pay on the part of Bill’s.
In an action by Tune against Bill’s for damages due (D) The parties’ good-faith dispute over return
to return of the 25 radios, Tune introduces the written of the radios suspended the duty of Bill’s,
agreement, which expressly permitted the buyer to if any, to pay any balance due.
return defective radios for credit but was silent as to
return of undefective radios for credit. Bill’s seeks to
introduce evidence that during the three years of the
agreement it had returned, for various reasons, 125 568. While driving at a speed in excess of the
undefective radios, for which Tune had granted full statutory limit, Dant negligently collided with
credit. Tune objects to the admissibility of this evidence. another car, and the disabled vehicles blocked
two of the highway’s three northbound lanes.
566. The trial court will probably rule that the When Page approached the scene two minutes
evidence proffered by Bill’s is later, he slowed his car to see if he could help
those involved in the collision. As he slowed, he
(A) inadmissible, because the evidence is was rear-ended by a vehicle driven by Thomas.
barred by the parol evidence rule. Page, who sustained damage to his car and was
(B) inadmissible, because the express terms seriously injured, brought an action against Dant
of the agreement control when those to recover damages. The jurisdiction adheres to
terms are inconsistent with the course of the traditional common law rules pertaining to
performance. contributory negligence.
(C) admissible, because the evidence supports
an agreement that is not within the relevant If Dant moves to dismiss the action for failure to
statute of frauds. state a claim upon which relief may be granted,
(D) admissible, because course-of-performance should the motion be granted?
evidence, when available, is considered the
best indication of what the parties intended (A) Yes, because it was Thomas, not Dant,
the writing to mean. who collided with Page’s car and caused
Page’s injuries.
567. For this question only, assume the following (B) Yes, if Page could have safely passed the
facts. When Bill’s returned the 25 radios in disabled vehicles in the traffic lane that
question, it included with the shipment a check remained open.
payable to Tune for the balance admittedly due (C) No, because a jury could find that
on all other merchandise sold and delivered to Page’s injury arose from a risk that was
Bill’s. The check was conspicuously marked, a continuing consequence of Dant’s
“Payment in full for all goods sold to Bill’s negligence.
to date.” Tune’s credit manager, reading this (D) No, because Dant was driving in excess
check notation and knowing that Bill’s had also of the statutory limit when he negligently
returned the 25 radios for full credit, deposited caused the first accident.
the check without protest in Tune’s local bank
account. The canceled check was returned to 569. Owen owned Greenacre in fee simple. The small
Bill’s a week later. house on Greenacre was occupied, with Owen’s
oral permission, rent-free, by Able, Owen’s son,
Which of the following defenses would best and Baker, a college classmate of Able. Able
serve Bill’s? was then 21 years old.

(A) Tune’s deposit of the check and its return Owen, by properly executed instrument,
to Bill’s after payment estopped Tune conveyed Greenacre to “my beloved son,
thereafter to assert that Bill’s owed any Able, his heirs and assigns, upon the condition
additional amount. precedent that he earn a college degree by the
(B) By depositing the check without protest time he reaches the age of 30. If, for any reason,
and with knowledge of its wording, Tune he does not meet this condition, then Greenacre
discharged any remaining duty to pay on shall become the sole property of my beloved
the part of Bill’s. daughter, Anna, her heirs and assigns.” At the
time of the conveyance, Able and Baker attended

137
a college located several blocks from Greenacre. court rulings with respect to the constitutionality,
Neither had earned a college degree. interpretation, and application of the statute
might adversely affect or delay its enforcement
One week after the delivery of the deed to Able, and, thereby, jeopardize the public health. As a
Able recorded the deed and immediately told result, they included a provision in the statute
Baker that he, Able, was going to begin charging providing that all legal challenges concerning
Baker rent since “I am now your landlord.” There those matters were to be initiated only by filing
is no applicable statute. suit directly in the United States Supreme Court.

Able and Baker did not reach agreement, and The provision authorizing direct review of the
Able served the appropriate notice to terminate constitutionality, interpretation, or application
whatever tenancy Baker had. Able then sought, of this statute only in the United States Supreme
in an appropriate action, to oust Baker. Court is

Who should prevail? (A) constitutional, because it is authorized by


the Article I power of Congress to enact
(A) Able, because the conveyance created a fee all laws that are “necessary and proper” to
simple subject to divestment in Able. implement the general welfare.
(B) Able, because Owen’s conveyance (B) constitutional, because Article III
terminated Baker’s tenancy. provides that the jurisdiction of the
(C) Baker, because Owen’s permission to United States Supreme Court is subject to
occupy preceded Owen’s conveyance to such exceptions and such regulations as
Able. Congress shall make.
(D) Baker, because Baker is a tenant of Owen, (C) unconstitutional, because it denies persons
not of Able. who wish to challenge this statute the
equal protection of the laws by requiring
570. Testator, whose nephew Bypast was his only them to file suit in a court different
heir, died leaving a will that gave his entire from that in which persons who wish to
estate to charity. Bypast, knowing full well that challenge other statutes may file suit.
Testator was of sound mind all of his life, and (D) unconstitutional, because it is inconsistent
having no evidence to the contrary, nevertheless with the specification in Article III of the
filed a suit contesting Testator’s will on the original jurisdiction of the United States
ground that Testator was incompetent when the Supreme Court.
will was signed. Craven, Testator’s executor,
offered Bypast $5,000 to settle the suit, and
Bypast agreed. Questions 572–573 are based on the following fact
situation.
If Craven then repudiates the agreement and
the foregoing facts are proved or admitted in Morten was the general manager and chief executive
Bypast’s suit against Craven for breach of officer of the Woolen Company, a knitting mill.
contract, is Bypast entitled to recover under the
prevailing view? Morten delegated all operational decision making to
Grouse, the supervising manager of the mill. The child
(A) Yes, because the Bypast-Craven agreement labor laws in the jurisdiction provide, “It is a violation
was a bargained-for exchange. of the law for one to employ a person under the age
(B) Yes, because the law encourages the of 17 years for full-time labor.” Without Morten’s
settlement of disputed claims. knowledge, Grouse hired a number of 15- and 16-
(C) No, because Bypast did not bring the will year-olds to work at the mill full time. He did not
contest in good faith. ask their ages and they did not disclose them. Grouse
(D) No, because an agreement to oust the court could have discovered their ages easily by asking for
of its jurisdiction to decide a will contest is identification, but he did not do so because he was not
contrary to public policy. aware of the law and believed that company policy
was to hire young people.
571. The National AIDS Prevention and Control
Act is a new comprehensive federal statute that 572. If the statute is interpreted to create strict
was enacted to deal with the public health crisis liability and Grouse is charged with violating it,
caused by the AIDS virus. Congress and the Grouse is
President were concerned that inconsistent lower

138
(A) guilty, because he should have inquired as mesh fence. On several occasions, Dooley turned and
to the ages of the children. looked directly at the hecklers with a scowl on his
(B) guilty, because he hired the children. face, but the heckling continued. Dooley wound up as
(C) not guilty, because in law the Woolen though he was preparing to pitch in the direction of
Company, not Grouse, is the employer of his catcher; however, the ball traveled from his hand,
the children. at high speed, at a 90-degree angle from the line to the
(D) not guilty, because he believed he was catcher and directly toward the hecklers in the stands.
following company policy and was not The ball passed through the wire mesh fence and
aware of the violation. struck Patricia, one of the hecklers.

573. If the statute is interpreted to create strict Patricia brought an action for damages against
liability and Morten is convicted of violating it, Dooley and the City Robins, based upon negligence
his contention that his conviction would violate and battery. The trial court directed a verdict for the
the federal Constitution is defendants on the battery count. The jury found for the
defendants on the negligence count because the jury
(A) correct, because it is a violation of due determined that Dooley could not foresee that the ball
process to punish without a voluntary act. would pass through the wire mesh fence.
(B) correct, because criminal liability is
personal and the Woolen Company is the Patricia has appealed the judgments on the battery
employer of the children, not Morten. counts, contending that the trial court erred in
(C) incorrect, because regulatory offenses are directing verdicts for Dooley and the City Robins.
not subject to due process limitations.
(D) incorrect, because he was in a position 575. On appeal, the judgment entered on the directed
to exercise control over the hiring of verdict in Dooley’s favor on the battery claim
employees for the Woolen Company. should be

(A) affirmed, because the jury found on the


evidence that Dooley could not foresee that
574. Dexter is being tried for the homicide of a girl the ball would pass through the fence.
whose strangled body was found beside a remote (B) affirmed, if there was evidence that Dooley
logging road with her hands taped together. was mentally ill and that his act was the
After Dexter has offered evidence of alibi, the product of his mental illness.
state calls Wilma to testify that Dexter had taped (C) reversed and the case remanded, if a jury
her hands and tried to strangle her in the same could find on the evidence that Dooley
location two days before the homicide but that intended to cause the hecklers to fear being
she escaped. hit.
(D) reversed and the case remanded, because a
The evidence is jury could find that Dooley’s conduct was
extreme and outrageous, and the cause of
(A) admissible as tending to show that Dexter physical harm to Patricia.
is the killer.
(B) admissible as tending to show Dexter’s 576. For this question only, assume that, on appeal,
violent nature. the court holds that the question of whether
(C) inadmissible, because it is improper Dooley committed a battery is a jury issue.
character evidence.
(D) inadmissible, because it is unfairly The judgment entered on the directed verdict in
prejudicial. favor of the City Robins should then be

(A) reversed and the case remanded, because a


Questions 575–576 are based on the following fact jury could find the City Robins vicariously
situation. liable for a battery committed by Dooley in
the course of his employment.
Dooley was a pitcher for the City Robins, a (B) reversed and the case remanded, only if a
professional baseball team. While Dooley was jury could find negligence on the part of
throwing warm-up pitches on the sidelines during a the Robins team management.
game, he was continuously heckled by some spectators (C) affirmed, because an employer is not
seated in the stands above the dugout behind a wire vicariously liable for an employee’s
battery.

139
(D) affirmed, if Dooley’s act was a knowing (A) even if he misled the store owner, he
violation of team rules. intended to pay for the items.
(B) he did not misrepresent any material fact.
(C) the store owner got his property back and
so suffered no harm.
577. Small retailers located in the state of Yellow are (D) the store owner could have asked for
concerned about the loss of business to certain payment in full at the time of the purchase.
large retailers located nearby in bordering
states. In an effort to deal with this concern, the 579. Which of the following items of evidence
legislature of Yellow enacted a statute requiring is LEAST likely to be admitted without a
all manufacturers and wholesalers who sell supporting witness?
goods to retailers in Yellow to do so at prices
that are no higher than the lowest prices at which (A) In a libel action, a copy of a newspaper
they sell them to retailers in any of the states purporting to be published by Defendant
that border Yellow. Several manufacturers and Newspaper Publishing Company.
wholesalers who are located in states bordering (B) In a case involving contaminated food, a
Yellow and who sell their goods to retailers in can label purporting to identify the canner
those states and in Yellow bring an action in as Defendant Company.
federal court to challenge the constitutionality of (C) In a defamation case, a document
this statute. purporting to be a memorandum from
the Defendant Company president to
Which of the following arguments offered by “All Personnel,” printed on Defendant
these plaintiffs is likely to be most persuasive in Company’s letterhead.
light of applicable precedent? (D) In a case involving injury to a pedestrian,
a pamphlet on stopping distances issued by
The state statute the State Highway Department.

(A) deprives them of their property or liberty 580. A federally owned and operated office building
without due process of law. in the state of West Dakota is heated with a
(B) imposes an unreasonable burden on new pollution-free heating system. However, in
interstate commerce. the coldest season of the year, this new system
(C) deprives them of a privilege or immunity is sometimes insufficient to supply adequate
of national citizenship. heat to the building. The appropriation statute
(D) denies them the equal protection of the providing the money for construction of the
laws. new heating system permitted use of the old
pollution-generating system when necessary to
578. Smith is a new lawyer who has three clients, supply additional heat. When the old heating
all of whom are indigent. To improve the system operates (only about two days in any
appearance of his office, he decided to purchase year), the smokestack of the building emits
some new furniture and to pay for it out of smoke that exceeds the state of West Dakota’s
future earnings. Wearing an expensive suit pollution-control standards.
borrowed from a friend, Smith went to a
furniture store and asked to purchase on credit a May the operators of the federal office building
desk and various other items of furniture. Smith be prosecuted successfully by West Dakota
told the store owner that he was a very able authorities for violating that state’s pollution-
lawyer with a growing practice and that he control standards?
expected to do very well in the future. The store
owner agreed to sell him the items on credit, and (A) Yes, because the regulation of pollution is
Smith promised to make monthly payments of a legitimate state police power concern.
$800. Smith has never had an income from his (B) Yes, because the regulation of pollution is
practice of more than $150 a month. Smith’s a joint concern of the federal government
business did not improve, and he did not make and the state and, therefore, both of them
any payments to the furniture store. After three may regulate conduct causing pollution.
months, the store owner repossessed the items. (C) No, because the operations of the federal
government are immune from state
If Smith is charged with obtaining property by regulation in the absence of federal
false pretenses, hibest argument for being found consent.
NOT guilty would be that

140
(D) No, because the violations of the state
pollution-control standards involved here
are so deminimized that they are beyond
the legitimate reach of state law.

581. Dieter parked her car in violation of a city


ordinance that prohibits parking within 10 feet
of a fire hydrant. Because Grove was driving
negligently, his car sideswiped Dieter’s parked
car. Plaintiff, a passenger in Grove’s car, was
injured in the collision.

If Plaintiff asserts a claim against Dieter to


recover damages for his injuries, basing his
claim on Dieter’s violation of the parking
ordinance, will Plaintiff prevail?

(A) Yes, because Dieter was guilty of


negligence per se.
(B) Yes, if Plaintiff would not have been
injured had Dieter’s car not been parked
where it was.
(C) No, because Dieter’s parked car was not
an active or efficient cause of Plaintiff’s
injury.
(D) No, if prevention of traffic accidents was
not a purpose of the ordinance.

141
142
Answer Key
1. B 55. A 109. B 163. C
2. D 56. A 110. D 164. B
3. C 57. C 111. A 165. D
4. B 58. C 112. C 166. D
5. C 59. D 113. A 167. C
6. C 60. A 114. B 168. D
7. A 61. C 115. A 169. A
8. A 62. D 116. C 170. B
9. D 63. B 117. D 171. A
10. D 64. C 118. C 172. C
11. C 65. D 119. C 173. D
12. B 66. C 120. A 174. A
13. A 67. B 121. C 175. C
14. D 68. D 122. B 176. B
15. C 69. C 123. A 177. C
16. B 70. A 124. D 178. B
17. A 71. B 125. D 179. B
18. D 72. D 126. B 180. C
19. C 73. A 127. A 181. A
20. C 74. C 128. B 182. C
21. A 75. D 129. C 183. C
22. B 76. B 130. B 184. D
23. B 77. A 131. D 185. A
24. D 78. C 132. A 186. C
25. A 79. B 133. B 187. C
26. C 80. C 134. D 188. B
27. A 81. B 135. C 189. A
28. B 82. D 136. A 190. D
29. C 83. A 137. A 191. D
30. B 84. D 138. C 192. C
31. A 85. A 139. D 193. B
32. A 86. B 140. A 194. A
33. D 87. B 141. C 195. D
34. D 88. C 142. D 196. A
35. A 89. D 143. D 197. D
36. B 90. B 144. C 198. C
37. C 91. B 145. D 199. D
38. B 92. C 146. C 200. B
39. C 93. A 147. B 201. B
40. C 94. C 148. A 202. B
41. D 95. B 149. A 203. B
42. C 96. D 150. B 204. D
43. C 97. B 151. B 205. B
44. D 98. C 152. B 206. B
45. B 99. D 153. D 207. C
46. D 100. C 154. C 208. B
47. D 101. D 155. C 209. C
48. A 102. B 156. D 210. B
49. D 103. A 157. A 211. C
50. C 104. B 158. A 212. A
51. B 105. A 159. B 213. A
52. C 106. A 160. C 214. D
53. B 107. D 161. A 215. D
54. D 108. B 162. A 216. A

143
217. C 274. A 331. C 388. C
218. B 275. D 332. B 389. D
219. B 276. D 333. C 390. D
220. D 277. B 334. B 391. C
221. C 278. A 335. A 392. D
222. D 279. A 336. B 393. A
223. B 280. C 337. D 394. C
224. C 281. D 338. B 395. D
225. C 282. C 339. D 396. B
226. D 283. A 340. D 397. D
227. B 284. D 341. C 398. C
228. C 285. C 342. D 399. A
229. D 286. C 343. C 400. C
230. B 287. C 344. D 401. D
231. C 288. A 345. D 402. D
232. B 289. B 346. C 403. D
233. B 290. B 347. B 404. C
234. C 291. B 348. C 405. D
235. D 292. D 349. A 406. C
236. B 293. B 350. D 407. D
237. C 294. B 351. D 408. D
238. A 295. A 352. A 409. A
239. C 296. D 353. B 410. B
240. D 297. C 354. C 411. C
241. D 298. D 355. C 412. B
242. A 299. A 356. B 413. A
243. C 300. B 357. C 414. C
244. C 301. D 358. A 415. A
245. A 302. A 359. B 416. D
246. C 303. D 360. D 417. C
247. A 304. A 361. B 418. C
248. B 305. B 362. D 419. D
249. A 306. A 363. A 420. C
250. D 307. B 364. C 421. A
251. B 308. D 365. A 422. D
252. A 309. A 366. B 423. C
253. B 310. B 367. A 424. C
254. A 311. C 368. C 425. B
255. C 312. A 369. A 426. B
256. A 313. B 370. D 427. B
257. A 314. C 371. A 428. D
258. B 315. D 372. D 429. D
259. D 316. B 373. B 430. A
260. B 317. B 374. C 431. A
261. C 318. D 375. B 432. C
262. A 319. C 376. D 433. B
263. B 320. D 377. A 434. A
264. C 321. C 378. D 435. A
265. B 322. A 379. B 436. D
266. A 323. D 380. C 437. B
267. C 324. B 381. A 438. D
268. D 325. A 382. B 439. A
269. C 326. B 383. A 440. B
270. A 327. C 384. A 441. D
271. B 328. B 385. D 442. D
272. B 329. C 386. C 443. C
273. B 330. A 387. B 444. A

144
445. D 502. C 559. C
446. C 503. A 560. D
447. A 504. D 561. B
448. B 505. D 562. D
449. A 506. B 563. A
450. D 507. B 564. D
451. C 508. D 565. B
452. C 509. D 566. D
453. D 510. B 567. B
454. B 511. B 568. C
455. A 512. C 569. D
456. A 513. A 570. C
457. B 514. B 571. D
458. A 515. A 572. B
459. D 516. D 573. D
460. A 517. D 574. A
461. D 518. B 575. C
462. A 519. C 576. A
463. B 520. D 577. B
464. C 521. C 578. B
465. C 522. B 579. C
466. B 523. C 580. C
467. B 524. B 581. D
468. A 525. A
469. A 526. A
470. A 527. C
471. C 528. A
472. D 529. A
473. C 530. B
474. A 531. D
475. D 532. B
476. A 533. C
477. B 534. D
478. D 535. D
479. C 536. B
480. D 537. C
481. B 538. A
482. D 539. A
483. A 540. A
484. A 541. D
485. B 542. D
486. B 543. B
487. D 544. C
488. A 545. A
489. C 546. C
490. A 547. B
491. A 548. D
492. D 549. B
493. A 550. D
494. C 551. A
495. A 552. A
496. A 553. D
497. D 554. B
498. B 555. B
499. D 556. C
500. D 557. D
501. A 558. C

145

S-ar putea să vă placă și